Big path MCQ Flashcards

1
Q

Alcoholic having treatment for Wernickes has rapid change in heart size over one
week. Change most likely due to ? Rob p839
1.Dehydration
2.Resolution of pericardial effusion
3.Projectional change on CXR
4.Beri-beri heart disease

A

Beri-beri heart disease this is caused by vit B1 (Thiamine) deficiency

How well did you know this?
1
Not at all
2
3
4
5
Perfectly
2
Q

Patient with drug resistant Parkinsons and autonomic neuorpathy ? Rob 844
1.Shy Drager Syndrome
2.Drug resistant Parkinsons
3.Striatnigeral degeneration
4.Olviopontocerebellar atrphy
5.Progressive supranuclear palsy
6.Huntingtons

A

Shy Drager Syndrome – type of MSA

MSA C – cerebellar. Pons, medulla and cerebellum small. -> hotcross bun (pontine)
MSA P – extrapyramidal. Low T2 + high T1 in putamen
MSA A – autonomic (Shy dragger).
PSP – Atypical parkinsons. Midbrain atrophy -> hummingbird. Midbrain : pons area calculation

How well did you know this?
1
Not at all
2
3
4
5
Perfectly
3
Q

Schwannoma versus plexiform neurofibroma ?

A

Schwannoma - Round or lobulated well delineated encapsulated tumours arising eccentrically from parent nerve and can be separated from it (c/w plexiform neurofibromas)

nf2 not nf1. spinal nerves usually not cutaneous and spinal. no malignant degen.

How well did you know this?
1
Not at all
2
3
4
5
Perfectly
4
Q

Child with post paravertebral mass, biopsy shows acute neural elements and schwann
cells not attached to nerve – DIAGNOSIS ?
1.Ganglioneuroma
2.Ganglioglioma
3.Neurofibroma
4.Schwannoma
5.Neuroblastoma

A

answer: Ganglioneuroma (Neurogenic neoplasm of sympathetic ganglia. Nerve fibres, schwann
cells, mature ganglion cells and mucous matrix)

2.Ganglioglioma
3.Neurofibroma (Each fascicle is infiltrated by neoplasm – not possible to separate lesion
from nerve)
4.Schwannoma (most common in 5th to 6th decade except in NF-2)
5.Neuroblastoma (Malignant tumour of sympathetic chain. Small round blue cells +
schwannian stroma cells)

How well did you know this?
1
Not at all
2
3
4
5
Perfectly
5
Q

Periventricular mass in patient with renal transplant
1.Lymphoma – 1°
2.GBM
3.Lymphoma – 2°

A

Lymphoma – 1° (Most common in immunosuppressed patients)

How well did you know this?
1
Not at all
2
3
4
5
Perfectly
6
Q

Cystic tumour in brain LEAST LIKELY is ?
1.Haemangioblastoma
2.JPA
3.PNET
4.Meningioma
5.Schwannoma
6.DNET

A

answer: Meningioma (Cystic or necrotic change may be present - most often in parasagittal
tumours (3 – 14%))

  1. Haemangioblastoma (60% are cystic masses with mural nodule that usually abuts pial
    surface)
    2.JPA (Cerebeallar JPA 30% of total of JPA - Well-circumscribed mass with large cyst, and
    small reddish-tan mural nodule)
    3.PNET (commonly cystic)
    5.Schwannoma (cystic change is common)
    6.DNET (Well-defined “pseudocystic” lesion (high water content))
How well did you know this?
1
Not at all
2
3
4
5
Perfectly
7
Q

CJD & variant CJD
1.caused by a slow virus
2.CJD patients live for <12 months, vCJD can live for a few years.
3.Associated with frontal atrophy.

A

CJD patients live for <12 months, vCJD can live for a few years.

How well did you know this?
1
Not at all
2
3
4
5
Perfectly
8
Q

Pilocytic Astrocytoma
1.Associated with NF2
2.50% are solid
3.Prognosis is less than 70% 5 year survival rate
4.Rosenthal fibres

A

Rosenthal fibres often present = eosinophilic bodies within astrocyte processes
up to 100% 5YS
Rosenthal fibers
Pilocytic astrocytoma
glioma
pineal tumor
Alexander’s disease
long-standing gliosis

How well did you know this?
1
Not at all
2
3
4
5
Perfectly
9
Q

Least common site for meningioma:
1.adjacent to hippocampus
2.parietal lobes
3.between cerebrum & cerebellum
4.adjacent to nose

A

adjacent to nose

Most common
Hemispheric convexity (20%)
Parasagittal (25%) → may occlude SSS

Very common
Sphenoid ridge, wing (15 – 20%) → may involve optic canal, wing meningiomas often en plaque –
extensive dural involvement. Usually extra-cranial extension into calvarium, orbit or soft tissue
Olfactory groove (5 – 10%)

Common
Parasellar (5 – 10%) CR p79 Case 63
Cavernous sinus
66% partially or totally encase carotid artery
33% narrow the artery

Less common
CPA, along clivus (posterior fossa – 10%)
Tentorium cerebelli
Foramen magnum

Rare
Optic nerve sheath (<2%)
Extracranial (nose, sinuses, skull (intraosseous))
Intraventricular
usually trigone of (L) lateral ventricle
most common trigonal mass in adults
Spinal canal (M : F = 1:10 )
mostly thoracic region
Sylvian fissure
Paediatric age group

How well did you know this?
1
Not at all
2
3
4
5
Perfectly
10
Q

Least likely site for hypertensive bleed in the brain is:
1.hippocampus
2.cerebellum
3.basal ganglia
4.thalamus

A

hippocampus

How well did you know this?
1
Not at all
2
3
4
5
Perfectly
11
Q

Which is least likely to involve the corpus callosum: (GC)
1.GBM
2.Marchifava Bignami
3.DAI
4.Dandy Walker
5.Lymphoma

A

4.Dandy Walker F - assocd with dysgenesis of the CC in 20-25% (cf. primary involvemt)

1.GBM T - most commonly spread via direct extension along WM tracts, including the CC -
classic butterfly pattern.
2.Marchifava Bignami T - primarily affects the CC - acute form affects the genu &
splenium, chronic form affects the body.
3.DAI T - classic triad of GW junction, dorsolateral brainstem, and CC (most commonly
eccentrically and in the splenium).
5.Lymphoma T - differ from GBM as usually less peritumoral oedema,

How well did you know this?
1
Not at all
2
3
4
5
Perfectly
12
Q

Hashimoto’s – FNA findings (TW)
1.Hurthle cells
2.Fibrosing nodules
3.Psammoma bodies

A

Hurthle cells -T - mononuclear inflammatory infiltrate containing small lymphocytes,
plasma cells, and well-developed germinal centers. The thyroid follicles are small and are
lined in many areas by epithelial cells with abundant eosinophilic, granular cytoplasm,
termed Hurthle cells.

Hurthle cells seen in:
Hashimotos
Follicular adenoma

2.Fibrosing nodules - F - Reidel’s thyroiditis
3.Psammoma bodies - F - in papillary thyroid carcinoma. Concentrically calcified
structures.

How well did you know this?
1
Not at all
2
3
4
5
Perfectly
13
Q

Which is not a feature of Alzheimer’s: (GC)
1.Hirano bodies
2.Lewy Bodies
3.Senile Plaques
4.Neurofibrillary tangles
5.Granulovacuolar degeneration
6.Amyloid

A

Lewy Bodies F - eosinophilic intracytoplasmic inclusions found in some neurones in
Parkinson’s disease.

How well did you know this?
1
Not at all
2
3
4
5
Perfectly
14
Q

.PNET, which is the most typical appearance:
1.Cortical
2.Angiogenesis
3.Cystic
4.Vasogenic oedema
5.Astrocytoma

A

Angiogenesis T - WHO grade IV. Supratentorial PNETs had highly branched capillaries with extensive endothelial cell hyperplasia. Glomeruloid arrays of microvessels extended from the capillaries. Small fragments of endothelial tubes were scattered throughout the tumor.

How well did you know this?
1
Not at all
2
3
4
5
Perfectly
15
Q

In HSV I encephalitis, which is least correct (TW)
1.Age 50-70 years
2.Typically involves superior frontal lobes
3.Common presentation is headache
4.May present with seizures,

A

Typically involves superior frontal lobes - F - abnormal signal and enhancemen t of
medial temporal and inferior frontal lobes. Affects limibic system: temporal lobes, insula,
subfrontal area and cingulate gyri typical.

Age 50-70 years - T - can occur at any age, with highest incidence in adolescents and
young adults. Bimodal distribution by age with 1st peak occurring younger than 20y
(primary infection), and second occuring in those older than 50y (reactivation of latent
infection).
3.Common presentation is headache - T - fever, headache, seizures, +/- viral prodrome.
4.May present with seizures, ataxia and lethargy - T - altered mental status, focal or diffuseneurologic deficit (<30%)

How well did you know this?
1
Not at all
2
3
4
5
Perfectly
16
Q

Child with posterior paravertebral mass, biopsy shows mature neural elements and
Schwann cells not attached to nerve – diagnosis is (TW)
1.Ganglioneuroma
2.Ganglioglioma
3.Neurofibroma
4.Schwannoma
5.Neuroblastoma

A

1.Ganglioneuroma - T - The most well-differentiation lesions (in the neuroblastoma
spectrum) - see ganglion cells and Schwann cells, and neuroblasts are no longer present.

2.Ganglioglioma - F - tumor of neoplastic astrocytes (rarely oligodendrocytes) and ganglioncells.
3.Neurofibroma - F - PNST arise from cells of the peripheral nerve (Schwann cells,
perineural cells, fibroblasts) - attached to nerve, but can be separated from it
4.Schwannoma - F - PNST - see ans 3.
5.Neuroblastoma - F- small, primitive-appearing cells with dark nuclei, scant cytoplasm,and poorly defined cell borders growing in solid sheets. Certain NBs may have some degreeof differentiation with clusters of larger cells resembling ganglion cells.

How well did you know this?
1
Not at all
2
3
4
5
Perfectly
17
Q

Child with mass FNA shows small round blue cells- least likely diagnosis is (TW)
1.Neuroblastom
2.Ewing’s sarcoma
3.Rhabdomyosarcoma
4.Wilms tumour
5.Retinoblastoma

A

Wilms tumour - F - classic Wilms tumor comprised of 3 cell types - Blastemal cells
(undifferentiated cells), Stromal cells (immature spindle cells and heterologous skeletal, cartilage, osteoid, or fat), and epithelial cells (Glomeruli and tubules).

1.Neuroblastoma - T - small, primitive-appearing cells with dark nuclei, scant cytoplasm,and poorly defined cell borders growing in solid sheets.
2.Ewing’s sarcoma - T - monotonous sheets of small round blue cells with hyperchromatic
nuclei and scant cytoplasm.
3.Rhabdomyosarcoma - T - subtypes Botryoid and spindle cell (leiomyomatous / Embryonal
/ Alveolar / Undifferentiated. Histo of embryonal and alveolar types - cells have scant cytoplasm and a centrally placed round nucleus that occupies the majority of the cell.
5.Retinoblastoma - T - sheets, trabeculae and nests of small blue cells with scant cytoplasm.
Ewing’s sarcoma family of tumors (EFT) includes Ewings sarcoma, extraosseous Ewing’s sarcoma,more differentiated neuroectodermal tumors (PNET: previously AKA neuroepithelioma, adultneuroblastoms, Askin’s tumor of chest wall).

How well did you know this?
1
Not at all
2
3
4
5
Perfectly
18
Q

Definition of Hamartoma is (TW)
1.Abnormal disorganised tissue in abnormal position
2.Abnormal disorganised tissue in normal position
3.Normal disorganised tissue in normal position
4.Normal disorganised tissue in abnormal position

A

Normal disorganised tissue in normal position - T - cellular elements are mature and identical to those found in remainder of organ, but do not reproduce the normal
architecture of the surrounding tissue. Tumor-like malformation with tissues of particular part of body arranged haphazardly, usually with excess of one or more of its components.

How well did you know this?
1
Not at all
2
3
4
5
Perfectly
19
Q

Pick’s disease, uncommon findings (TW)
1.Asymmetrical atrophy
2.Predominant frontal lobes
3.Cortical atrophy
4.Involvement of post ⅔ superior temporal gyrus & parietal lobe

A

Involvement of post ⅔ superior temporal gyrus & parietal lobe - F - spared posterior
aspect of superior temporal gyrus and pre- and postcentral gyri. Unremarkable parietal and occipital lobes.

Picks disease / Frontotemporal dementia - nonspecific songioform degneration, with gliosis and neuronal loss, sometimes with Pick cells and bodies. 25-40% of FTD is familial. 10-30% of patients with positive family history have tau mutations (Tauopathy).

1.Asymmetrical atrophy - T - worse atrophy of dominant hemisphere
2.Predominant frontal lobes - T - anterior frontotemporal atrophy.
3.Cortical atrophy - T - thin cortex. Gliosis of corticl gray matter. Soft, retracted subcortical
white matter. Almost complete loss of large pyramidal neurons, diffuse spongiosis and
gliosis.

How well did you know this?
1
Not at all
2
3
4
5
Perfectly
20
Q

40 year old female with stroke, underlying cause least likely is (TW)
1.Atherosclerosis
2.Dissection
3.Coarctation of aorta
4.Giant cell arteritis
5.Mitral valve prolapse

A

.Giant cell arteritis - F - GCA is a chronic vasculitis of large and medium sized vessels.
Mean age at Dx is approx 72yo, and the disease essentially never occurs in individuals
younger than 50yo (UpToDate).

How well did you know this?
1
Not at all
2
3
4
5
Perfectly
21
Q

Atypical Scenario (TW)
1.Craniopharyngioma in a 42 year old
2.Anaplastic thyroid cancer in a 29 year old
3.Bowel cancer in a 32 year old
4.Cholangiocarcinoma in a young adult with emphysema

A

Anaplastic thyroid cancer in a 29 year old - F - older patients, mean age 65yo

1.Craniopharyngioma in a 42 year old - T - Bimodal age distribution (peak 5-15yo;
papillary craniopharyngioma >50y).
3.Bowel cancer in a 32 year old - T - peak incidence for CRC is 60-70yo. CRC in a young
person, preexisting UC or one of teh polyposis syndromes must be suspected.
4.Cholangiocarcinoma in a young adult with emphysema - T - a-1-antitrypsin deficency
predisposes to cholangiocarcinoma.

How well did you know this?
1
Not at all
2
3
4
5
Perfectly
22
Q

Pilocytic Astrocytoma, which is true (TW)
1.Associated with NF2
2.50% are solid
3.Prognosis is less than 70% 5year survival rate
4.Multipolar cells with microcysts, and bipolar cells with Rosenthal fibres

A

4.Multipolar cells with microcysts, and bipolar cells with Rosenthal fibres - T - classic
“biphasic” pattern of two astrocyte populations: compacted biplar cells with Rosenthal fibers (electron dense GFAP staining cytoplasmic inclusions); Loose-textured multipolar cells with microcysts, eosinophilic granular bodies.

1.Associated with NF2 - F - NF1. 15% of NF1 patients develop PAs (most commonly in
optic pathway). Upt o 1/3 of patients with optic pathway PAs have NF1.
2.50% are solid - F - 40% solid with necrotic center, heterogeneous enhancement. 10%
solid, homogeneous. 50% non enhancing cyst with enhancing mural nodule.
3.Prognosis is less than 70% 5year survival rate - F - median survival rates at 20y >70%

How well did you know this?
1
Not at all
2
3
4
5
Perfectly
23
Q

Retinoblastoma, which is the least likely ? (TW)
1.Very radio sensitive. Excellent prognosis even if it extends retro-orbitally
2.Carriers of RB gene have a 90% risk
3.Can get extraocular Retinoblastomas

A

Very radio sensitive. Excellent prognosis even if it extends retro-orbitally - F - enuleation
usually is indicated for large tumors with not visual potential, blind, painful eyes, and/or
tumors that extend into the optic nerve. External beam XRT was original globe-sparing
treatment for Rb. Risk of tumor recurrence following ext XRT 7%, occurring within
40months. Also risk of secondary cancers with XRT.

2.Carriers of RB gene have a 90% risk - T - If a mutant RB allele arises in the germ line, it
can be transmitted as a dominant trait, and carriers are at high risk (>90% risk for most
mutations) for retinoblastoma. Robbins.
3.Can get extraocular Retinoblastomas - T - trilateral RB = bilateral RB with
neuroectodermal pineal tumor. Quadrilateral RB = trilateral RB with 4th focus in
suprasellar cistern. Dahnert 6th.

How well did you know this?
1
Not at all
2
3
4
5
Perfectly
24
Q

Paragangliomas, which is false : (TW)
1.paraganglioma, chemodectoma, and carotid body tumors can be used interchangeably
2.carotid body tumors often adherent to vessels resulting in incomplete excision and
recurrence of 10%
3.glomus jugulare and carotid body paragangliomas are the most common head and neck
paragangliomas
4.paragangliomas have bipphasic or biphenotypic pattern and composed of chief cells and sustentacular cells

A

paraganglioma, chemodectoma, and carotid body tumors can be used interchangeably - F - multiple names: glomus tumor, chemodectoma, endothelioma, perithelioma,
sympathoblastoma, fibroangioma, sympathetic nevi. Paragangliomas are classified based on their location, innervation, and microscopic appearance. Would need to specify location for paraganglioma / chemodectoma to be able to use interchangeably with carotid body
tumor.

2.carotid body tumors often adherent to vessels resulting in incomplete excision and
recurrence of 10% - T - Shamblin classification - Type I are localized and easily removed;
type II adherent and partially surround carotid vessels; type III adherent and completely surround carotid vesels and extremely difficult to resect often requiring resection of ICA and vein graft interposition. Prevalence of local recurrence and local invasion - 40-50% of glomus jugulare tumors, 17% for vagal paragangliomas, and about 10% for carotid body tumors.
3.glomus jugulare and carotid body paragangliomas are the most common head and neck paragangliomas - T - conflicting reports regarding the prevalence of these 2 subtypes some saying one is more prevalent, some saying the other is.
4.paragangliomas have bipphasic or biphenotypic pattern and composed of chief cells and sustentacular cells

How well did you know this?
1
Not at all
2
3
4
5
Perfectly
25
Q

CMV encephalitis: which is false? (TW)
1.characteristic inclusions
2.ependymal & subependymal involvement
3.may cause haemorrhage
4.majority of newborns have systemic signs of disease, of which about half have CNS
involvement

A

majority of newborns have systemic signs of disease, of which about half have CNS
involvement - F - most infected newborns appear normal. 10% have systemic signs of
disease (hepatosplenomegaly, petechiae, chorioretinitis, jaundice, and IUGR). 55% with
systemic disease have CNS involvement (microcephaly, parenchymal Ca+, SNHL, seizures, hypotonia or hypertonia).

1.characteristic inclusions - T - CMV inclusion-bearing cells. Prominent cytomegatic cells
with intranuclear and intracytoplasmic inclusions can be readily identified. Hallmark is cytomegaly with viral nuclear and cytoplasmic inclusions.
2.ependymal & subependymal involvement - T - may affect any cell type by striking
tendency for virus to localise in teh epehdymal and subependymal regions of brain. Replicates in ependyma, germinal matrix, and capillary endothelia.
3.may cause haemorrhage - T - results in severe necrotising ventriculo-encephalitis with massive necrosis, haemorrhage, ventriculitis and choroid plexusitis

How well did you know this?
1
Not at all
2
3
4
5
Perfectly
26
Q

IV contrast, 4 -12 hrs afterwards, it results in skin necrosis. It’s a repeatable
response. What type of reaction is it? (TW)
1.Type 1
2.Type 2
3.Type 3
4.Type 4
5.Non-immune reaction

A

Type 3 - T - Immunocomplex disease. Induced by antigen-antibody complexes that
produce tissue damage as a result of their capacity to active the complement system. Can be generalized (immune complexes formed in circulation adn deposited in many organs etc), or localized to particular organs such as kidney (GN), joints (arthritis) or the small blood vessels of the skin if the complexes are formed an deposited locally (the local Arthus reaction). See below. Eg SLE, serum sickness.

Local immune complex disease (Arthus reaction) - type III hypersensitivity reaction. Localized area of tissue necrosis resulting from acute immune complex vasculitis, usually elicited in the skin.Reaction can be produced by intracutaneous injection of antigen in an immune patient having
circulating antibodies against the antigen. Unlike IgE mediated type I reactions, which appearimmediately, the Arthus lesion develops over a few hours and reaches a peak 4-10hrs after injection, when it can be seen as an area of visible oedema with severe hemorrhage followed
occasionally by ulceration.

Type 1 hypersensitivity - F- Anaphylactic type. Rapidly developing immunologic reaction
developing within minutes after combination of antigen with antibody bound to mast cells or
basophils in individuals previously sensitized to antigen
2.Type 2 - F - Cytotoxic type. Mediated by antibodies directed toward antigens present on
surface of cells or other tissue component (subtypes: complement-dependent reactions;
antibody-dependent cell mediated cytotoxicity; antibody mediated cellular dysfunction. Eg
ABO)
Type 4 - F - Cell mediated (delayed). Initiated by specifically sensitized T lymphocytes.

How well did you know this?
1
Not at all
2
3
4
5
Perfectly
27
Q

.Bowen Disease (TW)
1.associated with BCC in 5% of cases if left untreated
2.may have a penile cutaneous horn
3.results from chronic irritation and/or inflammation with development of scaly palques
often involving the meatus
4.carcinoma in situ occurring within follicle-bearing epithelium

A

carcinoma in situ occurring within follicle-bearing epithelium – T - Usually appears as a
solitary, dull-red plaque with areas of crusting and oozing.

How well did you know this?
1
Not at all
2
3
4
5
Perfectly
28
Q

HIV positive patient with CD4 count less than 100 has a mass on CT brain. What is the
likely diagnosis? (TW)
1.Lymphoma
2.Toxoplasmosis
3.Cryptosporidium
4.TB
5.PML

A

Toxoplasmosis - T- most common cause of focal lesion in HIV+.

1.Lymphoma - F - less likely, although increased risk with immunosuppression (35x).
3.Cryptosporidium - F - less likely cf toxoplasmosis
4.TB - F - less likely
5.PML - F - diffuse/patchy WM abnormalities.

How well did you know this?
1
Not at all
2
3
4
5
Perfectly
29
Q

Neuroblastoma, which is associated with a better prognosis: (GC)
1.Age > 1 year
2.Stage 4B
3.Abdominal primary
4.Decreased N-myc amplification
5.Metastases to bone only

A

4.Decreased N-myc amplification T - less copies of gene is better

1.Age > 1 year F - age < 1yr has a better prognosis
2.Stage 4B F - stage 4S has a near 100% survival
3.Abdominal primary F - thoracic primary has a better prognosis
5.Metastases to bone only F - mets to liver and skin is associated with better prognosis

Evans anatomic staging: (prognosis - % survival)
1: confined to organ of interest (90%)
2: extension beyond organ but not crossing midline (75%)
3: extension crossing midline (30%)
4: distal mets (10%)
4S: age < 1yr, mets confined to liver, skin and bone marrow.
[Paeds pocket rad]

How well did you know this?
1
Not at all
2
3
4
5
Perfectly
30
Q

Commonest sites for ependymoma, which is least likely: (GC)
1.Periventricular areas
2.Lateral and third ventricle in infants
3.Fourth ventricle in children
4.Spine in adults

A

2.Lateral and third ventricle in infants - anaplastic ependymoma (rare), usually in infants and children.

In children: infratentorial 70%; supratentorial 30%, of which frontal > parietal > temporoparietal
juxtaventricular region (uncommonly intraventricular), lateral ventricle, 3rd ventricle. [Dahnert]

  1. Periventricular areas T - supratentorial ependymoma is more commonly seated in the brain parenchyma, typically arising near the trigone of the lateral ventricle. Thought to
    arise from embryonic rests of ependymal tissue trapped in the developing cerebral
    hemispheres. Tend to be larger in size, more often cystic components cf. infratentorial. [RG
    2005, eMedicine]
    3.Fourth ventricle in children T - most commonly arises from floor of 4th ventricle.
    4.Spine in adults T - common site, better able to excise completely and generally better
    prognosis. Most common intramedullary spinal neoplasm in adults.
How well did you know this?
1
Not at all
2
3
4
5
Perfectly
31
Q

.Huntington’s disease, which is true: (GC)
1.Autosomal recessive
2.Affects the putamen and caudate
3.Presents in the second decade
4.Chorea due to striatal excitation

A

Affects the putamen and caudate T - degeneration of the striatum (C&P).

1.Autosomal recessive F - autosomal dominant disorder
3.Presents in the second decade F - age of onset most commonly in 4th and 5th decades,
related to the length of the CAG repeat.
4.Chorea due to striatal excitation F - loss of striatal inhibitory output, from the
degeneration of GABA-containing neurons. This leads to disregulation of the basal ganglia that normally modulate motor output.

How well did you know this?
1
Not at all
2
3
4
5
Perfectly
32
Q

In drug resistant Parkinsons and autonomic neuropathy an MRI would look for all the
following except? (TW) (MSA a)

1.Striatonigral degeneration
2.Decreased putaminal ADC values
3.Cerebellar atrophy
4.Cruciform pontine hyperintensity

A

2.Decreased putaminal ADC values - F - DWI appears to differentiate Parkinsons Disease
from progressive supranuclear palsy and the Parkinsons variant of multiple system atropy

MSA - striatonigral degeneration: clinically similar to idiopathic PD in presentation but is
relatively resistant to L-dopa treatment (due to pattern of neuronal degeneration - ie both
dopaminergic projections and its target neurons are absent, therefore L-dopa cannot bulster neurotransmission as occurs in parkinsons disease). Also have gross atrophy of caudat nucleus and putamen. Some patients also show evidence of pontocerebellar degeneration.
DWI - appears to differentiate PD from progressive supranuclear palsy and Parkinson variant of MSA. in MSA-Pv get increased putaminal ADC values. In PSP have increased ADC in putamen,
GP, and caudate nucleus.

1.Striatonigral degeneration - T - predominant degeneration of substantia nigra and
striatum, with putamen > caudate nucleus.
3.Cerebellar atrophy - T - hemispheres > vermis
4.Cruciform pontine hyperintensity - T - hot cross bun sign (but not pathognomonic of MSA)

How well did you know this?
1
Not at all
2
3
4
5
Perfectly
33
Q

Patient with possible ADEM. Which clinical setting least likely? (TW)
1.2 week history of viral illness
2.female
3.headache and confusion progressing to coma in 48 hrs
4.HIV positive
5.Age less than 40

A

4.HIV positive - F - this is probably rare, however ADEM has been assoc with HIVseroconversion illness. Probably getting you to Dx from PML.

1.2 week history of viral illness - T - monophasic (usually) demyelinating disease following
either a viral infection, or, rarely, a viral immunisation. Typically develops a week or two
after anticedent infection.
2.female - T - common finding. Male predominance reported in some series 1.3 : 1
3.headache and confusion progressing to coma in 48 hrs - T - usually fever, malasie,
myalgia as prodromal phase. Then headache, fever, drowsiness. Can have cranial nerve
palsies, seizures, hemiparesis, decreased consciousness (from lethargy to coma).
5.Age less than 40 - T - peak age 3-5yo, but can occur at anytime

How well did you know this?
1
Not at all
2
3
4
5
Perfectly
34
Q

A new Nuc Med agent can attach to the amyloid in neuritic plaques. Which is most
correct? (TW)
1.Should help exclude congophilic angiopathy in the elderly
2.Uptake would allow distinction between Alzheimers and age matched Parkinsons disease
3.Alzheimers patients should have greater uptake in the medial temporal lobe than age
matched control patients
4.Cerebellar uptake would suggest ataxic telangiectasia
5.Deep cerebral uptake would suggest multi-infarct dementia or MS

A

Alzheimers patients should have greater uptake in the medial temporal lobe than age
matched control patients - T - degenerative process starts in medial temporal lobe, spreads to parahippocampal gyrus, temporal and frontal lobes, and finally involves motor and visual cortex. Get parietal and temporal cortical atrophy with disproportionate
hippocampal volume loss. Predominates ijn medial temporal and pareital lobes.

How well did you know this?
1
Not at all
2
3
4
5
Perfectly
35
Q

34/40 fetus with large mass protruding posteriorly from sacrum. No evidence of Chiari malformation. Most likely is (TW)
1.Benign sacrococcygeal teratoma
2.Malignant sacrococcygeal teratoma
3.Congenital neuroblastoma
4.Imperforate cloacal membrane
5.Mature ganglioneuroma

A

Benign sacrococcygeal teratoma - T - sacrococcygeal region is the most frequent tumor
site of teratomas. SCTs occur most commonly in infants and may be Dx in utero, or at birth
with the majority in females. Frequence of malignancy in this location varies from less than
10% when younger than 2mo, to more than 50% when older than 4mo (Nelson’s pediatrics).
Prevalence 1 in 35000 to 40000 births (Donnelly).

How well did you know this?
1
Not at all
2
3
4
5
Perfectly
36
Q

16 year old with Friedrichs ataxia has a poor quality MRI. Most likely cause is (TW)
1.Intention tremor
2.Recurrent facial tic
3.Orthopnoea
4.Salaam spasms
5.Hemiballismus

A

Orthopnoea

Friedrich ataxia - AR degenerative disorder. Most common hereditary ataxia. Most cases caused by loss of function in frataxin gene (frataxin is a mitochondrial protein whose prescise function is
unknown).
Neuropathology - degeneration of posterior columns and the spinocerebellar tracts of the spinal cord and loss of the larger sensory cells of the dorsal root ganglia.
Major clinical manifestations of FQ - neurologic dysfunction, cardiomyopathy, and diabetes mellitus. Neuro - ataxia of limbs, absence of lower limb reflexes, and presence of pyramidial signs.
Early loss of position and vibration sense.
Cardiomyopathy - concentric LVH, asymmetric septal hypertrophy, and globally decreased LVF
patterns of disease.
Major causes of death are complications related to the cardiomyopathy or bulbar dysfunction,
leading to an inability to protect the airway.

How well did you know this?
1
Not at all
2
3
4
5
Perfectly
37
Q

.Hemorrhagic areas in cerebrum on CT. Least likely (TW & GC)
1.Recent pelvic fracture
2.Past rheumatic fever
3.Active mastoiditis
4.Recent neck manipulation
5.Recent placental abruption

A

Recent neck manipulation - F - vert dissection, but probably least likely option.
Population-based, case-control study found pts under 45yo, those with vertebrobasilar
dissection or occlusion were 5x more likely than controls to have visited a chiropractor in
the previous week. Actual incidence reports vary (1 per 400 000 manipulations, to 2 per
million).

How well did you know this?
1
Not at all
2
3
4
5
Perfectly
38
Q

65 year old, 3rd yearly follow up scan for sCJD. Which is most correct? (TW)
1.This is expected as CJD is slowly progressive
2.Incorrect diagnosis
3.Patient most likely has variant CJD
4.CJD has variable progression with 10-15% having a long term survival of > 10 years
5.Patient more likely to have the more indolent familial form

A

Incorrect DX

1.This is expected as CJD is slowly progressive - F - rapidly progressive mental
deterioration and myoclonus (sCJD). Mean duration of illness for sCJD 4-5months
(howevers urvival range varies depending on subtype of sCJD).
3.Patient most likely has variant CJD - F - mean duration of illness 14months.
4.CJD has variable progression with 10-15% having a long term survival of > 10 years - F -
both vCJD and sCJD are progressive and uniformly fatal. Occasional case reports of
survival to 40-50months, but rare.
5.Patient more likely to have the more indolent familial form - F - fCJD accounts for 10- 15% cases. Longer survival - mean duration 26 months. Age little younger than sCJD (~60yo).

How well did you know this?
1
Not at all
2
3
4
5
Perfectly
39
Q

Least likely sites for Toxo in the brain (TW)
1.GW junction
2.Pons
3.Cerebellum
4.Spinal cord
5.Putamen

A

Spinal cord - F - toxoplasmosis of the spinal cord is less frequent than in the brain (in
HIV), and has become even less common cause of myelopathy after the introduction of
HAART )

How well did you know this?
1
Not at all
2
3
4
5
Perfectly
40
Q

Most at risk for cerebral venous infarction false (TW)
1.diabetic in renal failure
2.post obstetric patient
3.young girl on progesterone only pill
4.young girl with SLE

A

3.young girl on progesterone only pill - F - oestrogen is major prothrombotic component.
Progesterone much lower risk. Most frequent reisk factor for cerebral venous sinus
thrombosis is the OCP (but this is presumably the oestrogen containing one).

How well did you know this?
1
Not at all
2
3
4
5
Perfectly
41
Q

Solitary 3cm cystic lesion in the brain containing an opaque gel-like substance (TW)
1.Hydatid / echinococcus
2.Cysticercosis
3.Amoebiasis
4.Strongyloides
5.Ascaris

A

1.Hydatid / echinococcus - T - large uni- or multilocular cyst +/- detached germinal
membraine. Cyst fluid is opalescent. Cyst is 3cm.

2.Cysticercosis - F - although most common parasitic infection world wide, cyst size is
variable and typically 1cm, with range of 4-20mm. Parenchymal cysts 1cm or less, however
subarachnoid systs may be larger, up to 9cm reported (and recemose). Cysts are ovoid and
white-to-opalescnet, rarely exceeding 1.5cm. Contain an invaginated scolex with hooklets
that are bathed in clear cyst fluid. Fluid can be opalescent in the vesicular – colloidal stage,
but question says cyst is 3cm.
3.Amoebiasis - F - cerebral amebiasis is a rare cause of brain abscess. Amebic encephalitits

How well did you know this?
1
Not at all
2
3
4
5
Perfectly
42
Q

NF1 – which is not/least associated (TW)
1.Wilms -
2.Rhabdomyosarcoma
3.CML
4.phaeochromocytoma
5.osteosarcoma

A

osteosarcoma - F - couple of case reports of NF1 and osteosarc, but not indicated that is
associated. Note however that NF1 is assoc with sarcomas.

wilms also a little shakey but more assoc.

How well did you know this?
1
Not at all
2
3
4
5
Perfectly
43
Q

Children with Ependymoma, which is false: (GC)
1.Prognosis poor
2.Less common than medulloblastoma
3.Does not metastasize as frequently as medulloblastoma
4.Myxopapillary variant in filum terminale
5.4 year survival 50%

A

4 year survival 50% F - current 5YS rate for patients with intracranial ependymomas is
approximately 50%, when rates from children and adults are combined. Stratification based
on age reveals 5YS rates of 76% in adults and only 14% in children.

1.Prognosis poor – T - Despite the survival advantage of gross total resection, lesions of the
posterior fossa are in close proximity to cranial nerves with a significant risk of long-term
neurologic dysfunction and disability. Hence, most tumors of the posterior fossa cannot be
fully resected and are likely to recur without postoperative radiation. Osborn: “cannot
remove completely”
2.Less common than medulloblastoma T - third most common paediatric brain tumour,
after medulloblastoma & JPA.
3.Does not metastasize as frequently as medulloblastoma T - subarachnoid dissemination
via CSF rare, observed in <10% of patients at diagnosis when ependymoblastomas are
excluded. The incidence is higher with infratentorial ependymomas than with supratentorial
tumors (9% vs 1.6%).
4.Myxopapillary variant in filum terminale T - considered a biologically and
morphologically distinct variant of ependymoma, occurring almost exclusively in the region
of the cauda equina and behaving in a more benign fashion than grade II ependymoma.

How well did you know this?
1
Not at all
2
3
4
5
Perfectly
44
Q

Huntington’s chorea, what does it show: (GC)
1.Various locations including basal ganglia
2.Caudate nucleus + putamen
3.Cerebellum
4.Locus ceruleus

A

Caudate nucleus + putamen T - striking atrophy (heart or box-shaped frontal horns), less

How well did you know this?
1
Not at all
2
3
4
5
Perfectly
45
Q

.Pilocytic Astrocytoma, most common location: (GC)
1.Cerebrum
2.CPA
3.Brainstem
4.Hypothalamus
5.Pituitary

A

Hypothalamus
Most lesions occur in or near the midline. Commonest locations: cerebellum, hypothalamus
(around 3rd ventricle), optic n./chiasm

How well did you know this?
1
Not at all
2
3
4
5
Perfectly
46
Q

Peripheral MCA aneurysm, most likely history: (GC)
1.SLE
2.Past history of irradiation for fibrous dysplasia
3.Hypertension
4.History of rheumatic fever and tooth extraction

A

History of rheumatic fever and tooth extraction T - Aneurysms that develop at distal sites
in the intracranial circulation are often caused by trauma or infection. Mycotic aneurysms
are most commonly due to infected arterial embolus, occurring in subacute bacterial endocarditis. In such cases, they are often multiple, being situated in small
corticomeningeal branches of the cerebral artery.

1.SLE F - Commonly reported CNS vascular lesions with SLE include infarcts and
transient ischemic attacks. Intracranial hemorrhages are present in approximately 10% of
patients with CNS symptoms. Although uncommon, arteritic and nonvasculitic aneurysms
occur in SLE. These can be saccular, fusiform, or a bizarre-looking mixture of both.
2.Past history of irradiation for fibrous dysplasia F - Radiation is contraindicated as it has
been found to increase the rate of secondary malignancy by 400 times. If in the remote
past… intracranial aneurysms may develop (and rupture) post irradiation.
3.Hypertension F - Charcot-Bouchard microaneurysms secondary to segmental
lipohyalinosis of the walls of long penetrating arteries.

How well did you know this?
1
Not at all
2
3
4
5
Perfectly
47
Q

Dying of presumed Alzheimer’s disease, which is NOT suggested in the diagnosis: (GC)
1.Parietal atrophy
2.Silver-staining dystrophic neurites surrounding a central amyloid core
3.Amyloid
4.Neurofibrillary tangles in occipital region
5.Intracellular changes in hippocampal pyramidal cells

A

Neurofibrillary tangles in occipital region - NFT’s are commonly found in the entorhinal
cortex, as well as in pyramidal cells of hippocampus, amygdala, basal forebrain, raphe
nuclei. Perhaps in advanced dementia there may be occipital cortex involvement (?).
Posterior cortical atrophy (Benson’s syndrome) and Balint’s syndrome may be regarded as
variants of AD, whereby deposition of neuritic plaques and NFT’s are specifically revealed
in the posterior cerebral areas (occipital cortex, visual cortex). [Acta Neuropathologica
1993]

1.Parietal atrophy - gross examination of brain shows a variable degree of cortical atrophy
with widening of the cerebral sulci, most pronounced in the frontal, temporal and parietal
lobes with compensatory ventricular enlargement. Parietal involvement helps differentiate
from Pick’s disease - asymmetric frontal & temporal lobe atrophy, with sparing of the
posterior 2/3 of superior temporal gyrus and only rare involvement of the parietal or
occipital lobes.
2.Silver-staining dystrophic neurites surrounding a central amyloid core - neuritic (senile)
plaques are a Dx feature.
3.Amyloid - almost invariable accompaniment of AD.
5.Intracellular changes in hippocampal pyramidal cells - these include Hirano bodies,
NFT’s, granulovacuolar degeneration.

How well did you know this?
1
Not at all
2
3
4
5
Perfectly
48
Q

Subependymoma, which is true (TW)
1.Usually slow growing and asymptomatic
2.Associated with TS
3.5-10 yo
4.Commonly in lateral ventricle

A

Commonly in lateral ventricle - T - inferior fourth ventricle, frontal horn of lateral
ventricle most common sites.

1.Usually slow growing and asymptomatic - F - slow growing, however, although can be
asymptomatic, 50% become symptomatic. WHO grade I.
2.Associated with TS - F - TS assoc with cortical / subcortical tubers, subependymal
hamartomas, subependymal GCA. Zits, fits, nitwits (facial angiofibroma, epileptic seizures,
mental retardation).
3.5-10 yo - F - middle-aged / elderly adult (typically 40-50yo). Rare in children (unlike
ependymoma).

How well did you know this?
1
Not at all
2
3
4
5
Perfectly
49
Q

TB Meningitis, which is most correct: (GC) see August 2007
1.Endarteritis causes ischaemia.
2.Small multiple miliary nodules in brain
3.Cannot be seen macroscopically
4.Can be asymptomatic

A

1.Endarteritis causes ischaemia. T - ischaemic infarcts of BG + internal capsule in 20-40%,
due to vascular compression and/or occlusion of small perforating vessels in basal cisterns.

How well did you know this?
1
Not at all
2
3
4
5
Perfectly
50
Q

.Juvenile pilocystic astrocytoma, which is false: (GC) see August 2007
1.Even without surgery rarely fatal
2.Well circumscribed, easy to cut out
3.Commonly cortical
4.More benign behaviour when it occurs in patient with NF1
5.Abundant Rosenthal fibres

A

3.Commonly cortical F - most lesions occur in or near the midline, usually arising from the
cerebellum, optic n/chiasm, hypothalamic-thalamic region.

How well did you know this?
1
Not at all
2
3
4
5
Perfectly
51
Q

Leighs, which is incorrect… (TW)
1.Increased T2 signal in peri-aqueductal gray matter
2.Enhancement is uncommon
3.Lysosomal storage disorder
4.May be x-linked inheritance
5.Most present by 2y age

A

Lysosomal storage disorder – F – mitochondrial disorder characterised by
neurodegeneration. Brain and striated muscle highly dependent on oxidative
phosphorylation which are moste severely affected in mitochondrial disorders.

1.Increased T2 signal in peri-aqueductal gray matter – T – best diagnostic clue = bilateral,
symmetric increased T2/FLAIR in putamina and peri-aqueductal gray matter
2.Enhancement is uncommon - T
4.May be x-linked inheritance – Autosomal recessive, X-linked, and maternal inheritance of
mutated proteins involved in mitochondrial energy production underlie Leigh syndrome.
5.Most present by 2y age – T – majority by 2yo

How well did you know this?
1
Not at all
2
3
4
5
Perfectly
52
Q

Which is not a feature of Alzheimer’s: (GC) see April 2007
1.Hirano bodies
2.Lewy bodies
3.Senile plaques
4.Neurofibrillary tangles
5.Amyloid

A

2.Lewy bodies F - eosinophilic intracytoplasmic inclusions found in some neurones in parkinsons

How well did you know this?
1
Not at all
2
3
4
5
Perfectly
53
Q

Regarding vestibular schwannomas: (TW)
1.Bilateral tumours are associated with NF1
2.Haemorrhage is uncommon
3.About 70% are cystic
4.Meningeal reaction is typical
5.A melanotic variant is a recognized entity –

A

Haemorrhage is uncommon – T – rare to get haemorrhagic foci (0.5%).

1.Bilateral tumours are associated with NF1 – F – NF2 = MISME, multiple inherited
schwannomas, meningiomas, and Ependymomas.
3.About 70% are cystic – F – 15% have intramural cysts (Harnsberger)
4.Meningeal reaction is typical – F – this is a differentiating factor to Dx from meningioma.
5.A melanotic variant is a recognized entity – F - CME says F. However there are a handful
of cases reported (so very, very rare). Melanotic schwannoma is a rare melanin-producing
nerve sheat tumor. MSs are of neural crest origin, prob from neoplastic proliferation of a
common precursor cell for both Schwann cells and melanocytes. Most involve cranial or
spinal nerve roots (esp spinal Cx and thoracic nerves). 10% involve symp chain. There have
been reports of MSs of vestibular nerves. MSs may be component of Carney complex.

How well did you know this?
1
Not at all
2
3
4
5
Perfectly
54
Q

.Mesial temporal sclerosis, which is false (TW)
1.contrast enhancement of the para hippocampal gyrus
2.atrophy hippoc gyrus
3.may see fornix and/or mamillary body atrophy
4.dilatation of temporal horn may be evident

A

1.contrast enhancement of the para hippocampal gyrus - F – no enhancement.

2.atrophy hippoc gyrus – T – decreased size of hippocampus
3.may see fornix and/or mamillary body atrophy – T – location: mesial temporal lobe –
hippocampus > amygdale > fornix > mamillary bodies. Bilateral in 20% cases.
4.dilatation of temporal horn may be evident - T

How well did you know this?
1
Not at all
2
3
4
5
Perfectly
55
Q

The following statements regarding agenesis of the corpus callosum, false?: (TW)
1.Ventriculomegaly predominantly affects the occipital horns
2.Is associated with Down’s syndrome
3.The lateral ventricles are widely placed
4.There is an association with heterotopic gray matter
5.It is associated with colpocephaly

A

Is associated with Down’s syndrome – F – Assoc with abnormal karyotype (T13, 15, 18 –
not typically assoc with T21, however there are some case reports). Multiple other assoc:
Dandy-Walker cyst, interhemispheric arachnoid cyst, hydrocephalus, intracerebral lipoma
of CC, AC II malformation, porencephaly, Holoprosencephaly, polymicrogyria.

1.Ventriculomegaly predominantly affects the occipital horns – T – occipital horns often
dilated (colpocephaly). Pointed frontal horns. Can get dilation of the posterior temporal
horns in the absence of splenium.
3.The lateral ventricles are widely placed – T- “bat-wing” appearance of lateral ventricles
(= wide separation of lateral ventricles with straight parallel parasagittal orientation with
absent callosal body)
4.There is an association with heterotopic gray matter – T – polymicrogyria, gray-matter
heterotopia
5.It is associated with colpocephaly - T

How well did you know this?
1
Not at all
2
3
4
5
Perfectly
56
Q

Sellar lesions, which is true (TW)
1.normal sella favours meningioma rather than macroadenoma
2.squamous variant of craniopharyngioma is more likely cystic
3.intrasellar arachnoid cyst will displace stalk anteriorly
4.17mm pituitary is normal for pregnant female
5.macroadenoma more likely to compress ICA than meningioma

A

normal sella favours meningioma rather than macroadenoma - T - as pituitary MAs grow
they first expand sella and then grow upwards. Aggresive adenomas extend inveriorly,
invade sphenoid, and may destroy upper clivus. As they are soft - often are indented by
diaphragma sellae giving them a ‘snowman’ configuration which can help Dx from
meningoma.

2.squamous variant of craniopharyngioma is more likely cystic - F - squamous papillary
type (more common in adults) rarely calcifies, oftsn solid, isodense. Both
adamantinomatous and squamous papillary types enhance.
3.intrasellar arachnoid cyst will displace stalk anteriorly - F - 10% arachnoid cysts are
suprasellar & would push stalk back.
4.17mm pituitary is normal for pregnant female - F - normal pituitary measurements:
children 6mm, males and post-menopausal females 8mm, young menstruating females
10mm, pregnant/lactating females 12mm.
5.macroadenoma more likely to compress ICA than meningioma

How well did you know this?
1
Not at all
2
3
4
5
Perfectly
57
Q

Sensorineural hearing loss, which is true (TW)
1.destruction of ossicular chain from otitis media
2.cochlear SNHL is more common than retrocochlear SNHL
3.sudden loss after minor trauma is typical for vestibular aqueduct synd

A

sudden loss after minor trauma is typical for vestibular aqueduct synd - T

1.destruction of ossicular chain from otitis media - F - conductive
2.cochlear SNHL is more common than retrocochlear SNHL - F - other way around.
retrocochlear SNHL (abnormalities of neurons of the spiral ganglion) more common than
cochlear SNHL (damage to cochlea / organ or Corti). Dahnert 6th.

How well did you know this?
1
Not at all
2
3
4
5
Perfectly
58
Q

MS (TW)
1.juxta cortical favours MS vs age
2.enhancement lasts 3 months
3.usually thoracic spinal lesion
4.perpendicular to long axis if cord

A

enhancement lasts 3 months - T - Dahnert says Gd-DTPA enhancement of lesions on
T1WI up to 8weeks following acute demyelination. Osborn says >90% of enhancing lesions
disappear within 6 months.

1.juxta cortical favours MS vs age - F - calloseptal favors MS, plus infratentorial.
2.enhancement lasts 3 months - T - Dahnert says Gd-DTPA enhancement of lesions on
T1WI up to 8weeks following acute demyelination. Osborn says >90% of enhancing lesions
disappear within 6 months.
3.usually thoracic spinal lesion - F - predilection for cervical cord
4.perpendicular to long axis - F - parallell to long axis of spinal cord / perpendicular to the
axis of CC, ventricles.

How well did you know this?
1
Not at all
2
3
4
5
Perfectly
59
Q

.CC dysgenesis, false (TW)
1.genu always present in partial
2.cingulate gyrus normal
3.foramen monro enlarged
4.assoc with AC II

A

2.cingulate gyrus normal - F - dysgenesis of cingulate gyrus / persistent eversion of
cingulate gyrus medial parietal sulci affected - T - radial array pattern of medial cerebral
sulci, gyri “point to” 3rd ventricle

1.genu always present in partial - T - depending on time of arrested growth (anteroposterior
development of genu, body, splenium, however, rostrum forming last). Can have: genu only,
genu + part of body, genu + entire body, genu + body + splenium (without rostrum).
3.foramen monro enlarged - T - enlarged / elongated foramina of Munro
4.assoc with AC II - T - 7%. Also assoc with DW cyst, interhemispheric arachnoid cyst,
hydrocephalus, lipoma of CC, porencephaly, holoprosencephaly, polymicrogyria.

How well did you know this?
1
Not at all
2
3
4
5
Perfectly
60
Q

Congenital CNS infections, which is least likely:
Chicken pox
Rubella
CMV
Toxo
Herpes

A

Herpes

How well did you know this?
1
Not at all
2
3
4
5
Perfectly
61
Q

Macroscopic features of de Quervains Thyroiditis (AKA subacute). (TW)
1.Diffuse smooth symmetrical goiter
2.Multiple nodules
3.Bilateral or unilateral enlarged lobes

A

Bilateral or unilateral enlarged lobes - T - gland may be unilaterally or bilaterally
enlarged and firm, with an intact capsule. Both thyroid lobes are involved from the
beginning in most patients.

1.Diffuse smooth symmetrical goiter - F - only mild-moderate enlargement (Dahnert,
UptToDate), patchy with occasionally distinctly focal inlvolvement. On cut section -
involved areas are firm and yellow-white and stand out from the more rubbery, normal
brown thyroid substance.
2.Multiple nodules - F - early scattered follicles may be entirely disrupted and replaced by PMN forming microabscesses. Later the characteristic changes: aggregates of cells around damaged follicles.

How well did you know this?
1
Not at all
2
3
4
5
Perfectly
62
Q

.55 yr old man with oncocytes from biopsy of a solitary parotid cyst? (TW)
1.Benign pleomorphic adenoma
2.Warthins tumour
3.HIV.
4.Type 1 Branchial cleft cyst
5.Mucoepidermoid

A

Warthins tumour - T - benign neoplasm. Distinctive double layer of lining cells with
surface palisade of columnar cells having abundant, finely granular, eosinophilic
cytoplasm, imparting an oncocytic appearance, resting on a layer of cuboidal to polygonal cells.

1.Benign pleomorphic adenoma - F - heterogeneity. Epithelial elements resembling ductal cells or myoepithelial cells disposed in duct formations, acini, irregular tubules, strands, or sheets of cells.
3.HIV.
4.Type 1 Branchial cleft cyst - F - fibrous walls usually lined by stratified squamous or
pseudostratified columnar epithelium underlaid by an intense lymphcytic infiltrate or, more
often, well-developed lymphoid tissue with reactive follicles.
5.Mucoepidermoid - F - mucus secreting cells, often forming glandular spaces. Higher
grade tumors are composed of squamous cells with only a scatterin gof mucus-secreting
cells.

Oncocytes are epithelial cells stuffed with mitochondria that impart the granular
appearance to the cytoplasm.

How well did you know this?
1
Not at all
2
3
4
5
Perfectly
63
Q

Least likely to cause Hyperthyroidism? (TW)
1.Graves disease
2.Toxic adenoma
3.Hashimotos thyroiditis
4.Riedels Thyroiditis
5.Subacute thyroiditis

A

Riedels Thyroiditis - F - rare, chronic inflammatory disease of thyroid gland
characterized by a dense fibrosis that replaces normal thyroid capsule. Morbidity is most frequently related to local compressive symptoms. Most patients are euthyroid, and
hypothyroidism can be seen in 30% cases.

1.Graves disease - T - thyroid stimulating antibodies. Elevated T3 and T4.
2.Toxic adenoma - T - AKA Plummer disease / toxic autonomous nodule. Hyperthyroidism
caused by one / two hyperfunctioning nodules independent of normal pituitary-thyroid control.
3.Hashimotos thyroiditis - T - thyrotoxicosis in early stage 4%. Hypothyroid at presentation
20%. Chronic lymphocytic thyroidits. Autoimmune process with marked familial
predisposition.
5.Subacute thyroiditis - T - AKA subacute thyroiditis. Hyperthyrodisim 50% secondary to severe destruction. Short-lived hypothyroidism (25%) due to hormone depletion of gland.
Probably viral aetiology. Lymphocytic infiltration, granulomatous and foreign body giant cells

How well did you know this?
1
Not at all
2
3
4
5
Perfectly
64
Q

Which of the following is least correct? (TW)
1.Chordomas arise from physaliphorous cells
2.Paragangliomas are parasympathetic tumours
3.Cholestrol granulomas need an aerated pterous apex for formation
4.Cholestatomas arise from epithelial rests

A

1.Chordomas arise from physaliphorous cells - F - arise from embryonic remnants of the
primitive notochord. Microscopically, cordomas are composed of characteristic mucoid, fluid-containing, translucent cells of variable size with a large, intracytoplasmic vacuole, and they are rich in mucin and glycogen (physaliphorous cells)

2.Paragangliomas are parasympathetic tumours - T - rare neuroendocrine tumors arising from paraganglionic tissue found between base of skull and floor of pelvis. Adrenal
paraganglioma = phaeochromocytoma, extraadrenal paraganglioma sympathetic /
parasympath
3.Cholestrol granulomas need an aerated pterous apex for formation - T - results from
obsturction of the normal aeration of petrous air cells.
4.Cholestatomas arise from epithelial rests - T - derived from aberrant embryonic
ectodermal rests in temporal bone (primary, AKA epidermoid cyst). Ingrowth of squamous
epithelium throught tympanic membrane secondary to repeated inflammation / marginal perforation of ear drum.

How well did you know this?
1
Not at all
2
3
4
5
Perfectly
65
Q

Which of the following are true in regards to choristomas and hamartomas? (TW)
1.A hamartoma is an ectopic rest of tissue
2.A choristoma is abnormal tissue in a normal location
3.An adenoma is a typical hamartoma -
4.Adrenal tissue under the renal capsule is an example of a choristoma
5.Hamartomas predispose to an increased risk of malignant transformation

A

Adrenal tissue under the renal capsule is an example of a choristoma- T - normal cells or
tissues in an abnormal location.

1.A hamartoma is an ectopic rest of tissue- F - Tumor-like malformation with tissues of a
particular part of the body arranged haphazardly, usually with excess of one or more of its components. Excessive but focal overgrowth of cells and tissues native to the organ in which it occurs (cf teratoma - tissues NOT specific to the part)
2.A choristoma is abnormal tissue in a normal location - F - AKA heterotopia. Normal cells
or tissues that are present in abnormal locations. eg rest of pancreatic tissue found in the wall of the somtach or small intestine, or small mass of adrenal cells found in the kidney, lungs, ovaries, or elsewhere. The heterotopic rests ore usually of little significance, but can be confused clinically with neoplasms.
4.Adrenal tissue under the renal capsule is an example of a choristoma- T - normal cells or
tissues in an abnormal location.
5.Hamartomas predispose to an increased risk of malignant transformation - F - not
neoplastic. No tendency to excessive growth, being co-ordinated with surrounding tissues
and stops after adolescence.

How well did you know this?
1
Not at all
2
3
4
5
Perfectly
66
Q

Which is not a mixed neural/glial tumour? (TW)
1.Ganglioglioma
2.DNET
3.Lhermitte Duclos
4.Pleomorphic xanthoastrocytoma
5.Neurocytoma

A

Pleomorphic xanthoastrocytoma - F - rare distinct astrocytoma subtype tumor of children and young adults. Astrocytic tumor.

Neuroglial tumors - nerve cells and glial cells of the brain come from a primitive neuroepithelial precur cell and germinal matrix. Pure neurons = ganglion cells (gangliocytoma, neurocytoma).
Tumors of oligodendrocytes and astrocytes may be combined with ganglion cells and calledgangliogliomas. If abnormalities which are considered to incorporate all these tumor types - can be called neuroepithelial tumors (DNET).

1.Ganglioglioma - T - benign tumor composed ofneoplastic astrocytes and ganglion cells.
2.DNET - T - mixed neuronal-glial neoplasm of infancey with a distinct desmoplastic
component.
3.Lhermitte Duclos - T - dysplastic gangliocytoma of cerebellum - very rare, tumor-like neuronal lesion of the cerebellum, with distincet dysplastic features.
4.Pleomorphic xanthoastrocytoma - F - rare distinct astrocytoma subtype tumor of children and young adults. Astrocytic tumor.
5.Neurocytoma - T - Interventricular tumor composed of uniform round cells with
immunihistochem an ultrastructural features of neuronal differentiation. Arises from septum pellucidum.

How well did you know this?
1
Not at all
2
3
4
5
Perfectly
67
Q

Which of the following is least correct regarding fat embolism? (GC)
1.Symptoms of irritability and confusion
2.Petechial rash
3.Hypovolaemic shock
4.Passive flow of adipose tissue through the vascular system

A

Hypovolaemic shock - F - tachycardic, SOB and hypoxic (but not hypotensive) in fat
embolism syndrome, which occurs 1-3 days after injury. Hypovolaemic shock may have
been an issue at the time of injury.

1.Symptoms of irritability and confusion - T - systemic hypoxaemia and dyspnoea,
mentation changes (headaches, confusion), petechiae in 50% from coagulopathy (release of tissue thromboplastin), fever.
2.Petechial rash - T - see ans (1).
4.Passive flow of adipose tissue through the vascular system - T - fat enters circulation after fractures of long bones (ie. rupture of marrow vascular sinusoids), or after soft tissue injury (rupture of venules). Pathogenesis involves obstruction of pulmonary vessels by fat globules,
followed by chemical pneumonitis from unsaturated plasma fatty acids, producing
haemorrhage/oedema.

How well did you know this?
1
Not at all
2
3
4
5
Perfectly
68
Q

VHL is associated with all of the following except… (TW)
1.Clear cell RCC
2.Papillary cell RCC
3.Retinal angiomas
4.Pheochromocytomas
5.3p deletions

A

2.Papillary cell RCC - F - RCCs of predominant papillary, chromophobe, or oncocytic
histology are not associated with vHL disease (UTD). pRCC is the 2nd most common
subtype, and most common multifocal or bilateral renal tumor. Most pRCCs are sporadic.

1.Clear cell RCC - T - all vHL-associated RCCs are clear cell tumors, although these clear
cell tumors may contain minor papillary components (usually <25% of the lesion).
3.Retinal angiomas - T - see below
4.Pheochromocytomas - T - vHL phenotype: pheochromocytoma (frequently bilateral),
paraganglioma (rarely), retinal angioma, cerebellar hemangioblastoma, epididymal
cystadenoma, renal and pancreatic cysts, pancreatic neuroendocrine tumors, and renal cell carcinoma.
5.3p deletions - T - vHL gene Ch 3p25. Specific germline mutations or deletions of the vHL
gene can influence the clinical manifestations of vHL disease

How well did you know this?
1
Not at all
2
3
4
5
Perfectly
69
Q

70 year old man with a 5 year history of a slowly enlarging, painless parotid mass which has markedly increased in size over the last 3 mths. The most likely cause is: (–)
5.Pleomorphic adenoma with malignant transformation to adenocarcinoma
6.Infected first branchial cleft cyst
7.Warthin’s Tumour with cystic change
8.Haemorrhage into a pleomorphic adenoma
9.Erosion and fistula

A

Pleomorphic adenoma with malignant transformation to adenocarcinoma

A carcinoma infrequently arises in a pleomorphic adenoma, referred to variously as a carcinoma ex pleomorphic adenoma or a malignant mixed tumor.
The incidence of malignant transformation increases with the duration of the tumor, being about
2% for tumors present less than 5 years
10% for those of more than 15 years’ duration
The cancer usually takes the form of an adenocarcinoma or undifferentiated carcinoma, and often it virtually completely overgrows the last vestiges of the preexisting pleomorphic adenoma; but to substantiate the diagnosis of carcinoma ex pleomorphic adenoma, recognizable traces of the latter must be found.
Regrettably, these cancers, when they appear, are among the most aggressive of all salivary gland malignant neoplasms, accounting for a 30% to 50% mortality in 5 years

How well did you know this?
1
Not at all
2
3
4
5
Perfectly
70
Q

Nasal polyps which is TRUE? Big Rob p762
1.By definition < 2cm
2.Mostly associated with atopy
3.Ulcerate in chronic disease and not secondary to bacterial infection
4.30% have history of atopy
5.Histology is orderly mucosa with stromal

A

Ulcerate in chronic disease and not secondary to bacterial infection

1-4cm
most not atopy
not orderly; oedematous, loose stroma, hyperplastic or cystic mucus flands, infiltrated inflammatory

How well did you know this?
1
Not at all
2
3
4
5
Perfectly
71
Q

Cholestatoma which is TRUE? Big Rob p767
1.Contains cholestrol crystals
2.Stratified Squamous epithelium and keratinous material
3.Causes destructive lesion in inner ear

A

Stratified Squamous epithelium and keratinous material

Cholesteatomas associated with chronic otitis media are not neoplasms, nor do they always contain cholesterol.
Rather, they are cystic lesions 1 to 4 cm in diameter, lined by keratinizing squamous epithelium or metaplastic mucus-secreting epithelium, and filled with amorphous debris (derived largely from desquamated squames)
Sometimes they contain spicules of cholesterol

How well did you know this?
1
Not at all
2
3
4
5
Perfectly
72
Q

Tracheo-oesophageal fistula, MOST COMMON ? Rob p549 (TW)
1.Proximal fistula, distal blind pouch
2.“H” type
3.Distal fistula, proximal pouch
4.Misnomer because no atresia
5.Misnomer because no fistula

A

Distal fistula, proximal pouch - T - oesophageal atresia and tracheo oesophageal fistula
not in communication with proximal oesophagus (82%)

How well did you know this?
1
Not at all
2
3
4
5
Perfectly
73
Q

Classic appearance of Malloy Weiss tear is ? Rob p550 (–)
1.Linear at gastro-oesphageal junction
2.Linear with hiatus hernia
3.Undermined proximal mucosa
4.Circular at GOJ

A

Linear at gastro-oesphageal junction (linear lacerations in axis of oesophagus lumen, at or
below the G-E junction)

boerhaaves = rupture

How well did you know this?
1
Not at all
2
3
4
5
Perfectly
74
Q

What are risk factors for malignancy in a stomach ulcer ?
1.pernicious anaemia
2.Crohn’s disease
3.coeliac disease
4.H. Pylori

A

pernicious anaemia (Pernicious anaemia + gastritis- 3x risk only)
H. Pylori (H.P associated chronic gastritis is assoc. with 5x increased risk

Long-term risk of gastric cancer in Autoimmune gastritis = 2 – 4%
Even in heavily colonized stomachs, the organisms are absent from areas with intestinal metaplasia
As such, they probably account for the increased incidence of gastric cancer in atrophic forms of
gastritis, particularly in association with pernicious anemia
2004 QUESTION SAID OCCURS IN AREAS OF ATROPHY

How well did you know this?
1
Not at all
2
3
4
5
Perfectly
75
Q

Malignant gastric ulcer. What would be the most concerning for malignancy? Rob p561
(TW)
1.> 4cm diameter
2.single ulcer of greater curvature
3.heaped edges

A

heaped edges - heaping up of the margins is rare in the benign ulcer but is characteristic
of the malignant lesion.

1.> 4cm diameter - size does not diffentiate a benign from malignant ulcer, usually less than 2cm)
2.single ulcer of greater curvature - location of gastric carcinoma within stomach is: pylorus and antrum 50-60%; cardia 25%; remainder in body an fundus. The lesser curvature is involved in about 40% and the greater curvature in 12%. Thus, a favored location is the lesser curvature of the antropyloric region. Although less frequent, an ulcerative lesion on the greater curvature is more likely to be malignant.

How well did you know this?
1
Not at all
2
3
4
5
Perfectly
76
Q

Stomach cancer, which is NOT TRUE? Rob p561 (TW)
1.Benign ulcers on lesser curvature at antrum
2.Alcohol is not a risk factor
3.Most common is adenocarcinoma/ then lymphoma/ then carcinoid/then GIST
4.Early gastric cancer is confined to the mucosa/submucosa with negative perigastric nodes
5.Morphological types – excavated, exophytic or flat

A

Early gastric cancer is confined to the mucosa/submucosa with negative perigastric nodes - F - regardless (ie not influenced by) of presence of perigastric nodes (Robbins).

Ulcerated carcinomas usually have nodular raised margins with “dirty” necrotic bases, and lack surrounding radiating folds
Early gastric carcinoma: lesion confined to mucosa and submucosa, regardless of presence or absence of perigastric lymph node metastases.
Advanced gastric carcinoma: neoplasm that has extendd below the submucos into the muscular wall.

1.Benign ulcers on lesser curvature at antrum - T - gastric ulcers are predominantly located
along the lesser curvature, in or around the border zone between the corpus and the antral mucosa.
2.Alcohol is not a risk factor - T
3.Most common is adenocarcinoma/ then lymphoma/ then carcinoid/then GIST - T -
Adeono 90-95%, Lymphoma 4%, Carcinoids 3%, malignant stromal cell tumours (2%).
5.Morphological types – excavated, exophytic or flat (true)

How well did you know this?
1
Not at all
2
3
4
5
Perfectly
77
Q

Which of the following statements concerning GIT diseases IS INCORRECT: (JS)
1.Most gut lymphomas are T-cell type
2.Villous adenomas are most common in the rectum
3.In Hirshsprung’s Disease, the rectum is always involved
4.Angiodysplasia is most common in the proximal colon
5.Necrotising enterocolitis is most common in neonates

A

Most gut lymphomas are T-cell type - F - most are B cell lymphomas (90%)

How well did you know this?
1
Not at all
2
3
4
5
Perfectly
78
Q

Coeliac diseasease What is the least likely? (JS)
1.anti-gliadin antibodies
2.treatment prevents development of lymphoma

A

2.treatment prevents development of lymphoma - F - there is a long term risk of malignant disease at 2 times the usual rate - intestinal lymphomas (including disproportionately high number of T-cell lymphomas) and GIT and breast cancers.

1.anti-gliadin antibodies - T - due to a sensitivity to gluten (gliadin)

How well did you know this?
1
Not at all
2
3
4
5
Perfectly
79
Q

Coeliac disease associated with? Rob p571 (JS)
1.2x risk of malignancy
2.Bowel wall thinning

A

1.2x risk of malignancy - T - there is a long term risk of malignant disease at 2 times the
usual rate - intestinal lymphomas (including disproportionately high number of T-cell
lymphomas) and GIT and breast cancers.

2.Bowel wall thinning - F - crypts are elongated, hypoplastic and tortuous but the overall mucosal thickness remains the same

How well did you know this?
1
Not at all
2
3
4
5
Perfectly
80
Q

.Coeliac disease, T/F (JS)
1.definitive diagnosis is shown by showing histological total villous atrophy
2.10-20% have anti-gliadin Ab’s without having the disease

A

1.definitive diagnosis is shown by showing histological total villous atrophy - T - diagnosis
requires documentation of malabsorption, demonstration of intestinal lesion by small bowel biopsy and improvement in symptoms with gluten free diet.

2.10-20% have anti-gliadin Ab’s without having the disease - T - 90% of patients with
coeliac disease will have a positive IgA AGA before treatment but 10-20% of people will
have a false positive IgA AGA and a normal small bowel biopsy. IgG EMA (Endomysial
antibodies) has a much lower false positive rate (1%) and is found in 95-100% of patients
with the disease.

How well did you know this?
1
Not at all
2
3
4
5
Perfectly
81
Q

Which of the following statements concerning gastrointestinal diseases is most correct?
(JS)
1.The colon is the most common site for GIT lymphoma -
2.Pseudomembranous colitis is due to toxins of clostridium difficile
3.Inflammatory pseudo-polyps are a characteristic feature of Crohn’s colitis
4.Apthous ulcers are a characteristic feature of ulcerative colitis
5.Juvenile polyps typically occur in the ascending colon.

A

3.Inflammatory pseudo-polyps are a characteristic feature of Crohn’s colitis - F - associated
with UC but can occur in Crohns
4.Apthous ulcers are a characteristic feature

1.The colon is the most common site for GIT lymphoma - F - Stomach (55-60%), small
intestine (25-30%), proximal colon (10-15%) and distal colon (10%) with appendix and
oesophagus rarely involved.
2.Pseudomembranous colitis is due to toxins of clostridium difficile - T - Caused by
exotoxins A and B of C. difficle after antibiotic therapy of ulcerative colitis - F - associated with Crohns
5.Juvenile polyps typically occur in the ascending colon. - F - hamartomatous
malformations of mucosa, typically in the rectum

How well did you know this?
1
Not at all
2
3
4
5
Perfectly
82
Q

In which one of the following are granulomas NOT a characteristic feature? (JS)
1.Crohn’s disease
2.Sarcoidosis
3.Foreign body response
4.Ulcerative colitis
5.Tuberculosis

A

Ulcerative colitis

How well did you know this?
1
Not at all
2
3
4
5
Perfectly
83
Q

Which of the following IS NOT a recognised feature of diverticular disease of the colon:
(JS)
1.Hypertrophy of the circular muscle layer of the bowel
2.Dissection into the appendices epiploicae
3.Crypt abscesses
4.Fistulae
5.Haemorrhage

A

3.Crypt abscesses - F - Typical lesion of UC - collections of neutrophils within crypt

How well did you know this?
1
Not at all
2
3
4
5
Perfectly
84
Q

Concerning carcinoid tumours of the gastro-intestinal tract, which of the following
statements IS LEAST correct: (JS)
1.The appendix is the most common site
2.They have a characteristic desmoplastic reaction
3.They are benign tumours -
4.A characteristic feature is a solid, yellow-tan appearance on transactions
5.They are a more common small bowel neoplasm than primary adenocarcinoma

A

3.They are benign tumours - F - all are potentially malignant

1.The appendix is the most common site - T - Appendix most common, followed by terminal ileum, stomach, rectum & colon
2.They have a characteristic desmoplastic reaction - T
4.A characteristic feature is a solid, yellow-tan appearance on transactions - T - Solid,
yellow-tan appearance on cut section
5.They are a more common small bowel neoplasm than primary adenocarcinomas - T - most common lesion of small bowel – about the same incidence (both about half)

How well did you know this?
1
Not at all
2
3
4
5
Perfectly
85
Q

Amebiasis which is TRUE ?
1.Organism is Ngareri Fowleri
2.Invasion through mucosa and into peyers patches
3.10% of infected people have dysentery
4.Cysts release trophozoites secondary to pancreatic enzymes.
5.Release exotoxin and denude epithelium

A

3.10% of infected people have dysentery - T

1.Organism is Ngareri Fowleri - F - Entamoeba histolytica
2.Invasion through mucosa and into peyers patches - F - invade the crypts of the colonic
glands and burrow through the tunica propria and are halted by the muscularis mucosae.
4.Cysts release trophozoites secondary to pancreatic enzymes. - F - release trophozoites
under anaerobic conditions
5.Release exotoxin and denude epithelium - F - This is referring to C difficle in
pseudomembranous colitis. Amoebae produce proteins involved in tissue invasion including proteinases, lectin and amebapore, resulting in ulcers and sloughing of the mucosa

How well did you know this?
1
Not at all
2
3
4
5
Perfectly
86
Q

Crohn’s disease in counselling a sibling of risk (JS)
1.1% risk
2.No increased risk
3.10% risk

A

10% risk - T - lifetime risk if either a parent or sibling is affected is 9% (Robbins)

How well did you know this?
1
Not at all
2
3
4
5
Perfectly
87
Q

Crohns diease is NOT ASSOCIATED with ? Rob p573 (JS)
1.Hydronephrosis
2.Carcinoma
3.Hip arthopathy
4.Sclerosing cholangitis
5.Sclerosing peritonitis

A

Sclerosing peritonitis - T - associated with peritoneal dialysis

How well did you know this?
1
Not at all
2
3
4
5
Perfectly
88
Q

Patient with ulcerative colitis with elevated CEA (Carcinoembryonic antigen)? Rob p574,
208,585 (JS)
1.Likely colon cancer
2.May be colon cancer but also raised in UC active inflammation
3.Active ulcerative Colitis

A

May be colon cancer but also raised in UC active inflammation - T - produced in
embryonic tissue of the gut, pancreas and liver. Elevated in 60-90% of colorectal, 50-80%
of pancreatic and 25-50% of gastric and breast cancer. Also elevated in cirrhosis, hepatitis, UC, Crohns and in smokers. Lacks specificity and sensitivity for early cancer detection but can be used in prognosis and detection of recurrence.

How well did you know this?
1
Not at all
2
3
4
5
Perfectly
89
Q

Which does not cause bowel obstruction? mRob p578 (–)
1.Tuberous Sclerosis
2.GVHD
3.Collagen vascular disorders
4.Viral infections
5.Ingested toxins

A

Collagen vascular disorders (scleroderma, dermatomyositis – muscle dysfunction –
marked dilatation of small bowel simulating small bowel obstruction)

How well did you know this?
1
Not at all
2
3
4
5
Perfectly
90
Q

In colonic polyps which is TRUE ? Rob p579 (JS)
1.Peutz Jeger is sporadic hamartomatous polyps
2.HNPCC (Hereditary Nonpolyposis Colorectal Cancer) not associated with adenoma
3.Most common type in adults is villous
4.Juvenille polyps occur in ileum

A

HNPCC (Hereditary Nonpolyposis Colorectal Cancer) not associated with adenoma - T -
HNPCC is characterised by familial carcinoma of the colon, affecting predominantly
caecum and proximal colon, which DON’T arise within adenomatous polyps.

1 Peutz Jeger is sporadic hamartomatous polyps - F - Autosomal dominant syndrome (not sporadic) characterised by multiple hamartomatous polyps
3.Most common type in adults is villous - F - Tubular adenomas are the most common
(90%) followed by tubulovillous (5-10%) then villous (1%)
4.Juvenille polyps occur in ileum F - Focal hamartomatous polyps found most frequently in the rectum in children younger than 5y

How well did you know this?
1
Not at all
2
3
4
5
Perfectly
91
Q

Adenomatous polyps are NOT a feature of ? (JS)
1.Gardners Syndrome
2.Familial Polyposis
3.Turcot’s Syndrome
4.Peutz-Jeghers Syndrome
5.Sessile Villous lesions

A

Peutz-Jeghers Syndrome - T - hamartomatous lesions which don’t have malignant
potential themselves, but patients have an increased risk of other cancers (pancreas, breast, lung, ovary and uterus)

1.Gardners Syndrome - F - AD variant of FAP with intestinal polyps, osteomas and soft
tissue tumours
2.Familial Polyposis - F - AD disorder with innumerable adenomatous polyps with
progression to adenocarcinoma in 100%
3.Turcot’s Syndrome - F - rare variant of Gardners syndrome with colonic polyposis and tumours of the CNS
5.Sessile Villous lesions - F - type of adenomatous polyps

How well did you know this?
1
Not at all
2
3
4
5
Perfectly
92
Q

Following associated with increased risk of bowel malignancy EXCEPT? Rob p579 (JS)
1.Obesity
2.Diabetes
3.Crohns
4.Meat/low fibre diet

A

diabetes

Risk factors for colorectal cancer include diet (low fibre, high intake of refined carbs, intake of red meat, reduced intake of protective micronutrients such as vitamins A, C and E), obesity, physical inactivity, family history, IBD, previous XRT, FAP, HNPCC.

How well did you know this?
1
Not at all
2
3
4
5
Perfectly
93
Q

60 year old male has colonic carcinoma which involves muscularis mucosa. No regional lymph nodes, no distant metastasis. What is prognosis? Rob p582 (–) (GC & TW)
1.100% 5 year survival
2.95% 5 year survival
3.67% 5 year survival
4.40% 5 year survival
5.10% 5 year survival

A

3.67% 5 year survival B1 = 67% (path notes); TNM staging is 90%

STAGING
Single most important prognostic indicator = extent of tumour at time of diagnosis
Modified Dukes Classification / Astler-Coller Staging:
A Limited to the mucosa (TNM – ‘T is’ (in situ))
B1 Extending into but not through muscularis propria, no nodal spread (TNM – T1)
B2 Through muscularis propria, no nodal spread (TNM – T2)
C1 Into muscularis propria + nodal spread (ie B1 + nodes)
C2 Through muscularis propria + nodal spread (ie B2 + nodes)
D Distant metastases

Staging can only be applied after resection of neoplasm and surgical exploration
Recurrences most common (10%):
at operation site, near anastomoses
in peritoneal cavity
in liver and distant organs

PROGNOSIS
Related to stage:
A = ~100% 5YS
B1 = 67% MR says for TNM staging prognosis is 90%
B2 = 54% MR says for TNM is 78%
C1 = 43%
C2 = 23%
Overall 5 year survival 40 – 50%

How well did you know this?
1
Not at all
2
3
4
5
Perfectly
94
Q

Commonest site for adenocarcinoma colon commonest to rarest are ?

A

Caecum and ascending colon (38%) > Sigmoid (35%)> transverse (18%)> descending
(8%) > multiple sites (1%)

How well did you know this?
1
Not at all
2
3
4
5
Perfectly
95
Q

Histologically appendicitis characterised by ? Rob p588 (JS)
1.Neutrophils in the mucosa
2.Neutrophils in the the muscularis propria
3.Plasma cells/lymphocytes in muscularis
4.Serosal hyperaemia

A

Neutrophils in the the muscularis propria - T - Histologic criterion is neutrophilic
infiltration of muscularis. Usually neutrophils and ulceration is also present within the
mucosa but other conditions can cause mucosal infiltrate (ie spread from GIT infection elsewhere)

How well did you know this?
1
Not at all
2
3
4
5
Perfectly
96
Q

HNPCC – barium enema is performed to look for (TW)
1.Colonic carcinoma
2.polyps
3.detect colonic hamartoma

A

Colonic carcinoma - T - HNPCC (Lynch syndrome) an AD condition, characterised by
familial carcinomas of the colon, predominantly affecting the caecum and proximal colon.
These don’t arise in adenomatous polyps.

HNPCC / Lynch sydrome is the most common of the inherited colon susceptibility syndromes.
Significantly increase risk of colon cancer and endometrial cancer as well as small risk of sever other associated cancers (ovariian, upper GUT, gastric, small bowel, biliary/pancreatic, skin, brain).
Lifetime risk of developing CRC in Lynch = 70%. Like most sporadic CRCs, Lynch cancers appear to evolve from adenomas, but when detected the adenomas tend to be larger, flatter, and are oftenmore proximal.

Accelerated adenoma-carcinoma sequence, dysplasia in flat mucosa of colon.
There are several types of DNA damage, and, correspondingly, there are many forms of DNA repair.
HNPCC results from defects in genes involved in DNA mismatch repair.
DNA repair genes themselves are not oncogenic, but they allow mutations in other genes during the process of normal cell division
Historically
this is true, however, now shown to be due to a DNA mismatch repair gene, where there is
either accelerated adenoma-Ca sequence or dysplasia in an area of flat mucosa.

How well did you know this?
1
Not at all
2
3
4
5
Perfectly
97
Q

Patient with suspected mononucleosis, which would be atypical? (JS)
1.Abnormal LFT
2.Proteinuria with viral inclusions in epithelial cells in urine
3.Groin lymphadenopathy
4.Mononuclear meningitis
5.Atypical lymphocytes on FNA

A

2.Proteinuria with viral inclusions in epithelial cells in urine - T - this occurs in CMV
infection

How well did you know this?
1
Not at all
2
3
4
5
Perfectly
98
Q

Echinococcus Granulosus, which is true? (JS)
1.20% infected have liver lesions
2.Myocardial involvement is by antigen cross reaction rather than direct invasion
3.Entry is by breach in skin and mucosal membranes
4.Initially cysts begin at microscopic leve

A

Initially cysts begin at microscopic level - T - Cysts begin at microscopic size and
progressively increase in size (can reach 10cm in 5 years)

1.20% infected have liver lesions - F - liver is the most common organ of involvement
(73%), followed by lung (14%), peritoneum (12%), kidney (6%). Can also involve spleen,
CNS, orbit, bone, bladder, thyroid, prostate and heart.
2.Myocardial involvement is by antigen cross reaction rather than direct invasion - F -
3.Entry is by breach in skin and mucosal membranes - F - caused by ingestion of tapeworm eggs in dog faeces which hatch in the duodenum

How well did you know this?
1
Not at all
2
3
4
5
Perfectly
99
Q

Acute acalculous cholecystitis which is unlikely (JS)
1.Post partum
2.HIV positive
3.TPN
4.Amyloid
5.Post surgical

A

Amyloid - F - can cause features of a low grade chronic acalculous cholecystitis due to
infiltration of gallbladder wall

How well did you know this?
1
Not at all
2
3
4
5
Perfectly
100
Q

Staging hepatocellular carcinoma, portal nodes negative, next most likely site is (JS)
1.Bone
2.Lungs
3.Adrenal
4.Brain
5.Spleen

A

2.Lungs - T - most common site of mets (followed by adrenal, lymph nodes and bone)
(Dahnert)

How well did you know this?
1
Not at all
2
3
4
5
Perfectly
101
Q

Patient with elevated CA 125 (JS)
1.Colon cancer
2.Ovarian cancer
3.Any ovarian pathology

A

Any ovarian pathology- T - elevated in ovarian cancer, endometrial, cervical, pancreatic,
bronchial and breast cancer. Also with hepatoma, benign ovarian disease, PID,
endometriosis, hepatitis, pancreatitis and pregnancy.

How well did you know this?
1
Not at all
2
3
4
5
Perfectly
102
Q

Amyloid, which is false (JS)
1.In TB is AA type
2.Causes Sago spleen and Lardaceous spleen
3.Seen in x% of patients with renal failure

A

Seen in x% of patients with renal failure - F - although renal involvement is common,
renal function compromise is rare (RG 2004;24(2):405-416). Get amyloid (joint deposition) if on hemodialysis (incidence 21% 2yr dialysis, 50% 4-7y, 90% 7-13y, 100% >13y).

1.In TB is AA type - T - AA type is secondary amyloidosis - Crohns, JRA, Reiter syndrome,
ank spond, familial Mediterranean fever, Sjogren, dermatomyositis, vasculitis, chronic
osteomyelitis, TB, bronchiectasis, CF, SLE etc
2.Causes Sago spleen and Lardaceous spleen - T - Sago spleen refers to deposits of amyloid
within splenic follicles producing tapioca-like granules. Lardaceous spleen is seen with
fusion of the deposits giving rise to large maplike areas of amyloidosis

How well did you know this?
1
Not at all
2
3
4
5
Perfectly
103
Q

Megaloblastic anaemia is not caused by : (JS)
1.Pernicious anaemia
2.Crohn’s disease
3.Coeliac disease
4.Pancreatic calcification

A

Pancreatic calcification – F - CME

1.Pernicious anaemia - T - results in Intrinsic factor deficiency and therefore Vitamin B12
deficiency
2.Crohn’s disease - T - Ileitis can cause impaired absorption of Vit B12
3.Coeliac disease - T - Causes malabsorption

How well did you know this?
1
Not at all
2
3
4
5
Perfectly
104
Q

Sickle cell anaemia. An unusual feature would be (JS)
1.splenomegaly
2.splenic atrophy/ absent
3.fungal dacralitis
4.aplastic crises

A

.fungal dacralitis - F - dacrylitis is caused by vaso-occlusive crisis (hypoxic injury and
infarction caused by sickling), which can be precipitated by infection

1.splenomegaly - T - splenomegaly is seen in children in the early phase of the disease
2.splenic atrophy/ absent - T - continued scarring causes progressive shrinkage -
autosplenectomy
4.aplastic crises - T - refers to temporary cessation of bone marrow activity, usually
triggered by parvovirus infection, with a rapid worsening of anaemia

How well did you know this?
1
Not at all
2
3
4
5
Perfectly
105
Q

PRV and splenomagaly —least likely (TW)
1.CML
2.ALL
3.Budd-Chiari
4.Cirrhosis

A

Cirrhosis - F – however secondary polycythemia can be caused by HCC and EPO
secretion.

1.CML - T - certain features are common to each of the four chronic myeloproliferative
disorders (CML, PCV, essential thrombocytosis, myelofibrosis).
2.ALL - T - polycythemia rubra vera can transform into ALL
3.Budd-Chiari - T - common sites for thromboses in PRV include hepatic veins, protal and
mesenteri cveins, and the venous sinuses of the brain.

How well did you know this?
1
Not at all
2
3
4
5
Perfectly
106
Q

T/F Barretts has x40 increased association of SCC

A

falase, adenocarcinoma x40 with>2cm barretts

How well did you know this?
1
Not at all
2
3
4
5
Perfectly
107
Q

Crohn’s colitis is not associated with (JS)
1.Fat wrapping
2.Aphthous ulcer
3.Cobblestone
4.Backwash ileitis
5.Fistulas

A

Backwash ileitis - T - ulcerative colitis

How well did you know this?
1
Not at all
2
3
4
5
Perfectly
108
Q

H.Pylori which is least correct
1.Gram negative spiral
2.Attaches to epithelium in the small and large intestine
3.Role in gastric cancers
4.Associated with MALT lymphoma

A

Attaches to epithelium in the small and large intestine – F - adheres to gastric epithelium.

1.Gram negative spiral – T – spiral shaped, microaerophilic, gram negative bacterium.
3.Role in gastric cancers – T – HP can cause chronic active gastritis and atrophic gastritis,
early steps in the carcinogenesis sequence. Studies have shown a clear link between HP and gastric adenocarcinoma. Source of gastric cancer may not be from gastric epithelial cells themselves, but rather from bone-marrow derived cells that differentiate into gastric epithelial cells in the presence of HP (UTD).
4.Associated with MALT lymphoma – T – multiple studies have shown association, and
MALToma remission following eradication of HP.

How well did you know this?
1
Not at all
2
3
4
5
Perfectly
109
Q

Which is most correct? (JS)
1.Angiodysplasia most commonly affects the sigmoid colon
2.short segment Hirschsprung most common in females
3.Meckel’s diverticulum is present on the mesenteric side
4.complications of necrotising enterocolitis include a stricture
5.pseudopolyps are a feature of Crohn’s disease

A

4.complications of necrotising enterocolitis include a stricture - T - complications of NEC
include short bowel syndrome, malabsorption, strictures and recurrence of disease

1.Angiodysplasia most commonly affects the sigmoid colon - F - most often seen in the
caecum or right colon
2.short segment Hirschsprung most common in females - F - more common in boys
3.Meckel’s diverticulum is present on the mesenteric side - F - form on the antimesenteric side of the small bowel
5.pseudopolyps are a feature of Crohn’s disease - F - can occur in Crohns but more typical of UC

How well did you know this?
1
Not at all
2
3
4
5
Perfectly
110
Q

Patient has a peptic ulcer with increased serum gastrin levels, what would you scan for:
(TW, GC, JS)
1.Pancreas and duodenum
2.Pancreas, duodenum and lymph nodes
3.Mass in duodenum

A

Pancreas, duodenum and lymph nodes – T – although the majority of gastrinomas in ZES
and MEN-1 are in the 1st part of the duodenum, would presumably still search remainder of gastrinoma triangle, then nodes to assess for metastatic deposits.

gastrinoma triangle – region bounded by confluence of the cystic and common bile ducts superiorly, the 2nd and 3rd portions of the duodenum inferiorly, and the neck and body of the pancreas medially – 85% of gastrinomas.

In contrast to sporatic gastrinomas, which are usually solitary lesions, gastrinomas that occur in patients with ZES and MEN-1 are usually multiple, less than 5mm in size, and located in the proximal duodenum. In ZES most (75%) are in the 1st portion of the duodenum, 14% in the distal duodenum, 11% in jejunum

How well did you know this?
1
Not at all
2
3
4
5
Perfectly
111
Q

Which of the following statements concerning gastrointestinal diseases is most correct:
(GC)
1.The colon is the most common site for GIT lymphoma
2.Inflammatory pseudopolyps are a characteristic of Crohn’colitis
3.Aphthous ulcers are a characteristic feature of ulcerative colitis
4.Juvenile polyps typically occur in the rectum

A

Juvenile polyps typically occur in the rectum T - hamartomatous proliferations; in
general they occur singly and in the rectum. Usually large in kids under 5yo. (1-3cm),
smaller in adults (called retention polyps). No malignant potential.

1.The colon is the most common site for GIT lymphoma F - stomach 50% > SB > colon >
oesophagus; multicentric in 10-50%.
2.Inflammatory pseudopolyps are a characteristic of Crohn’colitis F - of UC; represent islands of regenerating mucosa.
3.Aphthous ulcers are a characteristic feature of ulcerative colitis - false, crohsn

How well did you know this?
1
Not at all
2
3
4
5
Perfectly
112
Q

Benign gastric ulcer, which feature is MOST likely: (JS)
1.Greater curvature
2.Heaped up edges
3.Radiating folds
4.Chronic atrophic gastritis

A

Chronic atrophic gastritis - T - almost universal in patients with peptic ulcer disease,
occuring in 85-100% of patients with duodenal ulcers and 65% with gastric ulcers

1.Greater curvature - F - predominantly located along the lesser curvature around the
border between the body and antrum, less commonly along the greater curvature
2.Heaped up edges - F - rare in benign ulcers, but characteristic of malignant lesions
3.Radiating folds - F - may have scarring that involves the entire thickness of the stomach
with puckering of the surrounding mucosa, causing mucosal folds which radiate from the
crater in spokelike fashion

How well did you know this?
1
Not at all
2
3
4
5
Perfectly
113
Q

Achalasia of the oesophagus, which is false: (JS)
1.Dilated vestibule
2).25% have a dilated oesophagus and colon with Chagas
3.Oesophageal perforation is a complication
4.Most oesophageal perforations are iatrogenic

A

1.Dilated vestibule - F - vestibule is in the region of the LOS which fails to relax in
achalasia - beaked tapering at GOJ

2.25% have a dilated oesophagus and colon with Chagas – T -The digestive forms of the
disease lead to megaesophagus and/or megacolon in approximately one third of chronic
cases, of which 20-50% also present with an associated cardiopathy.
3.Oesophageal perforation is a complication – T - Pneumatic dilatation for achalasia
carries a significant and recognized risk of esophageal perforation (5%). Obtain from
patients prior to the dilatation an informed consent emphasizing this risk of perforation.
4.Most oesophageal perforations are iatrogenic T – treatment as above.

How well did you know this?
1
Not at all
2
3
4
5
Perfectly
114
Q

Features of scleroderma include: (JS)
1.Upper third of oesophagus is preserved
2.Colon involvement rare
3.GIT involvement is rare
4.Oesophageal perforation is common

A

1.Upper third of oesophagus is preserved T - causes atony and aperistalsis of the lower 2/3
of oesophagus with patulous LOS and GOR

2.Colon involvement rare –F - colon is involved in 40-50% with pseudosacculations,
eventual loss of haustra, marked dilatation and stercoral ulceration from retained faecal
material
3.GIT involvement is rare – F - third most common manifestation (after skin and
Raynaud’s), occurs in 40-45%
4.Oesophageal perforation is common - F - not mentioned in Dahnert, Primer or Mayo
clinic book

How well did you know this?
1
Not at all
2
3
4
5
Perfectly
115
Q

Elderly man with large bowel narrowing, biopsy showed segmental fibrosis and chronicinflammation, most likely: (TW)
1.Ischemia
2.Vasculitis
3.Carcinoma

A

1.Ischemia - T - submucosal chronic inflammation and fibrosis can lead to stricture (in chronic ischemia).

How well did you know this?
1
Not at all
2
3
4
5
Perfectly
116
Q

Angiodysplasia, most likely: (JS)
1.Most common in sigmoid
2.Can’t be seen macroscopically because covered mucosa
3.Cause of 20% of significant lower GI bleeding, either as massive blood loss of chronic loss
4.Not associated with other GIT lesions or CVS abnormalities

A

3.Cause of 20% of significant lower GI bleeding, either as massive blood loss of chronicloss - T - account for 20% of significant lower intestinal bleeding

1.Most common in sigmoid - F - most common in caecum and right colon
2.Can’t be seen macroscopically because covered mucosa - F - tortuous dilations of
submucosal and mucosal blood vessels, separated from the intestinal lumen by only the vascular wall and a layer of attenuated epithelial cells
4.Not associated with other GIT lesions or CVS abnormalities - F - associated with aortic
stenosis (20%)

How well did you know this?
1
Not at all
2
3
4
5
Perfectly
117
Q

Known (or previous) strongyloides infection with oedema of the mucosa of ascending
colon, what does it mean: (TW)
1.Autoinfection in the immunocompromised
2.Not related – only infects duodenum and SB
3.Ischaemia due to vascular invasion

A

.Autoinfection in the immunocompromised - T - in contrast to other helminthic parasites,
can complete life cycle entirely within human host. Autoinfection limited by intact immune
respone, however in immunocompromised - can give rise to potentially fatal infection.
During autoinfection, the rhabditiform (non-infectious) mature into filariform larvae
(infectious) within the GIT - these can then penetrate the perianal skin or colonic mucosa to complete the cycle of autoinfection

2.Not related – only infects duodenum and SB - F, but prob true in immunocompetent host -
filariform larvae of Strongyloides stercoralia found in soil or in other materials
contaminated with human faeces. Larvae penetrate skin and migrate hematogenously to lungs where they penetrate the alveolar air sacs. Larvae then ascent tracheobronchial tree and are swallowed. Then mature into adult worms and burrow into mucosa of duodenum and jejunum. Adult worms may life for up to 5y. Female adulte produces eggs, from which develop non-infectious larvae within the lumen of the GIT (which then pass into your turds).

3.Ischaemia due to vascular invasion - F - oedema and inflammation secondary to invasion

Endoscopy can show: duodenum - oedema, brown discoloration of mucosa, erythrmatous spots, subepithelial haemorrhage, and megaduodenum; colon - loss of vascular pattern, oedema, apthous ulcers, erosions, serpiginous ulcerations, and xanthoma-like lesions; stomach - thickened folds and
mucosal erosions.

How well did you know this?
1
Not at all
2
3
4
5
Perfectly
118
Q

0 yo with 20 colonic polyps, suspect FAP, which is most likely: (TW)
1.Need >100 polyps
2.Attenuated FAP
3.Polyps in Peutz Jegher restricted to SB
4.Normal patients…inheritance, sporadic mutations are rare
5.Thyroid cancer and mastoid osteoma in Gardners

A

Attenuated FAP - T - (UTD) attenuated from of FAP - affected patients have fewer than
100 colorectal adenomas and a delayed onset of colorectal cancer. Polyposis typically
develops in 2nd or 3rd decate of life. Similarly high risk, but with older average age of
cancer (54yo).

1.Need >100 polyps - F - attenuated FAP can have <100.
3.Polyps in Peutz Jegher restricted to SB - F - small intestine 64%, colon 64%, stomach
49%, rectum 32%.
5.Thyroid cancer and mastoid osteoma in Gardners - F - 20 colonic polyps. Gardners = FAP + benign manifestations: osteomas and dental abnormalities; cutenous lesions; desmoid tumors; congenital hypertrophy of th retinal pigment; adrenal adenomas; nasal
angiofibromas. And malignant manifestations: extra-colonic malignancies duodenal; thyroid; pancreatic; gastric; CNS; hepatoblastoma; small bowel distal to duodenum; possibly adrenal.

How well did you know this?
1
Not at all
2
3
4
5
Perfectly
119
Q

Giardia Diagnosis (TW)
1.Stomach / Proximal bowel
2.Helminth
3.Normal commensal in colon
4.Common in neonatal ICU

A

Stomach / Proximal bowel – T – probably best answer – G. lamblia attach to mucosal
surface of duodenum and jejunum.

2.Helminth – F – flagellated protozoan parasite.
3.Normal commensal in colon - F
4.Common in neonatal ICU – F – especially common in areas where there are poor sanitary conditions and insufficient water treatment facilities. Hopefully not in NICU, unless feeing the bubs infected turds.

How well did you know this?
1
Not at all
2
3
4
5
Perfectly
120
Q

Alcoholic and cachetic patients with Bacteroides (TW)
1.Aspiration from mouth organism
2.Commensal from mouth
3.Aspiration from stomach bug
4.Bowel diverticulitis
5.Osteomyelitis

A

Aspiration from mouth organism - T - major anaerobic bacteria implicated in human
disease include organisms from genus Bacteroides, Fusobacterium, Porphyromonas, and Prevotella (all gram-neg bacilli), clostridium, actinomyces, eubacterium (gram positive bacilli), peptostreptococcus and peptococcus (gram pos cocci), and veillonella (gram neg coccus). Thes organisms are normally found in oral cavity and they may result in disease when aspirated. Therefore, conditions that favor the occurance of aspiration, eg impaired consciousness, seizure, stroke, drug ingestion, and alcoholism, or conditions that impair ability to clear aspirated secrtions favor development of pulmonary anaerobic infections.

How well did you know this?
1
Not at all
2
3
4
5
Perfectly
121
Q

Mid oesophageal stricture 40 yo female, most likely: (JS)
1.If adeno ca likely long segment Barretts
2.SCC only in <5% here
3.SCC there are 4 patterns: multifocal, luminal, polypoid, ulcer

A

1.If adeno ca likely long segment Barretts - T - Barretts is a cause of mid-oesophageal
strictures and results in adenocarcinoma (although it is usually distal)

2.SCC only in <5% here - F - for SCC, 20% in upper third, 50% middle third, 30% lower
third
3.SCC there are 4 patterns: multifocal, luminal, polypoid, ulcer - F - three patterns -
protruded (polypoid exophytic), flat (diffuse infiltrative) and excavated (ulceration)

How well did you know this?
1
Not at all
2
3
4
5
Perfectly
122
Q

bowel cancer risks (TW)
1.Peutz Jeghers
2.FAP nephew
3.5 villous polpys excised previously

A

Peutz Jeghers – F – non-neoplastic polyp (hamartoma). Has increased risk of pancreatic, breast, lung, ovary and uterine cancer, but not colon cancer (path notes). UTD says increased risk.

2.FAP nephew – T – Autosomal dominant. Polyposis typically develops in 10-30yo. 100%
malignant transformation by 20y after Dx; age at carcinomatous development usually 20- 40yo. This option would be only be true if accounting for appropriate age spread (between siblings [parent of nephew] + age difference with nephew).
3.5 villous polpys excised previously – T – as is seen in colon cancer, increasing size of
adenomas and the presence of villous features are both risk factors for the development of invasive carcinoma within an adenoma.

How well did you know this?
1
Not at all
2
3
4
5
Perfectly
123
Q

how to DX Giardia (? False) (TW)
1.total villous atrophy in jejunum
2.oocyst in stool
3.subtotal villous atropy
4.something about aspirate

A

1.total villous atrophy in jejunum – F – spectrum of findings ranging from none to mild to moderate to subtotal villous atrophy in severe cases (UTD). Although the intestinal
morphology may range from virtually normal to markedly abnormal, most commonly
Giardia causes clubbing of villi, a decreased villus-crypt ratio, and a mixed inflammatory
infiltrate of the lamina propria (Robbins).

Giardia lamblia is a flagellated protozoan parasite. Ingest cysts, excystation occurs in upper small bowel releasing trophozoites. These have adhesive disc for attachment to mucosal surface of duodenum and jejunum – does not invade or cause necrosis of mucosal epithelium – just a ‘resident’. Causes diarrhoea and malabsorption by mechanisms unknown. For the organism to persist in the infectious form, trophozoites must revert to cysts, which occurs in the large intestine.

How well did you know this?
1
Not at all
2
3
4
5
Perfectly
124
Q

If you can only do a sigmoidoscope % cancers missed are (TW)
1.5-15%
2.15-25%
3.40-60%
4.80-90%

A

40-60% - T – best answer.

How well did you know this?
1
Not at all
2
3
4
5
Perfectly
125
Q

H. Pylori (TW)
1.Associated with gastritis but most are asymptomatic
2.Causes a duodenitis more than a gastritis
3.Affects 1-2 % of adults

A

Associated with gastritis but most are asymptomatic – T – majority of infected individual are asymptomatic.

2.Causes a duodenitis more than a gastritis – F – in the duodenum H Pylori are confined to
foci of gastric metaplasia. Gastric metaplasia refers to the presence of gastric epithelium in
the 1st portion of the duodenum. Metaplastic foci provide areas for HP colonization, and
probably have a role in the development of duodenitis. HP is almost always accompanied by gastritis and the diagnosis should be suspect in its absence.
3.Affects 1-2 % of adults – F – Conservative estimates suggest 50% of the worlds
population is affected. In developing nations – majority of kids infected before 10yo,
prevalence of adults peaks at more than 80% before 50yo. Developed countries: infection in kids is usual, but becomes more common in adulthood, 50% in those >60yo.

How well did you know this?
1
Not at all
2
3
4
5
Perfectly
126
Q

.No meconium, complete micro colon, which is least correct? (TW)
1.Long segment hirschsrpungs
2.CF
3.Functional immaturity of the colon
4.Ilieal atresia

A

3.Functional immaturity of the colon – F – AKA meconium plug syndrome / small left colon
syndrome. Functional immaturity of ganglion cells in colon causing functional obstruction
of newborn colon. No microcolon.

1.Long segment hirschsrpungs – T – total colonic Hirschsprung – entire colon may be
small.
2.CF – F – meconium ileus is presenting finding in 10% of CF and is virtually
pathognomonic of the disease. Essentially all patients with MI have CF. Microcolon present
due to non-use. Ileal meconium plugs.
4.Ilieal atresia – T – Often no findings to Dx ileal atresia from other causes of distal bowel
obstruction, so require contrast enema. Get microcolon (however if atresia occurred late in fetal life, microcolon may not be present and colon may contain some meconium).

How well did you know this?
1
Not at all
2
3
4
5
Perfectly
127
Q

.GIST tumour – which is correct: (TW)
1.Malignant > benign
2.Histological subtype most important in prognosis
3.Occur mainly in stomach and small bowel
4.Grow as an epithelial tumour

A

Occur mainly in stomach and small bowel - T- stomach most common site (2/3). Small
bowel (esp duodenum) next most common site. Can occur anywhere in GI tract, and rarely occurs in oesophagus (where leiomyoma is more common).

1.Malignant > benign - F - No GIST can truly be labelled as benign. Most common
mesenchymal tumor of GIT. It is widely accepted that terms “benign” or “malignant” should not be applied to GIST, since these terms are not clinically useful for patient management (UTD). Clinical behaviour of GISTs is variable. Approximately 50% of completely resected GISTs can be expected to recur within 5y of follow up. All GISTs are now regarded as potentially malignant.
2.Histological subtype most important in prognosis - F - histo falls into 3 categories: spindle cell type 70%; epithelioid type 20%; mixed type 10%. Prognosis however is influenced by tumor size, mitotic rate, tumor site (sm intesting worse than stomach), and the completeness of resection
4.Grow as an epithelial tumour - F - GISTs are the most common nonepithelial tumors of the GIT. Submucosal tumor of GIT derived from interstitial cells of Cajal (which normally
regulate peristaltic activity) & express CD117 (KIT) and 2/3 also express CD34. GISTs are
of mesenchymal origin and are not related to leiomyomas or leiomyosarcomas.

How well did you know this?
1
Not at all
2
3
4
5
Perfectly
128
Q

Leiomyoma, least common site: (TW)
1.Heart
2.Myocardium
3.Blood vessels, vascular smooth muscle
4.Oesophagus
5.Stomach
6.Uterus

A

Myocardium – F – smooth muscle is found in the walls of all the hollow organs of the
body except the heart.

Heart – T – leiomyomas can be intravascular and extend to the heart. Primary IVC
leiomomyoma / uterine leiomyoma with intravascular extension.
Blood vessels, vascular smooth muscle – T – see answer 1. Leiomyomas of the head and
neck probably arise from vascular smooth muscle.

Leiomyomas represent the most common gynecologic and uterine neoplasms. Occusionally occur with unusual growth patterns or in unusual locations (eg diffuse periotenal leiomyomatosis, intravenous leiomyomatosis, benign metastasizing leiomyomas, retroperitoneal leiomyomas,
parasitic leiomyomas).

How well did you know this?
1
Not at all
2
3
4
5
Perfectly
129
Q

Which is not associated with Hamartoma? (JS)
1.Peutz-jeghers.
2.Cronkhite-canada.
3.Cowdens syndrome.
4.Turcots.

A

Turcots. - T - rare variant of FAP with colonic polyposis (adenomas not hamartomas) and
tumours of the CNS

How well did you know this?
1
Not at all
2
3
4
5
Perfectly
130
Q

.Associations with Gardners syndrome, which is false? (JS)
1.Desmoid tumour
2.Retinal anomalies
3.Brain tumours
4.Polyps

A

3.Brain tumours. - F - Turcot syndrome is a variant of FAP/Gardner syndrome with
polyposis and tumours of the CNS

1.Desmoid tumour. - T - Soft tissue tumours including sebaceous/epidermoid inclusion
cysts, fibromas, desmoid tumours, mammary fibromatosis, keloid formation
2.Retinal anomalies. - T - hypertrophy of retinal pigment epithelium (doesn’t affect vision) (Path outlines)
4.Polyps. - T - multiple colonic polyps

How well did you know this?
1
Not at all
2
3
4
5
Perfectly
131
Q

.Which is not a feature of Zollinger-Ellison syndrome? (GC)
1.Hypervascular GI mass.
2.Gastrinoma.
3.Increased ulcers.
4.Increased lymphoma in Gastric wall.
5.Atrophic Gastritis.

A

Increased lymphoma in Gastric wall. F - complications include: malignant islet cell
tumour (in 60%), liver mets with continued gastric secretion, perforated DU/jejunal ulcer,
oesoph stricture with reflux, obstruction, GI bleeding, gastric carcinoids (esp. in MEN1).

Gastric carcnoid tumours may be classified as:
Type I: ECL cell hyperplasia, hypergastrinaemia, chronic atrophic gastritis +/- pernicious anaemia; generally benign disease. ie. suspect if polyps found in a pt with chronic atrophic gastritis.
Type II: seen in the hypergastrinaemic state of ZES in assocn with MEN1; also arise from ECL
cells in the setting of hyperplasia; multicentric. ie. multiple masses in setting of diffuse gastric wall thickening.
Type III: sporadic tumours not assocd with hypergastrinaemia; large solitary, may show
ulceration; more likely to metastasise.

How well did you know this?
1
Not at all
2
3
4
5
Perfectly
132
Q

Which is most true of Carcinoid? (GC)
1.Need liver mets for carcinoid syndrome.
2.10% occur in terminal ileum.
3.Arise from enterochromaffin and Kulchitsky cells.
4.Hypervascular mass on CT in arterial phase

A

Arise from enterochromaffin and Kulchitsky cells. T - GIT carcinoids arise from
enterochromaffin cells in crypts of Lieberkuhn (= argentaffinomma due to affinity for silver stain). Kulchitsky cells are generally seen in the lung and give rise to KCC1-3 carcinoid tumours.

Hypervascular mass on CT in arterial phase. T - although small solitary or multifocal carcinoids are typically not identified on CT scans, and may be hard to see on MRI (best visualized on T1 post gad + fat sat, where they manifest as nodules or focal areas of mural thickening with moderately intense gadolinium enhancement.

1.Need liver mets for carcinoid syndrome. F - Elevated levels of 5-HIAA, a metabolite, are
found in blood and urine. Normally liver deactivates vasoactive amines (serotonin,
histamine, bradykinin, others) released from carcinoid tumors; clinical symptoms occur if liver metastases are present or if tumor venous blood flow bypasses the liver (ie. primary pulmonary or ovarian carcinoids). CS occurs in 1% with carcinoid tumors, 20% with
widespread metastases. Symptoms: vasomotor disturbances (cutaneous flushes, cyanosis of face and anterior chest, intermittent hypertension), palpitations, intestinal hypermotility (nausea, vomiting, diarrhea, cramps); also asthmatic attacks with bronchospasm, fibrosclerosis of AV and tricuspid valves, elastotic sclerosis of mesenteric vessels causing ischemia, dermal sclerosis, hepatomegaly
2.10% occur in terminal ileum. F - 25-35% occur in small bowel, of these, 91% occur in
distal ileum.

How well did you know this?
1
Not at all
2
3
4
5
Perfectly
133
Q

.Features of diverticular disease include (TW)
1.Multiple true diverticulae
2.Circular muscular hypertrophy
3.Sacro-ilieitis
4.Increased risk of colonic cancer

A

2.Circular muscular hypertrophy - T - important feature of colonic diverticula =
hypertrophy of the circular of the muscularis propria.

1.Multiple true diverticulae - F - acquired diverticula lack or have an attenuated muscularis propria (cf congenital diverticula / true diverticula which have all three layers of bowel wall). Tend to occur alon gthe taeniae coli. Colonic diverticula are thin walled and are composed of a flattened or atrophic mucosa, compressed submucosa and attenuated or totally absent muscularis propria.
3.Sacro-ilieitis - F - IBD, seronegative arthritis.
4.Increased risk of colonic cancer - ?F - the natural Hx of diverticulosis: 70%
asymptomatic; 5-15% diverticular bleeding; 15-25% diverticulitis (of this 15-25%, 3/4 are
simple, and 1/4 are complicated ie. abscess, obstruction, perforation, fistula). UTD says
there may be a relation between diverticulosis and colon cancer. One series of 7000 patients found an excess number of colon and rectal cancers in the first 2 yrs after the Dx of diverticular disease, but not with more prolonged follow-up. There was, however, a longterm excess of left-sided colon cancers, suggesting a possible relation between these tumors and diverticular disease

How well did you know this?
1
Not at all
2
3
4
5
Perfectly
134
Q

Which of the following is not associated with ulcerative colitis: (GC)
1.crypt abscess
2.continuity
3.cobblestone pattern
4.primary sclerosing cholangitis
5.uveitis

A

3.cobblestone pattern F - characteristic of Crohn’s (linear ulcers, relatively spared
intervening mucosa develops a coarse cobbletstone appearance). UC: pseudopolyps
(islands of regenerating mucosa), usually normal wall thickness & normal serosa.

Diffuse disease limited to colon, rectal involvement with continuous proximal involvement; no skip
lesions; no deep fissural ulcers; no transmural sinus tracts, no transmural lymphoid aggregates or
granulomas.
Extraintestinal manifestations: migratory polyarthritis, sacroiliitis, ankylosing spondylitis,
pyoderma gangrenosum, clubbing of fingertips, primary sclerosing cholangitis, pericholangitis,
uveitis, cholangiocarcinoma (rare).

1.crypt abscess T - neutrophils in glandular lumen. Image: crypt abscess causing mucosal
stippling.
2.continuity T - diffuse distribution, cf. skip lesions in Crohn’s.
4.primary sclerosing cholangitis T - feature of UC (50-74%) and CD (13%).
5.uveitis T - feature of both UC and CD.

How well did you know this?
1
Not at all
2
3
4
5
Perfectly
135
Q

Which of the following is not a feature of Zollinger-Ellison syndrome:
1.Hypervascular pancreatic mass
2.gastrinoma
3.multiple peptic ulcers
4.atrophic gastritis
5.lymphoma of gastric mucosa

A

5.lymphoma of gastric mucosa F - complications include: malignant islet cell tumour (in 60%), liver mets with continued gastric secretion, perforated DU/jejunal ulcer, oesoph stricture with reflux, obstruction, GI bleeding, gastric carcinoids (esp. in MEN1).

Gastric carcnoid tumours may be classified as:
Type I: ECL cell hyperplasia, hypergastrinaemia, chronic atrophic gastritis +/- pernicious anaemia; generally benign disease. ie. suspect if polyps found in a pt with chronic atrophic gastritis.

Type II: seen in the hypergastrinaemic state of ZES in assocn with MEN1; also arise from ECL
cells in the setting of hyperplasia; multicentric. ie. multiple masses in setting of diffuse gastric wall thickening.

Type III: sporadic tumours not assocd with hypergastrinaemia; large solitary, may show
ulceration; more likely to metastasise.

How well did you know this?
1
Not at all
2
3
4
5
Perfectly
136
Q

Which of the following is least frequently associated with malignancy: (GC)
1.Tubular adenoma
2.Peutz-Jegher syndrome
3.Villous adenoma
4.Gardner’s syndrome
5.Turcot’s syndrome

A

1.Tubular adenoma F - all degrees of dysplasia may be encountered; risk of subsequent adenomas or CRC is related to size; higher risk if 6-10mm or larger, multiple adenomas or family history.

2.Peutz-Jegher syndrome T - AD disease with incomplete penetrance (instestinal polyposis
+ mucocutaneous pigmentation); increased risk of developing cancer of GIT, pancreas,
breast, ovary, endometrium, testis; risk approaches 40% by 40yo.
3.Villous adenoma T - all degrees of dysplasia may be encountered; higher risk of
malignancy than tubular adenoma; 40% risk if sessile and >4cm. Synchronous Ca is found
in as many as 40% of these, the frequency being related to the size of the polyp.
4.Gardner’s syndrome T - AD disease (variant of FAP) with osteomas + soft tissue tumours.
Malignant transformation of colonic polyps in 100%.
5.Turcot’s syndrome T - AR disease with colonic polyposis and CNS tumours (esp.
supratentorial GBM, occ. medulloblastoma). Malignant transformation of colonic polyps in
100%; death from brain tumour in 2nd/3rd decade.

How well did you know this?
1
Not at all
2
3
4
5
Perfectly
137
Q

Which is not a feature of scleroderma (TW)
1.Dilated atonic oesophagus with distal stricture
2.Multiple SB sacculation
3.Barrett’s oesophagus in 30-40%
4.Raynaud’s
5.Sclerodactyly

A

1.Dilated atonic oesophagus with distal stricture - F - probably least correct. B&H: stiff
dilated oesophagus that does not collapse with emptying, and wide gaping LES with free
FOR. Despite free refulx, tight strictures of distal oesophagus are uncommon. Danhert says ‘fusiform stricture 4-5cm above GOJ from reflux oesophagitis”.

2.Multiple SB sacculation - T - pseudodiverticula in small bowel - asymmetric sacculations with squared tops and broad bases on mesenteric side (due to eccentric smooth muscle atrophy).
3.Barrett’s oesophagus in 30-40% - T - 30% incidence. UTD says prevalence of Barrett’s
metaplasia in PPI-treated systemic sclerosis patients is similar to that in other patients with GOR (13% in one study).
4.Raynaud’s - T - Raynaud may precede other symptoms by months/years
5.Sclerodactyly - T - “tapered fingers” = atrophy + resorption of soft tissues of fingertips + soft tissue calcifications

How well did you know this?
1
Not at all
2
3
4
5
Perfectly
138
Q

Which is not a non-neoplastic polyp? (JS)
1.Hamartoma
2.Juvenile polyp
3.Hyperplastic polyp
4.Tubular adenoma

A

Tubular adenoma - T - a form of adenomatous polyp which result from epithelial proliferative dysplasia ie neoplastic epithelial lesion

How well did you know this?
1
Not at all
2
3
4
5
Perfectly
139
Q

2.Which of the following has the lowest risk of malignant transformation? (JS)
1.Peutz Jegher hamartoma
2.Villous adenoma
3.Polyp in Gardner’s syndrome
4.Tubular adenoma
5.Polyp in Turcot’s syndrome

A

1.Peutz Jegher hamartoma - T - don’t have malignant potential themselves, but the patients are at increased risk of developing other cancers (pancreas, breast, lung, ovary and uterus)

2.Villous adenoma - F - all adenomas arise due to epithelial dysplasia and therefore have
malignant potential. Risk of cancer in villous adenomas >4cm is 40%
3.Polyp in Gardner’s syndrome - F - as for FAP, 100% progression to adenocarcinoma
4.Tubular adenoma - F - cancer is rare in tubular adenomas less than 1cm but there is still a risk
5.Polyp in Turcot’s syndrome - F - as for FAP, 100% progression to adenocarcinoma

How well did you know this?
1
Not at all
2
3
4
5
Perfectly
140
Q

Which is false regarding acute pancreatitis? (TW)
1.15% of people with gallstones will develop acute pancreatitis
2.Associated with the SPINK 1 gene
3.Associated with trypsinogen activation
4.Coxsackie virus
5.Macroscopic chalky white due to fat necrosis

A

1.15% of people with gallstones will develop acute pancreatitis - F - UTD: gallstones
(including microlithiasis) account for 35-40% of cases of pancreatitis. However, only 3-7%
of patients with gallstones develop pancreatitis.

2.Associated with the SPINK 1 gene - T - serine protease inhibitor Kazal type 1 (SPINK1)
which may act as a disease modifier has a low penetrance. Other genetic causes: CFTR
(CF), mutations at codons 29 and 122 of cationic trypsiongen gene, PRSS1.
3.Associated with trypsinogen activation - T - see ans 2.
4.Coxsackie virus - T - numerous infectious agents can cause pancreatitis. Viruses: mumps, coxsackievirus, hep B, CMV, VZV, HSV. Bacteria: mycoplasma, legionella, leptospira, salmonella. Fungi: aspergillus. Parasites: toxo, crypto, ascarias.
5.Macroscopic chalky white due to fat necrosis - T - fat necrosis. Released fatty acids
combine with calcium to form insuluble salts that precipitae in situ. Pancreatic substance
exhibits areas of blue-black haemorrhage interspersed with foci of yellow-white chalky fat necrosis

How well did you know this?
1
Not at all
2
3
4
5
Perfectly
141
Q

Diverticular disease causes partial bowel obstruction due to… (TW)
1.Diverticulitis
2.Hypertrophy of the circular layer of muscularis propria
3.Vesicocolic fistula

A

2.Hypertrophy of the circular layer of muscularis propria - T - taeniae coli are also usually prominent. Constipation and diarrhoea can result from the hypertrophy.

How well did you know this?
1
Not at all
2
3
4
5
Perfectly
142
Q

18 year old girl presents with a large mixed solid and cystic pancreatic mass, what is most
likely: (TW)
1.Neuroblastoma
2.Solid cystic papillary tumour
3.Mucinous cystadenoma
4.Mucinous cystadenocarcinoma
5.Microcystic adenoma

A

2.Solid cystic papillary tumour - T - Pancreatic mass of low malignant potential with solid
and cystic features. Well-demarcated large mass. Commonly in body and/or tail. <35yo.
F>M 9.5x. African-Americans or other non-Caucasian groups.

3.Mucinous cystadenoma - F - mucinous cystic pancreatic tumor (mucinous macrocystic neoplasm, macrocystic adenoma, mucinous cystadenoma or cysadenocarcinoma). Thickwalled, uni-/multilocular low grade malignant tumor composed of large, mucin-containing
cysts. Likes the body or tail of pancreas.F>M 9x. Mean age 50yo (20-95yo). 50% occur
between ages 40-60yo.
4.Mucinous cystadenocarcinoma - F - see option 3.
5.Microcystic adenoma - F - serous cystadenoma (glycogen-rich or micro-/macrocystic serious adenoma). Benign pancreatic tumor that arises from acinar cells. Honeycomb or sponge-like mass in pancreatic head (microcystic cystadenoma). Or as Steve Drew says “like the surface of a cut orange”. Mean age 65yo. Middle and elderly age groups. F>M 4x.

How well did you know this?
1
Not at all
2
3
4
5
Perfectly
143
Q

Ectopic pancreas least likely location (TW)
1.Stomach
2.Colon
3.Ileum
4.Jejunum
5.Duodenum

A

colon
May occur anywhere in the GIT. Most frequent locations are the stomach, duodenum or proximal
part of small intestine.

How well did you know this?
1
Not at all
2
3
4
5
Perfectly
144
Q

1.Which is least correct regarding scleroderma? (TW) - but dubious.
1.Involves lower oesophagus
2.Sacculations on anti-mesenteric side of small bowel
3.Associated with Barrett’s oesophagus
4.Sclerodactyly

A

Sacculations on anti-mesenteric side of small bowel - F - pseudodiverticula = asymmetric
sacculations with squared tops and broad bases on mesenteric side (due to eccentric smooth muscle atrophy)

1.Involves lower oesophagus - T - normal peristalsis above aortic arch (where striatued
muscle is present), hypotonia / atony and hypokinesia / aperistalsis in lower 2/3rd.
3.Associated with Barrett’s oesophagus - T - Barretts is a complication of chronic GOR
70%. Erosions and superficial ulcers (from asymptomatic reflux oesophagitis: No protective oesophageal contraction). Complications peptic stricture, aspiration, Barrett oesohagus, adenocarcinoma. Barretts occurs in about 30% of patients.
4.Sclerodactyly - T - “tapered fingers” = sclerodactyly = atrophy and resorption of soft
tissues of fingertips and soft-tissue calcifications.

Scleroderma encompasses a spectrum of related disorders, most of which share a characteristic clinical feature of skin thickening due to an excess of collagen fibers. Classification system takes into account the different potential complications, prognoses, and management strategies for these
disorders.

Simplest division:
Localised scleroderma - Linear scleroderma (abnormalities of skin and subcut tissue often
following dermatomal distribution and are found predominantly on 1 side of body; En coup de sabre (type of linear scleroderma = lesions look like sabre blow); Morphea (localized or genarlized - patches of sclerotic skin).
Systemic scleroderma - “Systemic sclerosis” (emphasizes frequent involvement of internal organs is generally the most important manifestation of these conditions) - Diffuse cutaneous SSc; Limited cutaneous SSc; SSc sine scleroderma (only internal organ involvement); Environmentally induced
scleroderma; Overlap syndrome where SSc coexist with elements ofther rheumatic disorders.

How well did you know this?
1
Not at all
2
3
4
5
Perfectly
145
Q

Fibrolamellar HCC, which is False (JS)
1.Female>Male
3.No association with hepatitis B
4.Hard scirrhous tumour
5.No cirrhosis

A

Female>Male - F - M=F
2.20-40 yo - T - occurs in young adults 20-40y (Robbins)

How well did you know this?
1
Not at all
2
3
4
5
Perfectly
146
Q

Cholangiocarcinoma which is true (JS + GC)
1.Bile stained
2.Lymph node mets in 30%
3.Male>Female
4.Moderate –well differentiated sclerosing adenocarcinoma
5.Only arise in liver parenchyma

A

Moderate –well differentiated sclerosing adenocarcinoma - T - resemble adenocarcinomas elsewhere in body. Most are moderately differentiated sclerosing adenocarcinomas with clearly defined glandular and tubular structures (Robbins, Pathoutlines)

1.Bile stained - F - arises from bile duct epithelium, so doesn’t produce bile (unlike HCC)
2.Lymph node mets in 30% - F - 50% metastatic to perihilar, peripancreatic and paraaortic nodes. 50-75% met to regional LN, lungs, vertebrae, adrenals, brain and elsewhere (pathoutlines)
3.Male>Female - F - according to pathoutlines, no gender predilection5.Only arise in liver parenchyma - F - may be intra or extrahepatic (involving CHD, CBD or
cystic duct)

147
Q

Hepatitis D can only cause disease in the presence of… (JS)
1.Hep A
2.Hep B
3.Hep C
4.EBV

A

2.Hep B - T - HDV is absolutely dependent on the genetic information provided by HBV for
multiplication and causes hepatitis only in the presence of HBV

148
Q

Which of the following has the lowest risk of malignant transformation? (GC)
1.Peutz Jegher
2.Villous adenoma
3.Polyp in Gardner’s syndrome
4.Tubular adenoma
5.Polyp in Turcot’s syndrome

A

4.Tubular adenoma - F - all degrees of dysplasia may be encountered; risk of subsequent adenomas or CRC is related to size. <5mm 0.5%, 5-9mm 1%, 10-20mm 5-10%, >20mm 10- 50%. Most adenomas are tubular (75%) and are usually <10mm in diameter.

– be wary
of this question, re: polyp and malignant transformation, vs malignancy in general.

1.Peutz Jegher hamartoma - T - AD disease with incomplete penetrance (intertestinal
polyposis and mucocutaneous pigmentation); increased risk of developing cancer of GIT,
pancreas, breast, ovary, endometrium, testis; risk approaches 40% by 40yo. Colonic polyps
do not have malignant potential - hence if this question was ‘least frequently associated with
colonic malignancy’ this would be best answer.
2.Villous adenoma - T - all degress of dysplasia may be encountered; higher risk of
malignancy than tubular adenoma; 40% risk if sessile and >4cm. Synchronous Ca is found
in as many as 40% of these, the frequency being related to the size of the polyp.
3.Polyp in Gardner’s syndrome - T - AD disease (variant FAP) with osteomas + soft tissue tumors. Malignant transformation polyps 100%.
5.Polyp in Turcot’s syndrome T - AR disease with colonic polyposis and CNS tumors (esp
supratentoria GBM, occ medulloblastoma). Malignant transformation of colonic polypa in
100%.

149
Q

Clear cells are not seen in… (GC)
1.Xanthogranulomatous pyelonephritis
2.Adrenal adenoma
3.Adrenal carcinoma
4.Wilms tumour
5.Clear cell carcinoma

A

Wilms tumour - F - triphasic with undifferentiated blastema (cellular with small blue cells with scanty cytoplasm).

1.Xanthogranulomatous pyelonephritis - T - replacement of renal parenchyma with foamy
histiocytes (also occ. MNGC and inflammatory cells).
2.Adrenal adenoma - T - resemble zona fasciculata cells due to numerous lipid-laden clear cells.
3.Adrenal carcinoma - T - clear or vacuolated cells comprising 25% or less of tumor.
5.Clear cell carcinoma - T - kidney, pancreas, ovary, vagina, cervix, urethra, salivary
glands…

150
Q

.Which disease is best explained by combination of Cirrhosis and emphysema (TW)
1.Alpha 1 AT deficiency
2.Wilsons

A

.Which disease is best explained by combination of Cirrhosis and emphysema (TW)
1.Alpha 1 AT deficiency - T - the main clinical manifestsions relate to 3 separate organs:
the lung, liver and musch less often the skin. In lung, severe deficiency of AAT predisposes
to COPD, esp panacinar emphysema (predominant basal involvement). Damage is
primarily due to destruction of elastin by elastase, the activity of which is increased because of deficiency of elastase inhibitory AAT. Liver damage is different, and is caused by pathologic polymerization of the variant AAT, resulting in intrahepatocyte accumulation of AAT molecules, rather than a proteolytic mechanism (AAT in hepatocytes stain with PAS). Skin disease = necrotizing panniculitis.
PI prefix, M, Z and S alleles. PIZZ severe. PIMM normal. PIMZ heterozygous. PISZ sx but PISM normal.

2.Wilsons - F - cause chronic hepatitis, portal hypertension, acute liver failure.
Neuropsychiatric disease. Fanconi syndrome. Nephrolithiasis. Cardiac arrhythmias.

151
Q

Which of the following IS LEAST correct in regards to hepatocellular carcinoma (HCC)?
6.HCC accounts for 90% of all primary liver cancers
7.The global distribution of HCC is strongly linked to prevalence of hepatitis B infection
8.In the cirrhotic liver HCC may arise within dysplastic nodules
9.HCC may have unifocal, multifocal or diffuse growth patterns, all of which have a
propensity to vascular invasion
10.Fibrolamellar carcinoma, as distinct variant of HCC, is associated with a less favourable prognosis

A

Fibrolamellar carcinoma, as distinct variant of HCC, is associated with a less favourable
prognosis

HCC
Overall, death occurs in 6/12 (30% 5 yr survival)  cachexia, GI or oesophageal variceal
bleeding, liver failure with hepatic coma
Fibrolamellar Variant
Often resectable
60% 5 yr survival

152
Q

FNA of a liver lesion shows atypia of the hepatocytes. This is consistant with ?
1.Well-differentiated HCC
2.FNH
3.Adenoma
4.Niemann-Pick Dz
5.Alcoholic cirrhosis and fatty change

A

well differentiated hcc

Note : Well Differntiated HCC is principally distinguished from borderline foci/nodules, from which it may arise (nodule in a nodule), by a nuclear density greater than twice normal and by mild but definite nuclear atypia (hyperchromasia, irregular nuclear contours)

153
Q

A 32-year-old man with weight loss for investigation has a 3 cm non-specific hepatic
nodule on CT. Fine needle aspirates are non-diagnostic and a core biopsy is performed. The report describes; “ abnormal spindle cells lining cleft-like dilated jagged vascular spaces. These lack an endothelial lining and are surrounded by similar spindle cells.” The most likely diagnosis is:
1.A cavernous haemangioma of the liver
2.A capillary haemangioma of the liver
3.Metastatic Kaposi Sarcoma
4.Metastatic angiosarcoma
5.Glomangioma of the liver (hepatic glomus tumour)

A

kaposis

Kaposi sarcoma
MIcroscopic examination discloses only dilated perhaps irregular and angulated blood vessels lined by endothelial cells with an interspersed infiltrate of lymphocytes, plasma cells, and macrophages (sometimes containing hemosiderin), lesions difficult to distinguish from granulation tissue. Over time, lesions in the classic disease spread proximally and usually convert into larger,
violaceous, raised plaques that reveal dermal, dilated, jagged vascular channels lined by
somewhat plump spindle cells accompanied by perivascular aggregates of similar spindled cells. Scattered between the vascular channels are red cells, hemosiderin-laden macrophages, lymphocytes, and plasma cells. Pink hyaline globules of uncertain nature may be found in the spindled cells and macrophages. Occasional mitotic figures may be present.

154
Q

Which of the following is the LEAST likely pattern of spread/invasion seen with
carcinoma of the gallbladder?
1.Extension along the cytic duct into the biliary tree
2.Invasion of the portal vein/ IVC
3.involvement of the porta hepatic lymph nodes
4.Peritoneal seeding
5.Pulmonary metastases

A

portal vein/ivc

155
Q

Which of the following is the LEAST likely to be a clinical manifestation of primary
hemochromatosis;
1.Arthritis
2.Loss of libido
3.Cardiac arrhythmias
4.Haematuria
5.Polyuria /polydipsia

A

Haematuria (only one not seen in text anywhere)

156
Q

.Recognised morphological appearances in acute fulminant hepatitis DO NOT include
which of the following?
1.Involvement of the whole liver
2.Patchy random areas of hepatic necrosis
3.Massive loss of liver substance
4.Sparring of the subcapsular hepatocytes
5.Wrinkling/ folding of the hepatic capsule

A

4.Sparring of the subcapsular hepatocytes

157
Q

.A member of your staff receives a needle stick injury. They are initially seronegative but seroconvert after exposure and six months later have HbsAg, (without HbeAg, HBV DNA or anti-HBc) on blood test. These results are best summarised as;
1.Successful eradication of Hepatitis B but with impaired immune response; persisting rise of infection
2.Successful eradication of Hepatitis B with normal immune response/ immunity
3.A carrier state but without definite persisting replication / liver damage
4.A carrier state with likely persisting replication / liver damage
5.Chronic hepatitis by definition

A

.A carrier state but without definite persisting replication / liver damage

HBsAg appears before the onset of symptoms, peaks during overt disease, and then declines to undetectable levels in 3 to 6 months
HBeAg, HBV DNA, and DNA polymerase appear in the serum soon after HBsAg, and all signify active viral replication
IgM anti-HBc becomes detectable in serum shortly before the onset of symptoms, concurrent with the onset of elevated serum transaminase levels. During months, the IgM antibody is replaced by IgG anti-HBc.
Anti-HBe is detectable shortly after the disappearance of HBeAg, implying that the acute infection has peaked and the disease is on the wane.
IgG anti-HBs does not rise until the acute disease is over and is usually not detectable for a few weeks to several months after the disappearance of HBsAg. Anti-HBs may persist for life, conferring protection; this is the basis for current vaccination strategies using noninfectious HBsAg.
The carrier state is defined by the presence of HBsAg in serum for 6 months or longer after
initial detection. The presence of HBsAg alone does not necessarily indicate replication of complete virions, and patients may be asymptomatic and without liver damage. In contrast, chronic replication of HBV virions is characterized by persistence of circulating HBsAg, HBeAg, and HBV DNA, usually with anti-HBc and occasionally with anti-HBs. In these patients, progressive liver
damage may occur

158
Q

A 60-year-old male has no cirrhosis but has imaging features suggesting post hepatic
venous obstruction. The three most common causes of this in Western societies, in no
particular order, are:
1.Severe right-sided heart failure, Budd Chiari Syndrome and constrictive pericarditis
2.Severe right-sided heart failure, Budd Chiari Syndrome, and mesothelioma
3.Budd Chiari Syndrome, mesothelioma and constrictive pericarditis
4.Budd Chiari Syndrome, constrictive pericarditis, and fibrosing mediastinitis
5.Idiopathic, Budd Chiari Syndrome and constrictive pericarditis

A

Severe right-sided heart failure, Budd Chiari Syndrome and constrictive pericarditis

159
Q

Fatty liver of pregnancy is ? Big Rob p884
1.A benign self-limiting condition
2.Associated with a spectrum of clinical significance including death
3.Associated with abnormality of transaminases only

A

2.Associated with a spectrum of clinical significance including death

Spectrum from mild hepatic dysfunction to hepatic failure, coma, and death
Usually 3rd TM
20-40% present with co-existent preeclampsia
Heterozygous deficiency of enzyme causing defect in mitochondrial fatty acid oxidation in high proportion of mothers

160
Q

CEA is least likely to be associated with:
1.cirrhosis
2.hepatitis
3.smoking
4.alcoholic cirrhosis
5.breast cancer
6.pancreatic cancer
7.RCC

A

rcc

Greater-than-normal levels of CEA may indicate:
Colon cancer
Breast cancer
Lung cancer
Pancreatic cancer
Thyroid cancer
Genitourinary carcinomas
Inflammatory gastrointestinal diseases (for example, ulcerative colitis, diverticulitis, cholecystitis,
pancreatitis)
Cirrhosis
Other liver disease
Peptic ulcer
Heavy smoking
Pulmonary infections

161
Q

Choledochal cysts
1.If jaundiced implies secondary stricture
2.Primary abnormality is stricture with secondary dilatation proximal
3.presents 10-20’s
4.pain always means pancreatitis
5.females 80%

A

females 80%

162
Q

Regarding necrosis (cell death). Which of the following is MOST CORRECT: (TW)
1.Liquefactive necrosis is a characteristic of ischaemic destruction of cardiac muscle
2.Councilman bodies in the liver in toxic or viral hepatitis is an example of apoptosis
3.The dead cell usually shows decreased eosinophilia
4.Caseous necrosis is encountered principally in the centre of an Aschoff nodule
5.Expansion of the nucleus of dead cells with unravelling of the chromatin is called
pyknosis

A

.Councilman bodies in the liver in toxic or viral hepatitis is an example of apoptosis – T –
cell injury in certain viral diseases, eg viral hepatitis, in which apoptotic cells in the liver
are known as Councilman bodies.

1.Liquefactive necrosis is a characteristic of ischaemic destruction of cardiac muscle – F –
coagulative necrosis. Implies preservation of basic outline of the coagulated cell for a span
of at least some days. MI is an excellent example in which acidophilic, coagulated,
anucleated cells may persist for weeks.
3.The dead cell usually shows decreased eosinophilia – F – increased eosinophilia.
Attributable in part to loss of the normal basophilia imparted by the RNA in the cytoplasm
and in part to the increased binding of eosin to denatured intracytoplasmic proteins.
4.Caseous necrosis is encountered principally in the centre of an Aschoff nodule – F –
Aschoff body (or nodule) is in rheumatic fever (foci of fibrinoid degeneration).
5.Expansion of the nucleus of dead cells with unravelling of the chromatin is called
pyknosis – F – pynkosis: nuclear shrinkage and increased basophilia. Karyolysis:
basophilia of chromatin may fade. Karyorrhexis: pnknotic or partially pnknotic nucleus
undergoes fragmentation.

163
Q

Regarding hemochromatosis which is least correct: (GC)
1.autosomal recessive
2.more common in females
3.accumulation is life long
4.effects are due to direct toxic effect of iron on cells
5.end stage cirrhosis and hyperbilirubinemia

A

more common in females - F - males predominate (5 to 7:1) with slightly earlier clinicalpresentation, partly because physiologic iron loss (menstruation, pregnancy) delays iron
accumulation in women.

1.autosomal recessive - T - most common form is an AR disease of adult onset caused by
mutations in the HFE gene (located on short arm of chromosome 6). Two common
mutations in the HFE gene: C282Y and H63D. Carrier frequency in Causasians of the
C282Y mutation is 1 in 70, and homozygotes 1 in 200. Approx 80% of haemochromatosis
pts are homozygous for the C282Y mutation.
3.accumulation is life long - T - starts from young age.
4.effects are due to direct toxic effect of iron on cells - T - by the following mechanisms:
lipid peroxidation by iron-catalyzed free-radial reactions; stimulation of collagen
formation; direct interactions of iron with DNA.
5.end stage cirrhosis and hyperbilirubinemia - T - iron accumulation - haemosiderin
granules in cytoplasm of periportal hepatocytes (stain blue with Prussian blue) -
progressive involvement of lobule, bile duct epithelium and Kupffer cell pigmentation - fibrous septa form - micronodular cirrhosis. High bilirubin is due to impaired secretion of
conjugated bilirubin into bile.

164
Q

Which cell is responsible for excess collagen production in cirrhosis (TW)
1.Ito cell
2.Kupffer cell
3.Hepatocyte
4.Lymphocyte

A

Ito cell - T - the hepatic stellate cell (previously called the lipocyte, Ito, fat-storing, or
perisinusoidal cell) is the primary source of extracellular matrix in normal and fibrotic
liver. These cells are located in subendothelial space of Disse. Stellate cells undergo a
transition from a quiescent vitamin A-rich cell into proliferative, fibrogenic, and contractile
myofibroblasts. Sinusoidal endothelial cells also play a role.

165
Q

Gauchers quick paff

A

results from lack of the lysosomal enzyme
glucosylceramidase and accumulation of glucosylceramide in mononuclear phagocytic
cells. In the most common, type I variant, affected phagocytes become enlarged (Gaucher cells) and accumulate in the liver, spleen, and BM, causing HSM and bone erosion. Type II and III have variable neuronal involvement.

166
Q

myelofibrosis quick paff

A

Primary myelofibrosis / chronic idiopathic myelofibrosis = chronic myeloproliferative disorder.
Get chronic myeloproliferation and atypical megakaryocytic hyperplasia. Secondary process of bone marrow fibrosis from nonclonal fibroblastic proliferation and hyperactivity induced by growth factors abnormally shed from clonally expanded megakaryocytes.
S&S: severe fatigue. Spelnomagaly (marked). Hepatomegaly. Extramedullary hematopoiesis. Osteosclerosis. periostitis. Secondary gout

167
Q

32 yo African negro with 5cm mass invading IVC, non cirrhotic liver, Which is the most
likely? (TW)
1.HCC in early stages of Hep B -
2.Atypical for HCC
3.In absence of aflatoxin exposure HCC would be unlikely
4.Cholangiocarcinoma should be considered
5.Leishmaniasis should be considered

A

In absence of aflatoxin exposure HCC would be unlikely - T - Although when considering answer 2., given history of non-cirrhotic liver, aflatoxin (mycotoxin produced by many species of Aspergillus) would be a consideration, especially when considering that HBV + aflatoxin effect is synergistic increasing the risk of HCC 60x.

Aflatoxin: mutations of the p53 tumor suppressor gene have been demonstrated in patients with
HCC who have chronically been exposed to aflatoxin.

  1. 60-90% in cirrohtic livers. majority in cirrohtic or late disease
  2. ?false - mean age decreasing in agrica
  3. too young without other rf’s
  4. causes splenomegaly massive
168
Q

ER / MRCP demonstrates irregular bile ducts, least likely cause: (GC)
1.Crohn’s
2.Hepatic artery injection with cytotoxins
3.SLE
4.Caroli’s
5.Biliary stone

A

3.SLE - F - primary biliary cirrhosis is associated with autoimmune disorders (RA,
Hashimoto’s, Sjorgren’s, scleroderma, membranous GN, coeliac disease). However, an association between PBC and SLE is rare. There are case reports of SLE patients with
autoimmune cholangiopathy [Liver 2002] or PSC [Digestive diseases 2004].

1.Crohn’s - T - PSC is strongly associated with IBD, although more commonly UC.
2.Hepatic artery injection with cytotoxins - T - hepatic a. infusion of chemo agents such as
floxuridine for palliative therapy for pts with colorectal liver mets. High rate of extraction
on first pass through the liver, producing an inflammatory fibrosing process about the
portal triads that simulates PSC. Mechanism is either direct effect of treatment or ischaemia
secondary to thrombosis of the intrahepatic arterial branches.
4.Caroli’s - T - segmental saccular / fusiform / beaded dilatation of IHBDs that retain a
communication with the biliary tree.
5.Biliary stone - T - ascending cholangitis secondary to stone, may mimic PSC.

169
Q

Intraductal mucinous pancreatic lesion – most typical: (GC)
1.No such type
2.Common in young girls
3.Common in pancreatic tail
4.Common in pancreatic head

A

Common in pancreatic head - T - head/neck&raquo_space; body/tail. Septated cystic lesion and
communication with main duct via a narrow neck suggest a branch duct-type IPMN.

Main duct IPMN may occur in a diffuse or segmental pattern.
Branch duct IPMN occurs in uncinate process&raquo_space; tail > body; usually communicates with main duct via a narrow neck; may have internal septations.
Spectrum: adenoma - borderline - in situ/invasive Ca

Features suggestive of invasive tumour:
main duct involvement, marked dilatation of main duct, diffuse/multifocal involvement
large mural nodule or solid mass, irregular thick wall.
Note that all are considered premalignant. A small lesion in the tail may be managed with distal pancreatectomy, cf. main duct or lesion in the head (Whipple’s).

170
Q

Causes of fatty change in liver: Which is false? (TW)
1.Smoking
2.DM
3.Pregnancy
4.Obesity
5.Hypoxia

A

Smoking - F

Insulin resistance has a key role in the development of hepatic steatosis, and potentially, steatohepatitis. Obesity and type 2 DM, conditions assoc with peripheral insulin resistance, are frequently observed in patients with NAFLD.
Obstructive sleep apnoea has been proposed to have a role in inducing inflammation in NAFLD.
Chronic intermittent hypoxia (in mice) showed signs of liver injury and lipid peroxidation.
Acute fatty liver of pregnancy: microvesicular fatty infiltration of hepatocytes. Thought related to inherited enzyme deficiency in beta-oxidation.

171
Q

Man from middle east with schistosomiasis: (GC)
1.Periportal fibrosis
2.ureteric fibrosis
3.Cirrhosis

A

ureteric fibrosis - T - S. haematobium most common in Middle East; causes GU disease.
Eggs laid in bladder wall incite a granulomatous response with chronic inflammation and subsequent fibrosis. Granulomatous cystitis, bladder wall calcification and bladder calculi
(also seminal vesicle and distal ureteric calcifn), distal ureteric strictures (cobra-head
pseudoureterocoele in VUJ region), pseudopolyps and ureteritis cystica. Reduced bladder capacity in fibrotic stage. SCC of bladder after 20-30yr latency period.

Flat worms (trematodes)
S. haematobium: Africa, Mediterranean, Middle East; lives in venous plexus near urinary bladder & ureters; this is the only one that doesn’t parasitize intestinal venules & spread to liver - causes urinary tract disease.
S. mansoni: Africa, Carribean, West Indies, northern S. America; lives in the IMV.
S. japonicum: China, Japan, Philipines; lives in the SMV.
Infectious cycle: fresh water snails release cercarial form into water; cercarie are mobile and penetrate human skin or mucosa, enter circulation, pass through lungs and lodge in hepatic branches of portal vein; mature into adults, copulate and migrate to colonic and rectal submucosa where female releases ova that enter colonic lumen, are defecated, and infect other snails; ova may also enter portal circulation and cause periportal fibrosis.

172
Q

Alpha-1 antitrypsin deficiency, which is incorrect: (GC)
1.Autosomal recessive
2.Von Meyenberg complexes
3.End stage get cirrhosis + cholestasis

A

Von Meyenberg complexes - F = multiple bile duct hamartomas; associated with
polycystic liver disease.

Rare AR disease, causing low serum levels of alpha-1-antitrypsin, and leading to emphysema (80%) and liver disease.
AAT is a protease inhibitor (Pi), which inhibits neutrophilic elastase released at sites of
inflammation; also inhibits trypsin.
Although there are 75 AAT forms, PiMM (normal phenotype) is present in 90% of population.
PiZZ: 1 per 7,000; have 10-15% of normal AAT levels, are at high risk for clinical disease;
accumulate AAT variant Z in endoplasmic reticulum and have slowdown in degradation pathway,
but only 10% get clinical disease.
PiZZ hepatic syndromes range from neonatal hepatitis (10%), biliary atresia (intra- or
extrahepatic), fibrosis, childhood cirrhosis; 2% develop HCC, not always assocd with cirrhosis.

173
Q

Which is not true in regards to liver disease?
1.Peliosis seen in HIV
2.Iron storage in Wilsons
3.Cholangiocarcinoma is an adenocarcinoma of the bile ducts

A

Iron storage in wilsons - actually copper (tried to trick u - did it work? you dumb dumb)

1.Peliosis seen in HIV - T - Bacillary angiomatosis and peliosis hepatis have most often involved HIV-infected individuals, otehr immunocompromised individuals, such as cancer patient and solid organ transplant recipients have also developed these manifestation.
2.Iron storage in Wilsons - F - autosomal recessive defect in cellular copper transport.
Decreased transport of copper from liver into bile, leading to copper excess. Normall Cu is
used to form ceruloplasmin (major carrier of Cu in the blood) in liver whic his released into
serum. Wilsons usually have lower ceruloplasmin levels (see below).
3.Cholangiocarcinoma is an adenocarcinoma of the bile ducts - T - cholangiocarcinoma is
an adenocarcinoma arising from bile ducts (so don’t produce bile like hepatocytes). Graywhite and firm, rarely bile stained.

Ceruloplasmin: originates as an inactive, non-copper containing form called “apoceruloplasmin”.
When Cu is added to apoceruloplasmin in liver - produces holoceruloplasmin, which is the functionally active form of ceruloplasmin. Routine measurement of ceruloplasmin measures apoceruloplasmin and holoceruloplasmin.

174
Q

Which is true regarding gallstones (T/F)? (TW & GC)
1.25% of cholesterol stones are radiopaque
2.50-75% of brown pigment stones are radiopaque
3.>50% are symptomatic
4.Marginal decrease in incidence after age of 40

A

25% of cholesterol stones are radiopaque – most correct

175
Q

A 78-year-old woman has a 7 cm multiloculated cystic lesion in the tail of the pancreas.
Her clinicians request a biopsy saying, “if it’s benign, we’ll leave it”. Which of the following
statements is most correct. (TW)
1.The presence of ciliated mucous secreting hyperchromatic cells on fine needle aspiration suggests a benign cystadenoma
2.The presence of ciliated mucous secreting hyperchromatic cells on fine needle aspiration suggests cystadenocarcinoma.
3.Core biopsy of the tumour margin is required as the criteria for malignancy is invasion of adjacent tissue rather than cell characteristics in pancreatic cystadenocarcinoma
4.Multiple core biopsies are required to assess the lesion as there is frequently heterogeneity in cystadenocarcinomas with some more indolent appearing regions.
5.It is not possible to reliably distinguish a cystadenoma from cystadenocarcinoma of the pancreas without the complete resected surgical specimen.

A

It is not possible to reliably distinguish a cystadenoma from cystadenocarcinoma of the
pancreas without the complete resected surgical specimen.

176
Q

An 85-year-old male with advanced carcinoma of the pancreas dies of a cerebro-vascular accident. Post mortem examination of the cardiac atrioventricular valves reveals the presence of small non-destructive vegetations along the line of closure of the valve leaflets. Which of the following statements about this condition ARE LEAST LIKELY correct: (–)

1.It is most commonly associated with mucinous adenocarcinomas
2.Endocardial trauma from an indwelling catheter is a well-recognised predisposing
condition
3.Pulmonary embolism frequently occurs concomitantly
4.Bacterial endotoxins play an important aetiologic role
5.The vegetations are only loosely attached to the underlying valve

A

Bacterial endotoxins play an important aetiologic role

NBTE frequently occurs concomitantly with venous thromboses or pulmonary embolism, suggesting a common origin in a hypercoagulable state with systemic activation of blood coagulation such as
disseminated intravascular coagulation.
This may be related to some underlying disease, such as a cancer and, in particular, mucinous adenocarcinomas of the pancreas.
The striking association with mucinous adenocarcinomas in general may relate to the procoagulant effect of circulating mucin, and thus NBTE can be a part of the Trousseau syndrome Lesions of NBTE, however, are also seen occasionally in association with non-mucin-producing malignancy, such as acute promyelocytic leukemia, and in other debilitating diseases or conditions
promoting hypercoagulability (e.g., hyperestrogenic states, extensive burns, or sepsis).
Endocardial trauma as from an indwelling catheter is also a well-recognized predisposing condition, and one frequently encounters right-sided valvular and endocardial thrombotic lesions
along the track of a Swan-Ganz pulmonary artery ca
In contrast to the vegetations of infective endocarditis, discussed previously, the valvular lesions of NBTE are sterile, do not contain microorganisms, and are only loosely attached to the underlying
valve

177
Q

Concerning mesothelioma, Which of the following statements IS INCORRECT:
a) Asbestos workers who smoke have a greater risk of developing mesothelioma than those who do not smoke
b) The epithelial form has a better prognosis than the sarcomatous form
c) Tumour characteristically extends into the fissures
d) Tumour can arise from the parietal or the visceral pleura
e) There is a greater risk of developing the tumour with the crocidolite-type of asbestos fibre than anthophyllite.

A

Asbestos workers who smoke have a greater risk of developing mesothelioma than those who do not smoke - false

178
Q

Concerning bronchogenic carcinoma, which of the following statements is
correct?
a) Adenocarcinoma has the strongest correlation with smoking.
b) Squamous cell carcinoma is the histologic sub-type that most often produce paraneoplastic syndrome
c) Small cell carcinoma is most responsive to chemotherapy
d) Bronchioloalveolar carcinoma is a sub-type of large cell carcinoma
e) There is no increased risk of developing the tumour in asbestos workers

A

Small cell carcinoma is most responsive to chemotherapy (Small cell is sensitive to
radiotherapy & chemotherapy, with potential cure rates of 15 – 25 % for localized disease)

179
Q

Concerning pulmonary tuberculosis, which of the following statements IS
INCORRECT?
a) Primary infection is usually asymptomatic
b) The Ghon focus is characteristic of post-primary (secondary) tuberculosis
c) Cavitation is characteristically seen in reactivation of tuberculosis
d) In miliary tuberculosis, grey-white nodules are scattered throughout the lung
parenchyma and the pleura
e) Bronchial stenosis is a complication

A

The Ghon focus is characteristic of post-primary (secondary) tuberculosis (Initial focus of 1 infection = Ghon complex)

180
Q

A patient with known coal worker’s pneumoconiosis has a CXR. They have
increased risk of all of the following conditions WITH THE EXCEPTION of:
a) Tuberculosis
b) Bronchogenic carcinoma
c) Progressive massive fibrosis
d) Chronic bronchitis
e) Emphysema

A

b) Bronchogenic carcinoma

181
Q

A patient with known asbestos exposure has an abnormal CXR. Which of the
following conditions IS NOT asbestos related?
a) Pleural effusion
b) Non calcified pleural plaques
c) Laryngeal carcinoma
d) Bronchogenic carcinoma
e) Pulmonary microlithiasis

A

Pulmonary microlithiasis

182
Q

Which of the following statements concerning asbestos exposure IS LEAST
correct?
a) There is an increased incidence of carcinoma in families of asbestos workers
b) Histologically asbestosis is characterised by a diffuse basal pulmonary fibrosis with visible asbestos fibres encased in an ironcontaining proteinaceous coating
c) The pulmonary fibrosis begins around the respiratory bronchioles and alveolar ducts.
d) Macroscopic nodule formation may occur in patients with rheumatoid disease
e) While the risk is markedly increased the overall lifetime incidence of mesothelioma with heavy asbestos exposure remains low – in the order of 1 in 1000.

A

While the risk is markedly increased the overall lifetime incidence of mesothelioma with heavy asbestos exposure remains low – in the order of 1 in 1000. (Lifetime risk with high exposure ~10% i.e. 1 in 10)

.(Asbestos bodies = fibres
coated with iron-containing proteinaceous material)
caplan - RA nodules

183
Q

In pure emphysema VS pure chronic bronchitis? BRob p459
a) Emphysema occurs younger
b) Chronic bronchitis more likely to have cardiomegaly
c) Elastic recoil is preserved in emphysema
d) bronchitis earlier and more short of breath
e) cor pulmonal more common in emphysema

A

Chronic bronchitis more likely to have cardiomegaly (Emphysema small heart vs Bronchitis large heart)

184
Q

Desquamative interstitial pneumonitis ? Rob p468
a) Hamman-Rich syndrome
b) Desquamative lymphocytic interstitial pneumonitis
c) Chronic interstitial pneumonitis
d) Treatable condition and can progress to UIP
e) Chronic form of UIP

A

Treatable condition and can progress to UIP

185
Q

Young girl with moderate to several sarcoid presents with sudden death – cause?
(Normal Calcium Score) Rob p470
a) Renal failure
b) Pulmonary HT
c) Ruptured berry aneurysm
d) Hypertensive cerebral haemorrhage
e) Cardiac sarcoid

A

Cardiac sarcoid

186
Q

In Goodpatures syndrome ? Rob p473
a) Lung involvement occurs later than renal
b) Lung involvement characterized by necrotizing haemorrhagic pneumonitis
c) may avert pulmonary disease with renal treatment
d) minimal, same type (pulm/renal disease)
e) severe pulmonary disease secondary to vasculitis
f) pulmonary disease secondary to granulomatosis

A

Lung involvement characterized by necrotizing haemorrhagic pneumonitis true

A condition characterized by the simultaneous appearance of 1. Proliferative
gomerulonephritis 2. Necrotizing haemorrhagic interstitial pneumonitis
Characteristic findings of focal proliferative glomerulonephritis in early cases, or rapid
progressive/crescentic GN (RPGN) in rapidly progressive cases

187
Q

In pulmonary emboli – which is TRUE? Rob p475
a) Haemorrhage implies infarction
b) Always infarct as vessels are end-arteries
c) With haemorrhage underlying parenchyma is preserved

A

With haemorrhage underlying parenchyma is preserved

Smaller emboli can travel out into the more peripheral vessels, where they may or may
not cause infarction.
In patients with adequate cardiovascular function, the bronchial artery supply can often sustain the lung parenchyma despite obstruction to the pulmonary arterial system.
Under these circumstances, hemorrhages may occur, but there is no infarction of the
underlying lung parenchyma.
Only about 10% of emboli actually cause infarction.
Although the underlying pulmonary architecture may be obscured by the suffusion of blood, hemorrhages are distinguished by the preservation of the pulmonary alveolar architecture; in such cases, resorption of the blood permits reconstitution of the preexisting architecture
Pulmonary embolism usually causes infarction only when the circulation is already
inadequate, as in patients with heart or lung disease.
For this reason, pulmonary infarcts tend to be uncommon in the young.
About ¾’s of all infarcts affect the lower lobes, and in more than ½ multiple lesions
occur

188
Q

Small cell carcinoma, which is best?
a) Also known as oat cell
b) Neuroendocrine granules and most likely to have paraneoplastic syndrome
c) Treatment is with lobectomy and chemotherapy
d) Ususally peripheral but has hilar nodes
e) Not associated with smoking

A

Neuroendocrine granules and most likely to have paraneoplastic syndrome

This highly malignant tumor has a distinctive cell type. The epithelial cells are generally
small, have little cytoplasm, and are round or oval and, occasionally, lymphocyte like
(although they are about twice the size of a lymphocyte).
This is the classic oat cell, which is a specific subtype. (not all Small cell are oat cell!!)
true Other small cell carcinomas have
spindle-shaped polygonal cells
and may be thus classified (spindle or polygonal small cell carcinoma). The cells grow in clusters that exhibit neither glandular nor squamous organization.
Electron microscopic studies show dense-core neurosecretory granules in some of these tumor cells. The granules are similar to those found in the neuroendocrine argentaffin (Kulchitsky) cells present along the bronchial epithelium, particularly in the fetus and neonate.
3-10% of lung cancers develop a paraneoplastic syndrome

189
Q

67 year old man, 4cm small cell lung Ca, hilar lymph node, no mets – query
survival without treatment?
a) 40% 5 years
b) 20% 5 years
c) 50% 5 years
d) 50% 6 months
e) 6-18 weeks

A

6-18 weeks
untreated survival of small cell is 6 - 17 weeks
Even with treatment mean survival after diagnosis is about 1 year

190
Q

Laryngeal cancer ? Rob p507
a) Associated with alcohol, smoking and irradiation
b) Polyps and nodules are associated with increase risk of cancer
c) Usually arises from false cords
d) Polyps and nodules usually involve false cords

A

a) Associated with alcohol, smoking and irradiation true

b) Polyps and nodules are associated with increase risk of cancer (virtually never give rise to
cancers)
c) Usually arises from false cords (true vocal cords)
d) Polyps and nodules usually involve false cords (true vocal cords)

191
Q

Asbestos plaques – least associated with ?
a) asbestos bodies
b) pleural effusions
c) anterior/posterolateral site
d) diaphragm

A

asbestos bodies

192
Q

Pleural fibroma. Unusual finding
a) Slightly mobile and on pedicle
b) 1-2 cm in size
c) 8-10 cm in size
d) associated with pulmonary effusion
e) cystic formation with the mass

A

no effusion

50% pedicle
variable size

193
Q

Alpha feto-protein is least likely to be associated with:
a) colonic ca
b) lung ca
c) pancreatic ca
d) smoking
e) nonseminiferous tumour of testes
f) GIST
g) Uncomplicated cirrhosis

A

gist

AFP : glycoprotein synthesised early in foetal life by the yolk sac, foetal liver and foetal
GIT.
Abnormal plasma elevations are encountered in adults with cancer arising principally in
the liver and germ cells of the testis.
Less regularly in CA of colon, lung and pancreas
Non-neoplastic : cirrhosis, toxic liver injury, hepatitis and pregnancy (especially when
there is foetal distress or death)

194
Q

Bronchial carcinoid ? Rob p504
a) 0.1-0.05 % lung tumors
b) 5th –6th decade of life
c) 2-3:1 male:female
d) 10% atypical (of which will get carcinoid recurrence in 50%)
e) histology keratin pearls

A

10% atypical (of which will get carcinoid recurrence in 50%)

Bronchial carcinoids represent 1 to 5% of all lung tumors
Most patients with carcinoid tumors are younger than 40 years of age
incidence is equal for both sexes
The reported 5- to 10-year survival rates for typical carcinoids are 50 to 95%.
A minority (10%) of tumors show cytologic atypia, necrosis, and aggressive
behavior (50% recurrence or metastasis after 2 years) and are designated atypical
carcinoids. (note: Adelaide notes say 25%!!)
Histologically the tumor is composed of nests, cords, and masses of cells separated by a
delicate fibrous stroma. In common with the lesions of the gastrointestinal tract, the
individual cells are quite regular and have uniform round nuclei and infrequent mitoses

195
Q

a1-ATdefic – incorrect
a) A Recessive
b) Von Meyenberg complexes
c) End stage get cirrhosis + cholestasis

A

Von Meyenberg complexes

alpha1 -Antitrypsin deficiency is an autosomal recessive disorder marked by abnormally
low serum levels of this protease inhibitor (Pi)
However, homozygotes for the PiZZ protein have circulating alpha1 -antitrypsin levels
that are only 10% of normal levels
Neonatal hepatitis with cholestatic jaundice appears in 10% to 20% of newborns with
the deficiency. In adolescence, presenting symptoms may be related to hepatitis or
cirrhosis. Attacks of hepatitis may subside with apparent complete recovery, or they may become chronic and lead progressively to cirrhosis.
Hepatocellular carcinoma develops in 2% to 3% of PiZZ adults, usually but not always
in the setting of cirrhosis

196
Q

Most diagnostic of asbestosis
a) Asbestos bodies
b) Interstitial fibrosis

A

asbestos bodies

Asbestos bodies appear as golden brown, fusiform or beaded rods with a translucent
center and consist of asbestos fibers coated with an iron-containing proteinaceous
material
There are two distinct geometric forms of asbestos: serpentine (curly and flexible fibers)
and amphibole (straight, stiff, and brittle fibers).
The serpentine chrysotile chemical form accounts for most of the asbestos used in
industry.
Amphiboles include crocidolite, amosite, tremolite, anthophyllite, and actinolyte.

amphiboles, even though less prevalent, are more pathogenic particularly with respect to
induction of malignant pleural tumors (mesotheliomas).
Some studies of mesotheliomas have shown the link is almost always to amphibole
exposure.
The relatively few cases of mesotheliomas arising in chrysotile workers are in all
likelihood due to contamination of chrysotile with the amphibole tremolite.
The greater pathogenicity of straight and stiff amphiboles is apparently related to their
aerodynamic properties and solubility.
Chrysotiles, with their more flexible, curled structure, are likely to become impacted in
the upper respiratory passages and removed by the mucociliary elevator. Furthermore,
once trapped in the lungs, chrysotiles are gradually leached from the tissues because they
are more soluble than amphiboles.
In contrast, the straight, stiff amphiboles may align themselves in the airstream and thus
be delivered deeper into the lungs, where they can penetrate epithelial cells and reach the
interstitium.
The length of amphibole fibers also plays a role in pathogenicity, those longer than 8
mm and thinner than 0.5 mm being more injurious than shorter, thicker ones.
Nevertheless, both amphiboles and serpentines are fibrogenic, and increasing doses are
associated with a higher incidence of all asbestos-relq

197
Q

Characteristic features of Rh nodule

A

Central fibrinoid necrosis surrounded by palisading histiocytes
Microscopically, they have a central zone of fibrinoid necrosis surrounded by a
prominent rim of epithelioid histiocytes and numerous lymphocytes and plasma cells

198
Q

Immune reaction to tuberculosis is an example of
a) Antibody mediated immunity
b) Delayed hypersensitivity reaction
c) Immune complex mediated
d) Immunosuppression

A

Delayed hypersensitivity reaction

199
Q

Emphysema is defined as:
a) dilation of airspace distal to terminal bronchioles with destruction of walls
b) destruction of airspaces proximal to distal bronchioles with no destruction of walls
c) dilated bronchioles
d) subpleural fibrosis

A

dilation of airspace distal to terminal bronchioles with destruction of walls

200
Q

Regarding pulmonary emboli and infarcts, which of the following is MOST
CORRECT:
a) Majority (>50%) result in white infarcts because it is an end circulation
b) 75% are in the lower lobes
c) Pulmonary haemorrhage suggest infarction
d) The presence of viable alveoli pneumocytes/endothelium in an area of haemorrhage is
against a diagnosis of PE
e) >50% of infarcts are solitary

A

75% are in the lower lobes

Only 15% of emboli actually cause infarction, due to dual blood supply (pulmonary
& bronchial)
75% small emboli in lower lobes & asymptomatic
pulmonary haemorrhage may occur without infarction
50% are multiple

201
Q

Patient with suspected psittacosis for HRCT: (GC)
a) Inflammation- histiocytes /lymphocytes confined to alveolar walls
b) Neutrophilic exudate within alveoli
c) Grey / white slough on bronchial walls
d) Mucous plugging and areas of atelectasis

A

Inflammation- histiocytes /lymphocytes confined to alveolar walls

Psittacosis is an infection caused by the obligatory intracellular bacterium Chlamydia
psittaci; assocd with birds & poultry.
Predominant manifestation is respiratory tract infection with constitiutional symptoms.
May also have CNS, skin, spleen, renal, haematologic, joint involvement…
The most common imaging finding is unilateral, LL dense infiltrate/consolidation. May
present in a bilateral, nodular, miliary, or interstitial pattern.
Histo findings may include tracheobronchitis and interstitial pneumonitis with air-space
involvement and predominant mononuclear cell infiltration. Findings may also include
macrophages that contain cytoplasmic inclusion bodies (ie, Levinthal-Coles-Lillie
[LCL] bodies), focal necrosis of hepatocytes along with Kupffer cell hyperplasia in the
liver, and hepatic noncaseating granulomata.

202
Q

In patient with multiple positive blood cultures for Candida, which would be atypical:
a) Empyema
b) Osteomyelitis
c) Tricuspid valve lesions
d) Splenic lesions
e) Kidney lesions

A

a) Empyema F - rare… only a few case reports of candida empyema (eg. Indian J Pathol
Microbiol 2008). A Taiwanese study (Chest 2000) looked at 67 patients with fungal
empyema (of which 18 were immunocompromised); found that Candida was the most
commonly encountered species (64%).

b) Osteomyelitis T - originates either exogenously (direct inoculation) or endogenously
(candidaemia or disseminated infection). Mostly due to haematogenous seeding, involves
the vertebral discs and often progresses to discitis with contiguous extension to the vetebral
body. Other bones include wrist, femur, scapula, proximal humerus. Can also cause
arthritis, costochondritis, myositis.
c) Tricuspid valve lesions T ? - however, endocarditis most commonly involves the aortic and
mitral valves. Infrequently involves tricuspid valve and rarely the pulmonary valve (Scand J
Infect Dis 2007).
d) Splenic lesions T - splenic abscess and hypersplenism - both are manifestations of
disseminated candidiasis and are usually simultaneously associated with liver involvement.
e) Kidney lesions T - manifestation of candidaemia or disseminated candidiasis.

203
Q

ARDS takes a long time to resolve because: (GC)
a) Usually associated with bacterial superinfection
b) Always goes on to fibrosis
c) Abnormality is ude to endothelial damage, as well as increased capillary permeability
d) Takes a long time for type II pneumocytes to recover

A

Always goes on to fibrosis T - the acute (exudative) phase usually resolves completely. Less commonly, residual pulmonary fibrosis occurs, in which the alveolar spaces are filled with
mesenchymal cells and new blood vessels. This process seems to be facilitated by IL-1.

Exudative stage:
endothelial - increased vascular permeability (oedema, inflammation, haemorrhage), leads to fibrin exudation and formation of hyaline membranes.
epithelial - damage to type II pneumocytes, resulting in widespread reduction in surfactant production (with subsequent decreased lung compliance and alveolar collapse).
Proliferative (organising) stage: type II pneumocytes attempt to regenerate the alveolar lining, organisation of fibrin exudate.
Fibrotic stage: variable scarring and cyst formation.

204
Q

Definition of Thymoma: (GC)
a) Any benign thymic tumour
b) Benign tumour of epithelial cells
c) Any tumour of thymic epithelial cells
d) Any tumour of thymus
e) Benign tumours of epithelial and mesenchymal origin

A

c) Any tumour of thymic epithelial cells = the term “thymoma” is restricted to tumours in
which epithelial cells constitute the neoplastic element [Robbins]. Divided into “invasive”
and “non invasive” (cf. benign and malignant, as in Robbins) because invasive thymoma
may lack the histologic features of malignancy [RG].

205
Q

Young female with tufts of capillary formation with dilated thin walled arteries implies (JS)
a) Primary pulmonary hypertension
b) Goodpasture’s
c) SLE
d) Recurrent thromboemboli

A

Primary pulmonary hypertension -T - plexogenic pulmonary arteriopathy (see Robbins) is
seen in PPH or congenital heart disease with left to right shunts - tuft of capillary
formations that spans the lumens of dilated thin-walled arteries

206
Q

In pure emphysema versus pure chronic bronchitis (JS)
a) Emphysema occurs younger
b) Chronic bronchitis more likely to have cardiomegaly
c) Elastic recoil is preserved in emphysema
d) Bronchitis earlier and more short of breath
e) Cor pulmonal more common in emphysema

A

Chronic bronchitis more likely to have cardiomegaly T - long standing bronchitis is
associated with cor pulmonale and heart failure

Chronic bronchitis = defined clinically: any patient who has persistent cought with
sputum production for at least 3 months in at least 2 consecutive years.
Emphysema = abnormal permanent enlargement of the airspaces distal to the terminal
bronchiole, accompanied by destruction of their walls, and without obvious fibrosis.

a) false bronchitis 40 emphysema 50-75
c) Elastic recoil is preserved in emphysema F - normal in bronchitis, low in emphysema
d) Bronchitis earlier and more short of breath F - Emphysema is associated with more severe SOB
e) Cor pulmonal more common in emphysema F - cor pulmonale more common in bronchitis

207
Q

) Coal workers pneumoconiosis, which is not true: (GC)
a) 2-5 x risk of lung cancer independent of smoking
b) <10% go on to Progressive massive fibrosis
c) Characterised by coal macules and coal nodules
d) 4. PMF tends to progress even in the absence of further exposure
e) 5. Increased incidence of TB

A

a) 2-5 x risk of lung cancer independent of smoking F - once smoking-related risk is taken
into account, there is no increased frequency of bronchogenic carcinoma (distinguishes
from silicosis and asbestosis).

b) <10% go on to Progressive massive fibrosis T - fewer than 10% of cases of simple CWP
progress to PMF, a generic term that applies to a confluent fibrosing reaction in the lung
(complication of any of the fibrogenic pneumoconioses - CWP, silicosis, asbestosis).
Generally takes many years to develop. PMF in CWP is a fibrotic mass of haphazardly
arranged collagen with numerous pigment-laden macrophages and with abundant free
pigment, +/- necrosis, cholesterol clefts, chronic inflam infiltrate. Hx differs from silicosis of PMF: central hyalinised collagen surrounded by a rim of macrophages, focal necrosis common +/- granulomatous inflammation.
c) Characterised by coal macules and coal nodules T - macules are dust laden macropahges, 1-2mm. Nodules are slightly larger (still <5mm), and have additional delicate network of collagen fibres. Lesions are scattered throughout, but predominantly in upper lobes and upper zones of lower lobes. Radiographically indistinguishable from silicosis.
d) 4. PMF tends to progress even in the absence of further exposure T
e) 5. Increased incidence of TB T - as in silicosis.

208
Q

Which does not cause Bronchiolitis Obliterans: (GC)
a) Tuberous sclerosis
b) GVHD
c) Collagen vascular disorders
d) Viral infections
e) Bleomycin

A

a) Tuberous sclerosis F - 1-4% have lung involvement - LAM.

b) GVHD T - chronic, ie. in bone marrow transplant. Also chronic rejection (lung or heartlung transplants).
c) Collagen vascular disorders T - RA, scleroderma, SLE.
d) Viral infections T - also mycoplasma in kids. Swyer-James syndrome = variant of
constrictive bronchiolitis with acute obliterative bronchiolitis, bronchiectasis, distal airspace destruction, developing in 7-30 months.
e) Bleomycin T - other drugs - gold salts, MTX, cyclophosphamide, penicillamine.

Definition: inflammation of bronchioles leading to (sometimes reversible) obstruction of bronchiolar lumen.
Path: submucosal and peribronchiolar fibrosis = irreversible fibrosis of small airway
walls with narrowing/obliteration of airway lumina by granulation tissue of immature
fibroblastic plugs (Masson bodies).
Other causes:
inhalation of toxic fumes
cystic fibrosis - as a complication of repeated episodes of pulm infection
idiopathic in immunocompetent patients

209
Q

In Goodpasture’s syndrome (JS)
a) Lung involvement precedes renal involvement
b) Lung involvement occurs later than renal
c) Lung involvement characterised by necrotizing haemorrhagic pneumoniti
d) Usually mild

A

a) Lung involvement precedes renal involvement T (??) - glomerulonephritis and pneumonitis occur simultaneously but it begins clinically with respiratory symptoms (haemoptysis) and focal pulmonary consolidation

c) Lung involvement characterised by necrotizing haemorrhagic pneumonitis T - acute focal necrosis of alveoli, intra-alveolar haemorrhage, fibrous thickening of septa etc = necrotising haemorrhagic interstitial pneumonitis

210
Q

What is most controversial subgroup in Hodgkin’s lymphoma classification: (GC)
a) Nodular sclerosis
b) Lymphocytic predominant
c) Lymphocytic rich
d) Lymphocyte depleted
e) Mixed cellularity

A

Lymphocyte depleted - rare and somewhat controversial. Most cases prove to be large cell lymphoma. Older male patients with disseminated disease, HIV patients, developing countries.

a) Nodular sclerosis - most common, young adults M=F. Hx: lacunar variant of RS cell,
collagen bands with formation of nodules, CD30/CD15+. Excellent prognosis.
b) Lymphocytic predominant - young adult males. Hx: L+H (popcorn) variant of RS cell, panB-cell markers +. Excellent prognosis; 3-5% transform into DLBCL.
c) Lymphocytic rich - rare.
e) Mixed cellularity - older men. Hx: lots of RS cells and heterogeneous infiltrate. Usually disseminated disease + B symptoms.

211
Q

Sarcoid, which is true: (GC)
a) Relapsing/ remitting course
b) Complete recovery in 60-80%
c) Residual pulmonary/ visual impairment in 30%
d) Parenchymal involvement only has a better prognosis than nodes + lung changes
e) Eyes are involved in 40% of cases

A

complete recovery in 60-80% true

a) Relapsing/ remitting course F - majority recover, 10% develop progressive pulmonary fibrosis and cor pulmonale.
c) Residual pulmonary/ visual impairment in 30% F - 20%
d) Parenchymal involvement only has a better prognosis than nodes + lung changes F - stage 3 versus stage 2 disease (Silzbach classification on CXR: Stage 0 - normal, 1 - adenopathy, 2 - LN + lung, 3 - lung only, 4 - pulmonary fibrosis).
e) Eyes are involved in 40% of cases F - 25%, uveitis. Mikulicz syndrome = uveitis + bilateral parotid gland involvement. Panda sign on Ga-67 scan = bilateral lacrimal and parotid
gland uptake.

212
Q

an does not want surgery, hilar mass, oncologist wants to know if small cell, does CT most likely bit of hx that would help: (GC)
a) CXR 1 year ago, hilar mass, lost to follow up
b) Normal CXR 9 months ago
c) Hyponatraemia
d) Peripheral location
e) Mediastinal widening

A

Hyponatraemia T - secondary to SIADH; main features of the syndrome consist of hyponatremia and hypotonicity (< 280mOsm/kg), absence of fluid volume depletion, inappropriate urinary osmolality (> 100 mOsm/kg), increased urinary sodium excretion
(>40mmol/L) while on normal salt and water intake, and absence of thyroid, adrenal, pituitary or renal dysfunction. SIADH results from 3 factors: 1. inappropriate stimulation from pulmonary pathology (bacterial pneumonia, TB, lung abscess) or drugs (eg.
cytotoxics); 2. uncontrolled secretions from virtually any CNS disorder (including trauma, surgery); 3. ectopic ADH elaboration by tumours, particularly small cell (oat cell) lung carcinoma, duodenum and pancreatic cancers, olfactory neuroblastoma and lymphomas.

213
Q

Hodgkin’s disease, 50yo with chemo, develops lung parenchymal mass:
a) CMV
b) HD recurrence
c) Primary TB
d) Secondary TB

A

d) Secondary TB

214
Q

Community acquired pneumonia – least correct: (GC)
a) Viral
b) Mycoplasma + Chlamydia
c) Px with symptoms of dyspnoea, haemoptysis, effusion
d) Bronchial ulceration means bacterial superimposed infection

A

Bronchial ulceration means bacterial superimposed infection F - causes include HSV, radiation, drugs, TB, malignancy, Wegener’s, post transplant lymphoproliferative disease.

215
Q

The pathogenic agent in Legionnaires disease is L. pneumophilia. This agent is best
described as: (GC)
a) Gram negative bacteria
b) Spore forming coccus
c) Helminth
d) Protozoa
e) Rickettsia

A

Gram negative bacteria T - 1-2 um, aerobic, weakly acid-fast, silver impregnation stain.

Legionnaires’ disease: epidemic and sporadic forms of pneumonia caused by L. pneumophilia.
Pontiac fever: a related self-limiting URTI (without oneumonic sypmtoms).
L. pneumophilia fluorishes in artificial aquatic environment (water cooling towers, potable
water supplies); transmission is by inhalation of aerosolized organisms or aspiration of contaminated drinking water. Predisposed: mid-elderly, immunosuppressed, alcoholics, COPD, diabetes, cardiovascular disease, CRF, transplant recipients.
Fatality rate of 30-50% in immunosuppressed pts, 6% in healthy pts.

216
Q

35yo outpatient HRCT request ‘Late onset asthma’ worsening dyspnoea, initially episodic and now chronic. Biopsy shows granuloma. Which of the following is most likely with this history:
a) silicosis
b) TB
c) HSP
d) sarcoid
e) histoplasmosis

A

Hypersensitivity pneumonitis - immunologically mediated response to an extrinsic antigen
that involves both immune-complex (type III) and delayed-type (IV) hypersensitivity
reactions. Clinically: acute reaction (large loadfever, cough, SOB 4-8hrs post exposure),
subacute (acute episodes, background of progressive respiratory deterioration - chronic
cough and SOB), or chronic disease. Non caseating granulomas in >2/3 of cases.

a) Silicosis - occupational exposure. Hx: silicotic nodule of whorled collagen fibres, silica
particles are weakly birefringent.
b) TB - reactivation (secondary) TB either from reactivation (due to weakened host
resistance) or exogenous reinfection (in areas of high contagion). <5% of those with
primary disease develop secondary TB. May be asymptomatic if localised; or insidious
onset of low-grade fever, night sweats, sputum production, haemoptysis, pleuritic pain;
extrapulmonary manifestations.
Sarcoid - multisystemic disease of unknown aetiology; non caseating granulomas. 50% may
be asymptomatic; 2/3 of symptomatic cases present with gradual onset of SOB, dry cough,
vague substernal discomfort, or constitutional symptoms (fever, wt loss, anorexia, fatigue,
night sweats).
e) Histoplasmosis - H.capsulatum, endemic in Ohio and central Mississippi region. 3 clinical
forms: acute (primary), chronic (cavitary), disseminated miliary (in immunocompromised,
infants). Symptoms resemble a flu-like syndrome, most often self-limited. Cavitary disease
occurs in vulnerable host (upper lobe, similar to secondary TB) - symptoms of cough,
haemoptysis, SOB and chest pain may occur. Hx: granulomas, strikingly similar to TB,
differentiation requires identification of yeast forms (PAS or silver stains).

217
Q

Concerning centrilobular emphysema, which of the following is most correct: (JS)
a) The acini are uniformly enlarged from the level of terminal bronchioles to terminal alveoli.
b) Proximal part of acini is enlarged , relative or complete sparing of distal acini.
c) Proximal part of the acinus normal or near normal with dominant involvement of distal
portion.
d) Whole acini destroyed leaving irregular spaces > 1cm in diameter.
e) Acini uniformly involved but disease effects central zone of secondary pulmonary lobule.

A

Proximal part of acini is enlarged , relative or complete sparing of distal acini. T

a) the acini are uniformly enlarged from the level of terminal bronchioles to terminal alveoli. F - Panacinar/panlobular
c) Proximal part of the acinus normal or near normal with dominant involvement of distal
portion. F - Paraseptal/distal acinar
d) Whole acini destroyed leaving irregular spaces > 1cm in diameter. F - can occur with both
panacinar and centrilobular - if severe
e) Acini uniformly involved but disease effects central zone of secondary pulmonary lobule. F

218
Q

Patient with markedly thickened pleura encasing one lung. Which of the following statements is most correct? (GC)

a) Only approximately 10-20% with patients with mesothelioma will have associated lower
lobe fibrosis
b) Mesothelioma and metastatic adenocarcinoma have markedly different appearances on light microscopy
c) Presence of asbestos bodies on pleural biopsy suggest change is more likely reactive /
fibrosis
d) Granuloma in pleural tissue suggests chronic irritation rather than neoplastic infective
etiology
e) Apical lung lesion suggests it is most likely infective in nature

A

a) Only approximately 10-20% with patients with mesothelioma will have associated lower
lobe fibrosis T - 90% of malig. mesotheliomas are related to asbestos but only 10% have
asbestosis. As opposed to peritoneal mesothelioma, where 50% have asbestosis. [Robbins]

b) Mesothelioma and metastatic adenocarcinoma have markedly different appearances on light
microscopy F - differentiated by antigenic keratin positivity, CEA stain negative, presence
of long microvilli on EM.
c) Presence of asbestos bodies on pleural biopsy suggest change is more likely reactive /
fibrosis F - diffuse pleural thickening is rare, involves visceral pleura (cf. pleural plaques -
parietal). Neither of theses entities (or mesothelioma) contain asbestos bodies on histology.
d) Granuloma in pleural tissue suggests chronic irritation rather than neoplastic infective
etiology F
e) Apical lung lesion suggests it is most likely infective in nature

219
Q

FNA of a 52 yo hilar lung mass shown abdundant cells with eosinophilic cystoplasm
and intercellular bridges, together with minor necrotic tissue, most characteristic of: (TW)
a) Bronchial carcinoid
b) Nodal involvement with Sarcoidosis
c) Bronchogenic cysts -
d) SCC
e) Small cell carcinoma of the lung

A

SCC - T - production of keratin and intercellular bridges in well-differentiated forms. Pathoutlines: Pink cytoplasm with distinct cell borders and intercellular bridges characteristic for squamous cell carcinoma

a) Bronchial carcinoid - F - nests, cords, and massess of cells separated by a delicate fibrous stroma. Regular cells, uniform round nuclei.
b) Nodal involvement with Sarcoidosis - F - noncaseating granulomas, each composed of an aggregate of tightly clustered epithelioid cells, often with Langerhans or foreign body-type giant cells. Laminated concretions (Schaumann bodies), stellate inclusions (asteroid bodies).
c) Bronchogenic cysts - F - lined by bronchial type epitleium and are usually filled with
mucinous secretions or with air.
e) Small cell carcinoma of the lung - F - small epithelial cells, little cytoplasm, round or oval
and, occasionally, lymphocyte like (but about 2x size of lymphocyte) - “classic oat cell”.
KCC type 3 (Kulchitsky cell carcinoma, where type 1 is “classic carcinoid”, type 2 is “atypical carcinoid).

220
Q

62 yo smokers CXR is introduced at a clinical meeting as showing ‘occupational
asbestos exposure with complications’. Which of the following is least likely to be an
associated finding: (TW)
a) Focal non calcified pleural thickening
b) Pleural effusion
c) Mediastinal lymphadenopathy
d) Lobar collapse
e) Progressive massive fibrosis

A

Progressive massive fibrosis - F - eMed: unlike other pneumoconioses, progressive massive
fibrosis tend not to occur. Also says lymphadenopathy tend not to occur too.

221
Q

Recent decrease in Kaposi sarcoma. Main reason for this: (GC)
a) Decreased incidence of EBV
b) Increased proportion of heterosexuals getting HIV
c) New retroviral treatment
d) ?Community education
e) Prophylactic treatment

A

New retroviral treatment T - AIDS-associated (epidemic) KS was originally found in 1/3 of
AIDS patients, particularly male homosexuals. However, with current regimens of intensive
antiretrovrial therapy, the incidence is now <1% (although still the most prevalent
malignancy in AIDS pts in the US).

Four forms of KS:
Classic KS - typically older men of Eastern European or Mediterranean descent, red-purple skin plaques/nodules esp. on the distal extremities, tumours tend to remain localised to skin and subcutaneous tissue.

Endemic KS (aka lymphadenopathic or African) - children of subsaharan Africa (same
geographic distribution as Burkitt’s), present with lymphadenopathy, occ. involves viscera,
extremely aggressive.

Transplant- assocd KS - solid organ transplant recipients on long-term immunosuppression,
aggressive with nodal, mucosal, and visceral involvement.

AIDS-assocd (epidemic) KS

222
Q

Primary TB: (TW)
a) Only 1/10, 000 primary infection is clinically significant
b) 80% of patients asymptomatic
c) Progressive primary disease most commonly causes middle or lower lobe consolidation

A

80% of patients asymptomatic - T - natural Hx of TB in a proxpective study of new
tuberculin converters (1932-1946). Fever most common symptoms occurring in 70%,
however when excluding fever, only 25% had additional symptoms. Dahnert says 91%
asymptomatic.

c) Progressive primary disease most commonly causes middle or lower lobe consolidation - F - extensive consolidation and cavitation develop, either at the site of the initial pulmonary parenchymal focus of infection or in the apical and posterior segments of the upper lobes. Miliary TB.

223
Q

Which of the following is least likely associated with mesothelioma (TW)
a) Exposure to chrysotile fibres
b) Exposure to amphibole fibres
c) Latency period between exposure and tumour
d) Pulmonary fibrosis

A

a) Exposure to chrysotile fibres - F - serpentine fiber is less carcinogenic than amphibole.
Chrysotile fibers dissolve relatively quickly (UTD). Robbins says - both amphiboles and
serpentines are fibrogenic, and increasing doses are associated with a higher incidence of asbestos-related disease except that only amphibole exposure correlates with
mesothelioma.

b) Exposure to amphibole fibres - T - straight, rod-like fiber (crocidolite, amosite, tremolite)
are more carcinogenic.
c) Latency period between exposure and tumour - T - there is a long latency of approximately 30 to 40y from the time of asbestos exposure to the development of mesothelioma.
d) Pulmonary fibrosis - T - asbestos - fibrosis (asbestosis) - mesothelioma. Radiation therapy can have fibrosis changes and mesothelioma (also with long latent period). 20% of pts with pleural mesothelioma have radiographic signs of asbestosis (probably higher on CT).

Asbestos fibers are divided into two categories based on their shape. Serpentine fibers, of which chrysotile is the only significant commercial variety, are long, curly strands, where as amphibole fibers (crocidolite, amosite, tremolite, and others) are long, straight, rod-like structures.

Mesothelioma arises in relation to the parietal pleura. Classified pathologically as epithelial 50%, sarcomatous 25%, or mixed 25% (TI)
Staging:
T1 - tumor limited to ipsilateral parietal pleura or assoc with scattered foci of visceral pleural involvement.
T2 Ipsilateral PP tumor with involvment of diaphragm, or confluent involvement of the visceral pleura.
T3 Ipsilateral PP tumor with limited chest wall mediastinal, or pericardial invasion
T4 Extensive chest wall, subdiaphragmatic, mediastinal or pericardial invasion.
N and M stage same for lung cancer.

224
Q

Hodgkin’s which is incorrect (TW, GC)
a) Reid Sternberg pathognomonic
b) Increased HIV
c) Lymphocyte rich is common
d) Nodular sclerosing is a type

A

Lymphocyte rich is common - F - Different subtypes, can be lymphocyte rich, but also
mixed cellularity (small lymphocytes, eosinophils, plasma cells, macrophages and RS cells), and also lymphocyte deplete subtype. Lymphocyte rich and lymphocyte deplete subtypes are rare.

a) Reid Sternberg pathognomonic - T- considered the true neoplastic element in HD (which are considered to be B-cells) with owl-eye nuclei.
b) Increased HIV - T - HIV infected individuals have a significantly increased risk of
developing HD. HIV related HD is characterised by the preponderance of aggressive histological subtypes, advanced stage at presentation, and highly malignant clinical course. Unlike HD in the general population, the majority of HIV related HD cases are pathogenetically linked to EBV.
Nodular sclerosing is a type - T - Most common form of HD - presence of a particular variant of RS cell (lacunar cell), and presence of collagen bands that divide the lymphoid tissue into circumscribed nodules. NS type is the only form of HD that is more common in women and strikes adolescents or young adults.

225
Q

Cryptococcus in immunocompromised (TW)
a) Basal exudate
b) Single coin lesion
c) Military lung
d) End arteritis

A

Military lung - T - In patients with AIDS, a diffuse interstitial pattern, variously described as reticular or nodular, resembling P. jiroveci pneumonia, may be seen. A miliary pattern may occur, as may single or multiple nodules, occasionally with cavitation. Lymph node enlargement is uncommon, as is pleural effusion. In healthy patients, cryptococcal infection usually manifests as one or more peripheral, circumscribed nodules, usually without cavitation. Less commonly, air-space consolidation is seen

226
Q

RE: ABPA, incorrect (TW, GC)
a) Gelatinous exudate around lung lesion
b) Chronic haemoptysis
c) Invasion is deep to alveolar basement membrane

A

Invasion is deep to alveolar basement membrane - F – this refers to airway invasive
aspergillosis. In ABPA fungal hyphae without evidence of tissue invasion. Septated hyphae
with acute dichotomous branching may be seen in the mucus-filled bronchial lumen, but
fungi do not invade mucosa.

ABPA is a hypersensitivity reaction ot Aspergillus antigens (usually Aspergillus fumigatus) - IgE type I hypersensitivity and IgG type III hypersensitivity reactions. Dx criteria: asthma, peripheral blood eosinophilia, immediate positive skin test for Aspergillus antigens, increased
serum IgE levels, and pulmonary opacity on chest radiographs.

a) Gelatinous exudate around lung lesion - T - plugging of airways by hyphal masses with
distal mucoid impaction.
b) Chronic haemoptysis - T - asthma, recurrent episodes of bronchial obstruction, fever, malaise, peripheral blood eosinophilia, and at times hemoptysis.

227
Q

Mucor – not typical (TW)
a) Diabetics and immunosuppressed
b) Lots of small nodules in brain
c) Sinus disease
d) Bowel wall thickening

A

Lots of small nodules in brain - ?F - CNS infection typically arises from an adjacent
paranasal sinus infection (typically rhinocerebral). However, there have been cases of isolated CNS zygomycosis described (~30), infection thought to result from seeding of the brain during episode of fungemia (?nodules).

a) Diabetics and immunosuppressed - T - immunocompromised patients / diabetics.
c) Sinus disease - T - most common clinical presentation of zygomycosis is rhino-orbitalcerebral infction. Usually presents as acute sinusitis with fever, nasal congestion, purulent nasal discharge, headache, and sinus pain.
d) Bowel wall thickening - T - Intestinal zygomycosis is manifested by erosive necrotic ulcers with thromboses and gangrene in the GIT from the oesophagus to colon. These lesions may perforate GI wall leading to peritonitis. Death usually caused by shock from hemorrhage of the bowel resulting in peritonitis and bowel infarction.

Zygomycetes = class of fungi that can cause a variety of infections if humans, particularly in
immunocompromised patients and those with diabetes. Zygomycetes are ubiquitous in nature, and can be found on decaying vetetation and in the soil.
Grow rapidly and release large numbers of spores that can become airborne. All humans have ample exposure to these fungi during day-to-day activities. The fact that zygomycosis is a rare human infection reflects the effectiveness of the intact human immune system.
Genera most commonly found in human infections - Rhizopus, Mucor, and Cunninghamella.

228
Q

Wegner’s granulomatosis, what is false: (TW)
a) ANA +ve
b) ANCA +ve
c) Generalised vasculitis
d) Granulomatosis of the upper and lower respiratory tracts

A

a) ANA +ve - F - uncommon, but can be positive. ANA is the characteristic finding in SLE.. Others with positive ANA - scleroderma, MCTD, polymyositis, RA, Sjogrens.

b) ANCA +ve - T - c-ANCA positive - 96% sensitive for generalised disease, 99% specific
c) Generalised vasculitis - T - systemic necrotizing granulomatous destructive angitis.
d) Granulomatosis of the upper and lower respiratory tracts - T - respiratory tract
granulomatous inflammation. Paranasal sinus involvement 91%, nasopharynx 64%.
Pulmonary disease 94%.

229
Q

Immune reaction to TB is an example of: (GC)
a) Antibody mediated immunity
b) Delayed hypersensitivity reaction
c) Immune complex mediated
d) Immunosuppression

A

Delayed hypersensitivity reaction T - type IV immune reaction. Occurs ~3wks after
exposure. Processed mycobacterial antigens reach draining LNs and are presented in an
MHC class II context by dendritic cell macrophages to CD4+ T cells. Helper T cells release IFN which activates macrophages - they then (a) secrete TNF - recruitment and activation of monocytes, differentiate into “epithelioid histiocytes” (granulomatous response), (b) induce NO formation - generates free radicals, and (c) generate reactive O2 species (antibacterial).

230
Q

Regarding lymphoma: (TW)
a) Lymphocyte deplete is the most common form
b) RS cell is derived from the B cell
c) HD and NHD cannot coexist
d) FNA cannot diagnose HD as it is the architecture of the lymph node that makes the diagnosis

A

RS cell is derived from the B cell - T - neoplastic R/S cells are now considered to be B-cells, typically CD30 / CD15 positive, and CD 20 / EMA / CD45 negative. They are not classified as b-cell lymphomas as they don’t act like them. Molecular studies have established a B-cell origin for RS cells in the vast majority of cases of HL.

a) Lymphocyte deplete is the most common form - F - rare entity. Nodular sclerosis type is
most common.
c) HD and NHD cannot coexist - F - “Composite lymphoma” - term used to denote 2 distinct types of lymphoma occurring within a single organ or tissue. Synchronous occurrence of 2
types of NHLs is more common than an NHL and HL (But occurs).
d) FNA cannot diagnose HD as it is the architecture of the lymph node that makes the diagnosis - F - can Dx with FNA (RS cells). Lymph node excision Bx is used for determining type which is based on morphologic and immunophenotypic features.

Two distinct groups of diseases within the HL”: “Classical” HL which includes the nodular
sclerosis HL, mixed cellularity HL, lymphocyte rich classical HL, and lymphocyte depelated
subtypes. Other group is nodular lymphocyte predominant HL (REAL / WHO classification).
The distinction between these groups is based on the differences in the phenotype and morphology of the RS cells.
RS cells of classical HL fail to exhibit phenotypes typical of any normal cell. T cell, and B cell markers are usually negative, granulocytes may express CD15 (diagnostic marker that is most specific among lymphomas for classic RS cells). Evidence points towards germinal
center B cells as the RS cells.
Nodular sclerosis type is most common form of HL. Mixed cellularity is next most common.

231
Q

Which is true regarding Churg Strauss Disease? (GC)
a) c-ANCA
b) Not associated with eosinophilia
c) Allergic rhinitis and gastroenteritis
d) Lower frequency of cardiac involvement than Wegener’s

A

Allergic rhinitis and gastroenteritis - T - ? hypersensitivity response to inhaled agent.
Allergic rhinitis or asthma (phase 1); periph/tissue eosinophilia (phase 2); systemic small vessel granulomatous vasculitis (phase 3) usually develops within 3yrs of onset of asthma. Eosinophilic tissue infiltration causes eo pneumonia and gastroenteritis. GIT involvement occurs in 20% - ulceration, haemorrhage, perforation, bowel wall thickening. Can also get extravascular “allergic” granulomas / eosinophilic abscesses.

a) c-ANCA - F - p-ANCA in 70%
b) Not associated with eosinophilia - F - peripheral blood + tissue eosinophilia with Loeffle syndrome.
d) Lower frequency of cardiac involvement than Wegener’s - F - higher; heart involved in up to 47% (coronary vasculitis, myocarditis, pericardial tamponade accounts for 50% of
deaths)

232
Q

CT guided biopsy of an atypical non-functional kidney, shows “replacement of
the normal architecture by numerous foamy macrophages intermingled with plasma cell,
lymphocytes, polymorphonuclear leukocytes and occasional giant cells. The latter are not of
the Langerhan’s type,” This suggests:
6.Renal Tuberculosis
7.Renal cell carcinoma
8.Xanthogranulomatous pyelonephritis
9.Oncocytoma
10.Post obstructive atrophy

A

XGP

Xanthogranulomatous pyelonephritis is an unusual and relatively rare form of chronic
pyelonephritis characterized by accumulation of foamy macrophages intermingled with plasma cells, lymphocytes, polymorphonuclear leukocytes, and occasional giant cells. Often associated with Proteus infections and obstruction, the lesions sometimes produce large,
yellowish orange nodules that may be confused with renal cell carcinoma

233
Q

A 50-year-old diabetic woman has papillary necrosis on IVP. Which of the
following pathological features would most favour analgesic nephropathy over diabetic
nephropathy?
1.Duration of diabetes of less than 15 years
2.The absence of calcification
3.Less than 5 papillae affected
4.Lesions of differing ages
5. accelerated atherosclerosis

A

Lesions of differing age

Kidneys either normal or slightly decreased size with irregular outline due to cortical atrophy overlying necrotic papillae
Papillae in various stages of necrosis with calcification, fragmentation, sloughing (cf diabetic papillary necrosis where all papillae are at same stage of necrosis)

papillary necrosis
- nsaids
- sickle cell
- acetaminophen (panadol) and phenacetin
- infection
- dm or dehydration

234
Q

The features of analgesic nephropathy do not include ? Rob 530
1.Renal artery stenosis
2.Renal calculi
3.Increased urinary Tamm-Horsfall protein
4.Nephritis
5.TCC

A

1.Renal artery stenosis

2.Renal calculi (Nephrocalcinosis & stones 20
acquired renal tubular acidosis)
3.Increased urinary Tamm-Horsfall protein (Ischaemic ATN)
4.Nephritis (Papillary necrosis occurs first 20
tubulointerstitial nephritis)
5.TCC

Tamm-Horsfall mucoprotein is the basic matrix of all renal casts it originates from tubular epithelial cells and is always present in the urine.
When the casts contain only mucoproteins, they are called hyaline casts and may not have any pathologic significance.
Hyaline casts may be seen in the urine after exercise or heat exposure, but may also be observed in pyelonephritis or chronic renal disease
Tamm-Horsfall protein = urinary glycoprotein
Seen in multiple myeloma
Ischaemic ATN
Papillae in various stages of necrosis with calcification, fragmentation, sloughing (cf diabetic papillary necrosis where all papillae are at same stage of necrosis)

235
Q

40 year old patient has staghorn calculus not present previous year, LEAST
LIKELY cause is ? Rob p537
1.Hyperuricaemia
2.Crohns Disease
3.Hypercalcuria
4.Hyperoxaluria
5.Recurrent urinary tract infections

A

hyperuricaemia (false)

staghorn calculi
The majority of staghorn calculi are infection stones.
They may start as matrix calculi composed of a glycoprotein cast within the collecting system, upon which triple phosphate infection stones are deposited
Most staghorn stones are composed of struvite
Other stone materials less often encountered that can also assume a staghorn configuration include cystine, calcium oxalate monohydrate, and uric acid
crohns frequently causes oxalate stones, which may cause staghorn type calculi

hyperuricemia
uric acid stones are present in 22% of patients with gout and 42% of those with secondary
hyperuricemia
causes urine to be acidic(c.f. Infection by urea-splitting organisms – Proteus and Staph convert urea
to ammonia  alkaline urine with precipitation of magnesium ammonium phosphate salts )

236
Q

Which is NOT a type of renal cell cancer ? Rob p539
1.Clear
2.Papillary
3.Collecting duct
4.Chromophobe
5.Small cell

A

small cell (false)

1.Clear (most common 80%)
2.Papillary (15%)
3.Collecting duct (rare variant showing irregular channels lined by highly atypical
epithelium with a hobnail pattern)
4.Chromophobe (5%)

237
Q

Features of Clear cell renal cell Ca include ? Rob p539
1.commonest at lower pole
2.tan/white tumour macroscopically

A

tan/white tumour macroscopically (yellow-gray-white in colour)

238
Q

Which is TRUE? Rob p541
1.Intestinal cystitis/ Hunner Ulcer is associated with SLE
2.Malakoplakia is metaplasia secondary to chronic infection
3.5% of bladder tumours are adenocarcinoma
4.5 year survival for grade I tumours is 75%
5.Sessile bladder tumour – 1/3 of low grade, 80% of high grade TCC

A

1.Intestinal cystitis/ Hunner Ulcer is associated with SLE

2.Malakoplakia is metaplasia secondary to chronic infection - Malakoplakia is caused by defects in phagocytic or degradative functions of histiocytes in response to gram negative coliforms; laminated mineralized concretions known as Michaelis-Gutmann bodies are typically present forming an inflammatory plaque but no metaplasia) (leukoplakia =
keratinizing squamous metaplasia / dysplasia) Cystitis Glandularis and Cystitis Cystica = intestinal metaplasia).
3.5% of bladder tumours are adenocarcinoma (rare )
4.5 year survival for grade I tumours is 75% (99%)
5.Sessile bladder tumour – 1/3 of low grade, 80% of high grade TCC (low grade tumours are almost always papillary)

239
Q

Features of Bladder tumours do NOT include:
1.70% solitary
2.low grade lesions tend to be flat
3.Transitional cell in 95%

A

.low grade lesions tend to be flat - false (low grade tumours are almost always papillary)

240
Q

Retroperitoneal sclerosing fibrosis NOT due to T/F? Big Rob p1000
1.Crohns
2.Lymphoma
3.Migraine medication
4.Sclerosing conditions elsewhere

A

all true causes

1.Crohns (inflammatory bowel disease)
2.Lymphoma (true)
3.Migraine medication (ergotamine )
4.Sclerosing conditions elsewhere (mediastinal fibrosis, sclerosing cholangitis, Riedel fibrosing thyroiditis)

The disorder occurs in middle to late age. In some cases, specific causes can be identified, such a drugs (ergot derivatives, beta-adrenergic beta-blockers) adjacent inflammatory conditions (vasculitis, diverticulitis, Crohn disease) malignant disease (lymphomas, urinary tract carcinomas) autoimmune (referred to as mediastinal fibrosis, sclerosing cholangitis, and Riedel fibrosing thyroiditis)
70% of cases have no obvious cause and are considered primary, or idiopathic

241
Q

Bladder cancer morphology path

A

The gross patterns of urothelial cell tumors
vary from purely papillary to nodular or flat to mixed papillary and nodular.
The tumors may also be invasive or noninvasive.
The papillary lesions appear as red elevated excrescences varying in size from less than 1 cm in diameter to large masses up to 5 cm in diameter.
Multicentric origins may produce separate tumors.
As noted, the histologic changes encompass a spectrum from benign papilloma to highly aggressive anaplastic cancers.
Overall, about half of bladder cancers are high-grade lesions.
Most arise from the lateral or posterior walls at the bladder base.

Papillomas and low-grade lesions are almost always papillary.
Higher grades may be flat or papillary.
Many high-grade III lesions may be fungating, necrotic, sometimes ulcerative tumors that
have unmistakably invaded deeply .
With the higher grade neoplasms, in areas of the bladder devoid of tumor, there may frequently be areas of mucosal hyperplasia, dysplasia, or carcinoma in situ.
In most analyses, less than 10% of low-grade cancers invade, but as many as 80% of high-grade transitional cell carcinomas are invasive.
Aggressive tumors may extend only into the bladder wall, but the more advanced stages invade the adjacent prostate, seminal vesicles, ureters, and retroperitoneum, and some produce fistulous communications to the vagina or rectum.
About 40% of these deeply invasive tumors metastasize to regional lymph nodes.
Hematogenous dissemination, principally to the liver, lungs, and bone marrow, generally occurs late, and only with highly anaplastic tumors

242
Q

Focal renal infarct, least likely: (TW & GC)
1.Atherosclerosis
2.Mitral valve regurgitation
3.PAN
4.HUS
5.Endocarditis

A

MVR (false)

1.Atherosclerosis (thrombosis in advanced atherosclerosis)
3.PAN (acute vasculitis of polyarteritis nodosa)
4.HUS (Acute cortical necrosis not focal renal infarct)
5.Endocarditis (vegetative endocarditis)

Most due to embolism
Major source of emboli = mural thrombi from L atrium and vent. from AMI
Less common sources
vegetative endocarditis
thrombosis in aortic aneurysms and aortic atherosclerosis
Rare causes
thrombosis in advanced atherosclerosis
acute vasculitis of polyarteritis nodosa
On gross examination, the kidneys are swollen and pale; many fleabite hemorrhages are on the surface.
In HUS and TTP, the platelet and fibrin microthrombi within the renal microvasculature are accompanied by thrombocytopenia and a microangiopathic hemolytic anemia.
Vasculitis is usually absent.

243
Q

Magnesium ammonium phosphate stones
1.UTI
2.Nidus

A

UTI
Magnesium ammonium phosphate stones (struvite) are formed largely after infections by urea-splitting bacteria (e.g., Proteus and some staphylococci), which convert urea to ammonia.
The resultant alkaline urine causes the precipitation of magnesium ammonium phosphate salts.
These form some of the largest stones, as the amounts of urea excreted normally are huge.
Indeed, so-called staghorn calculi are almost always associated with infection.

244
Q

Fatty lesion next to effaced kidney. Liposarcoma? Which of the following is most
relevant?
1.Tuberous sclerosis
2.Previous radiotherapy

A

radiotherapy

Liposarcomas are one of the most common sarcomas of adulthood and appear in the forties to sixties
they are uncommon in children.
The cause of most soft tissue tumors is unknown.

There are documented associations, however, between radiation therapy and rare instances in which chemical burns, heat burns, or trauma were associated with subsequent development of a sarcoma
They usually arise in the deep soft tissues of the proximal extremities and retroperitoneum and are
notorious for developing into large tumors.
Histologically, liposarcomas can be divided into the following variants.
well-differentiated
myxoid
round cell
pleomorphic

The cells in well-differentiated liposarcomas are readily recognized as lipocytes.
In the other variants, most of the tumor cells are not obviously adipogenic, but some cells indicative of fatty differentiation are almost always present.
They are known as lipoblasts, which mimic fetal fat cells
They contain round cytoplasmic vacuoles of lipid that scallop the nucleus.
The myxoid and round cell variant of liposarcoma has a t(12;16) chromosomal abnormality in most cases.
well-differentiated variant is relatively indolent, the myxoid type is intermediate in its malignant behavior round cell and pleomorphic variants usually are aggressive and frequently metastasize.
All types of liposarcoma recur locally and often repeatedly unless adequately excised.
2/3rd’s of retroperitoneal liposarcomas are low-grade, well differentiated neoplasms.
Most well-differentiated tumors contain mature adipose tissue and only occasional atypical cells.
These neoplasms are often referred to as well-differentiated, lipogenic, or lipoma-like liposarcomas.

245
Q

Incidence of polycystic kidneys
1.1:1000
2.1:10000
3.1:10
4.none of the above

A

APCKD : 1 in 400-1000

246
Q

Which of the following are all recognised bladder carcinoma morphological
patterns:
1.Papillary, ulcerative, diffuse, cystic, infiltrative
2.Papillary, tubular, sessile, excavative
3.Papillary, tubular, cystic, flat
4.Flat, excavative, polypoid
5.Flat, papillary-invasive, papilloma-papillary

A

Flat, papillary-invasive, papilloma-papillary

247
Q

Regarding renal cell carcinoma. Which of the following is the LEAST
CORRECT:
1.Strong association with Von-Hippel-Lindau syndrome
2.Clear cell carcinoma is the most common (70-80%)
3.Increased risk of renal cell carcinoma in chronic renal failure
4.Most frequent presentation is flank pain or discomfort
5.Hereditary tumours are more frequently bilateral and multifocal

A

The three classic diagnostic features of costovertebral pain, palpable mass, and hematuria unfortunately appear in only 10% of cases. The most reliable of the three is hematuria, but it is usually intermittent and may be microscopic; thus, the tumor may remain silent until it attains a
large size.

248
Q

Which of the following is not derived from renal tubules?
1.Papillary cell carcinoma
2.clear cell carcinoma
3.oncocytoma
4.renal tubular adenoma
5.angiomyolipoma

A

aml

249
Q

Regarding prostatic carcinoma. Which of the following is the least correct:
1.70-80% of males between the ages of 70-80 have foci
2.Histologically sheets or cords of cells in dense fibrous sroma
3.70% arise in the peripheral zone
4.Local invasion into the seminal vesicles, bladder and ureter
5.Carcinoma of the prostate is less common in Asia compared with Australia and New
Zealand

A

.Histologically sheets or cords of cells in dense fibrous sroma false

Histologically, most lesions are adenocarcinomas that produce well-defined, readily demonstrable gland patterns.

250
Q

Following NOT associated with increased risk of testicular malignancy:
1.Cryptorchidism
2.Testicular feminisation
3.Testicular microlithiasis
4.Klinefelters syndrome
5.TB of the testis

A

tb

251
Q

In epididymitis, which is most likely to have epididymitis precede orchitis:
1.Syphilis
2.TB
3.Pseudomanas
4.E. Coli
5.Gonococcus

A

tb
Is this recall meant to be ‘which is most likely to occur only in the testis?’
It is classically taught that, of the three major specific inflammatory states, gonorrhea and
tuberculosis almost invariably arise in the epididymis, whereas syphilis affects first the testis.
In sexually active men younger than age 35 years, the sexually transmitted pathogens Chlamydia
trachomatis and Neisseria gonorrhoeae are the most frequent culprits.
In men older than age 35, the common urinary tract pathogens, such as Escherichia coli and
Pseudomonas, are responsible for most infections.

252
Q

40 yo has staghorn calculus not present previous year, least likely cause is:
1.Hyperuricemia
2.Crohns disease
3.Hyper calciuria
4.Hyperoxaluria
5.Recurrent urinary tract infections

A

Hyperuricemia

253
Q

Regarding testicular tumours
1.Choriocarcinoma are large haemorrhagic masses (
2.Spermatologic seminoma occurs in 15-40 year olds
3.Seminoma is most common homogenous testicular mass
4.Mixed tumours are rare in adults
5.teratoma second commonest adult tumour

A

Seminoma is most common homogenous testicular mass

chorio usu <5cm
sperm seminaoma >65
mixed tumours 60%
pure teratoma 2-3%

254
Q

Nodular hyperplasia of prostate which is not true
1.micro seen in 20% in men <40 years old
2.occurs in periurethral and transitional zones
3.demonstate regions of infarction and metaplasia
4.only 50% are symptatic
5.has 2-3X time have increase risk of carcinoma

A

5.has 2-3X time have increase risk of carcinoma false

1.micro seen in 20% in men <40 years old (20% of 40yo men)
2.occurs in periurethral and transitional zones (Almost exclusively in periurethral and
transitional zones of gland extending from bladder neck to verumontanum)
3.demonstate regions of infarction and metaplasia (Frequently, foci of squamous metaplasia and small areas of infarction)

255
Q

DEFINITION of Bowen disease ?

A

A persistent progressive non-elevated red scaly or crusted plaque which is due to an
intradermal carcinoma and is potentially malignant

256
Q

t/f Choriocarcinoma of the testis usually involves the whole testis with capsular invasion

A

false - small lesions

now path cos fuck u

Choriocarcinoma is a highly malignant form of testicular tumor that is composed of both
cytotrophoblast and syncytiotrophoblast.
Identical tumors may arise in the placental tissue, ovary, or sequestered rests of totipotential cells
(e.g., in the mediastinum or abdomen).
In its “pure” form, choriocarcinoma is rare, constituting less than 1% of all germ cell tumors.
As emphasized later, foci of choriocarcinoma are much more common in mixed patterns
Despite their aggressive behavior, pure choriocarcinomas are usually small lesions.
Often, they cause no testicular enlargement and are detected only as a small palpable nodule.
Because they are rapidly growing, they may outgrow their blood supply, and sometimes the primary testicular focus is replaced by a small fibrous scar, leaving only widespread metastases.
Typically, these tumors are small, rarely larger than 5 cm in diameter.
Hemorrhage and necrosis are extremely common.
Histologically the tumors contain two cell types
The syncytiotrophoblastic cell is large and has many irregular or lobular hyperchromatic nuclei and an abundant eosinophilic vacuolated cytoplasm.
As might be expected, HCG can be readily demonstrated in the cytoplasm of syncytiotrophoblastic
cells.
The cytotrophoblastic cells are more regular and tend to be polygonal with distinct cell borders and clear cytoplasm; they grow in cords or masses and have a single, fairly uniform nucleus.

257
Q

Scrotal US – correct
1.Varicocele usu right sided
2.Hydroceles not associated with ca
3.Cause of hydrocele my be unknown
4.Cryptorchidism may be associated with inguinal hernia

A

Cause of hydrocele my be unknown T – primary: idiopathic, secondary

258
Q

Regarding pheos
1.T2hyperintense
2.Extra-adrenal pheos are less malignant than those associated with adrenal
3.2cm at Dx

A

t2 hyper

259
Q

HUS - ? most likely
1.Following viral infection
2.Kidneys may appear normal acutely

A

following viral infection

260
Q

20year old with 6cm testicular tumour and raised bHCG. ?Most likely
1.Choriocarcinoma
2.Spermatocytic seminoma
3.Yolk Sac tumour
4.Mature teratoma
5.Seminoma

A

seminoma - elevated bhcg in 10%

too big for chorio
could be mixed if that were an option doeee

261
Q

Newborn with multiple small renal cysts. Mother, father and sister to be imaged.
Which is most likely? (? Unilateral, ? bilateral)
1.a. Mother, father and sister have normal imaging
2.b. M & F normal, sister may be abnormal
3.c. Sibling may have it
4.Both parents may have it
5.Autosommal recessive, patients die young

A

M & F normal, sister may be abnormal(Depending on age of sister)

262
Q

Kleinfelters, which is true?
1.a. XYY
2.b. 1 : 10 000
3.Recognized presentation is gynaecomastia in adolescence
4.Diagnosed by characteristic facies in neonate (or by age 10)

A

Recognized presentation is gynaecomastia in adolescence T

1.a. XYY F – 47XXY
2.b. 1 : 10 000 F – 1:750 (most common chromosomal aberration)
4.Diagnosed by characteristic facies in neonate (or by age 10) F

263
Q

Which is not a sex cord stromal tumour?
1.Sertoli
2.Leydig
3.Yolk sac

A

yolk sac - nsgct

264
Q

Prostatic carcinoma false:
1.70% males b/w 70-80
2.Common in Asians
3.80% have foci of PIN
4.Gleesons score correlates with well and poorly differentiated areas

A

Common in Asians F – extremely rare in Asians

265
Q

BPH which is true:
1.30-40% at 70 years
2.Severity correlates with symptoms
3.Macroscopic specimen nodular
4.If necrosis, malignant degeneration

A

Macroscopic specimen nodular T – nodules are fairly readily identifiable

necrosis - infarction
90% age 70

266
Q

TCC risks – least likely: (GC & TW)
1.Schistosomiasis
2.Analgesic nephropathy
3.COP with lymphoma
4.Bowel Cancer – treated
5.Upper tract TCC

A

Schistosomiasis - F - SCC

267
Q

Kidney, 1yo with 3cm mass removed, pathology says nephrogenic rests, most correct:
1.Not a real entity
2.Increased risk of Wilm’s
3.Possible entity but not in this age group
4.Only micro diagnosis

A

2.Increased risk of Wilm’s (T)

Nephrogenic rests can also be classified into
perilobar (associated with Wilms)
intralobar (associated with Drash syndrome and WAGR)
on the basis of location and the syndromes with which they are associated

268
Q

Severe adult polycystic kidneys – least likely:
1.Bilateral elevated hemidiaphragms
2.Contour abnormality of the aorta
3.Basal cysts
4.Fluid overload
5.Hyperparathyroidism

A

basal cysts - false

269
Q

Amyloidosis which is NOT correct (CC)
1.2-5% cause renal diseas
2.Is associated with TB
3.40% associated with haemodialysis
4.have types call lardaceous spleen and sago spleen
5.RA 5%

A

1.2-5% cause renal disease - Amyloidosis of the kidney is the most common and cause of
major morbidity [Robbins 5thed p 236]. Dahnert goes on to say primary amyloid is found in 35% and 85% in secondary renal amyloid, the latter would mostly be dialysis associated in first world populations

2.Is associated with TB - Is associated with TB. Chronic inflammatory infective and non
infective states is associated with AA type deposition from protracted cell breakdown
including chronic osteomyelitis, rheumatoid arthritis, scleroderma, dermatomyositis
[Robbins 6th ed]
3.40% associated with haemodialysis – Robbins states 70% associated with haemodialysis [BR 6th ed ] however option A would be the most incorrect of the options.]
4.have types call lardaceous spleen and sago spleen - (T) – Robbins 6th ed
5.RA 5% - (in 3% of pts with RA – Robbins 5th ed p 234)

Renal disease is a frequent manifestation of the systemic amyloidoses and often is the major source of morbidity for individuals with these disorders. Without treatment, amyloidosis-associated kidney disease usually progresses to end-stage renal disease (ESRD). J Am Soc Nephrol 17: 3458–3471,
2006

270
Q

Old diabetic guy with addison’s, which is true? (TW)
1.Hypothalamic / pituitary cause may be possible
2.Cutaneous hyperpigmentation is lacking
3.Autoimmune aetiology accounts for majority of cases
4.Reduction in serum glucocorticoids, with normal mineralocorticoid levels
5.TB is the most common cause

A

Autoimmune aetiology accounts for majority of cases – T – accounts for 60-70% of cases.
May occur as a sporadic or familial disorder. May coexist with other autoimmune disease
(eg Hashimoto, pernicious anemia, T1DM, idiopathic hypoparathyroidism)
4.Reduction in serum glucocorticoids, with normal mineralocorticoid levels - F

1.Hypothalamic / pituitary cause may be possible – F – Addisons is primary adrenocortical insufficiency with progressive destruction of adrenal cortex. Hypothal / pituitary cause is secondary.
2.Cutaneous hyperpigmentation is lacking – F – hyperpigmentation can be seen in Addisons. With decreased levels of glucocorticoids / mineralocorticoids there is increased ACTH precursor hormone levels which stimulate melanocytes. Melanotropic hormone levels are low in secondary adrenocortical insufficiency (ie suppressed ACTH).
4.Reduction in serum glucocorticoids, with normal mineralocorticoid levels - F - >90% of
adrenal cortex destroyed – levels of circulating glucocorticoids and mineralocorticoids are significantly decreased.
5.TB is the most common cause - F – previously this was the most common cause
accounting for 90% cases – now less common with antituberculous agents – but is on the
rise again.

Addisons
90% due to autoimmune attack on adrenal gland.
Other causes: lymphomas, amyloidosis, sarcoidosis, hemochromatosis (could also cause DM),
fungal, adrenal hemorrhages and TB (which used to account for 90%)

271
Q

Fibroadenomas – which of the following is false? (JS)
1.Increase in size with lactation and infarction,
2.Usually less than 35
3.In patients on cyclosporine
4.Are typically infiltrative into the surrounding parenchyma
5.Can calcify

A

4.Are typically infiltrative into the surrounding parenchyma F Usually well circumscribed and demarcated from surrounding tissue (Robbins)

1.Increase in size with lactation and infarction, T – “an inc in size due to lactation or not uncommonly infarction”
2.Usually less than 35 T - Occur at any age but are more common in women under 30 years
(Robbins)
3.In patients on cyclosporine T – “almost half of women receiving cyclosporin after renal
transplant developed fibroadenoma”
5.Can calcify T

272
Q

ARPCKD – Which is false? (JS)
1.not associated with berry aneurysms
2.Associated with bile duct cysts
3.Associated with liver fibrosis
4.Associated with the PKHD1 gene, encoding fibrocystin
5.Histiology demonstrated dilatation of the collecting tubules

A

Associated with bile duct cysts F - ADPCKD

1.not associated with berry aneurysms T - ADPCKD
3.Associated with liver fibrosis, T – Congenital hepatic fibrosis – periportal fibrosis and
proliferataion of well-differentiated biliary ductules (Robbins)
4.Associated with the PKHD1 gene, encoding fibrocystin T
5.Histiology demonstrated dilatation of the collecting tubules T

273
Q

Phaeochromocytoma not a common location? (AB)
1.Adrenal cortex.
2.Adrenal medulla.
3.Mediastinum.
4.Bladder.
5.Organ of Zuckerkandal

A

Adrenal cortex

Answer: A. 90% of phaeochromocytomas occur in the adrenal medulla. Outside of this they are called paragangliomas, and occur in the sympahetic chain. The organ of Zuckerkandl is the most common extra-adrenal site. 1% occur in the thorax (mainly posterior mediastinal). They can enter
urogenital organs like the bladder.

274
Q

Typical features of xanthogranulomatous pyelonephritis? (AB)
1.Foamy macrophages with lipid.
2.5% have staghorn calculi.
3.No perinephric spread.
4.Typically bilateral.

A

Foamy macrophages with lipid.

XGP is a chronic inflammatory disorder associated with infection (Proteus, E.coli).
Calculi in 80%, usually staghorn. XGP is grossly destructive and can infiltrate adjacent organs. Typically unilateral. Foamy lipid-laden macrophages are pathognomonic. Gradual replacement of renal parenchymja and perinephric spread by macrophages becoming enlarged with undigested
bacteria.

275
Q

Bladder CA – LEAST LIKELY (TW)
1.Stones increased risk SCC
2.Schistosomiasis increase risk SCC
3.Urachal remnants Adeno carcinoma-
4.Analine dye – TCC
5.Flat CA better prognosis than papillary CA

A

Flat CA better prognosis than papillary CA - F - flat is bad

1.Stones increased risk SCC - T - metaplasia, with increased risk of SCC
2.Schistosomiasis increase risk SCC - T
3.Urachal remnants Adeno carcinoma- T - urachal remnants are at increased risk of
adenocarcinoma.
4.Analine dye – TCC - T - most aromatic amines and their precurors or derivatives.
Polycyclic aromatic hydrocarbons. Analine dye, arylamines.

276
Q

Which tumour is associated with Schiller-Duval bodies? (TW)
1.Spermatocytic seminoma
2.Embryonal
3.Yolk sac tumour
4.Teratoma
5.Choricarcinoma

A

Yolk sac tumour - T - AKA endodermal sinus tumor. Shiller-Duval bodies. Pure yolk sac
tumor is the most common malignant testicular GCT in pre-pubertal children. Pure YS is rare in adult, but occurs in 40% of mixed GCTs.

277
Q

What is true regarding primary hyperaldosteronism? (JS)
1.Hyperkaelaemia
2.Adenoma <2cm
3.Adenoma >3cm

A

Adenoma <2cm - T - 80% of cases are caused by an adenoma in one gland (Conn
syndrome). These are almost always solitary and small (<2cm). Other causes include
primary adrenocortical hyperplasia or glucocorticoid suppressible hyperaldosteronism

.Hyperkaelaemia - F - Primary hyperaldosteronism is a group of syndromes characterised by excess aldosterone secretion, causing sodium retention and potassium excretion with resultant hypertension and hypokalaemia

278
Q

Which is true regarding cervical cancer? (GC)
1.Extension into the upper vagina is associated with a poor prognosis
2.Adenocarcinoma has a worse prognosis than squamous cell carcinoma
3.An early lesion looks like a fungating ulcer

A

.Adenocarcinoma has a worse prognosis than squamous cell carcinoma - T - AdenoCa is
derived from the mucus-secreting columnar epithelium of the cervical canal; comprises
about 5-10% of all primary malignant tumours of the cervix. Compared with SCC it has a worse prognosis. [Adelaide notes]

.Extension into the upper vagina is associated with a poor prognosis - F- still surgically
manageable (stage IIA), 5YS 82%.
3.An early lesion looks like a fungating ulcer - F - invasive Ca develops in the
transformation zone, manifests in 3 patterns: fungating (or exophytic), ulcerating (see
below), and infiltrating cancer. Macroscopically, early lesions appear as granular irregular areas of the cervical epithelium. Late lesions appear as fungating ulcerated areas which
destroy the cervix [Robbins; Adelaide notes]

279
Q

A 61-year-old women has a bone scan after surgical resection of a node-negative 4-
millimetre medullary breast carcinoma. This shows long areas of increased activity in the
right fibula, left 6th and 12” ribs and the left frontal bone. This distribution is most typical of which of the following; (N.B. All alternatives are true causes of increased activity on bone scan).
4.Paget’s Disease of Bone
5.Metastatic disease
6.Fibrous dysplasia
7.Insufficiency fractures
8.Langerhan’s Histiocytosis

A

pagets - usually pelvis, femur, skull, tibia, but can have ribs/fibula

FD unilateral
LCH - long segments unusual

Medullary Carcinoma
Higher incidence in women carrying BRCA1 gene (13%)
Usually 2 – 3 cm, but may be large fleshy tumours up to 5cm
The prognosis is slightly better than that of other types ? all those lymphocytes fighting the cancer, or that this tumor is a distinct disease? Maybe both
5mm node negative with mets??? Give me a break, cant be true

Pagets
Polyostotic (85%)
pelvis (75%) > femur > skull > tibia > vertebra > clavicle > humerus > ribs
Polyostotic without Endocrine Dysfunction
lesions may be confined to a single limb or single side of skeleton

Langerhan’s Histiocytosis
Any bone in skeletal system maybe involved
most commonly skull (diploic space of parietal bone) (note epidermoid has sclerotic rim), ribs and femora

280
Q

As a radiologist in a trial of non-accidental injury you are questioned about Vitamin C
deficiency. Which of the following statements about scurvy is most correct?
1.There is a insufficient, osteoid matrix formation with slowing of cartilage resorbtion
2.There is a proportional reduction in both bone mineral and osteoid deposition
3.There is rapid cartilage growth leading to poor periosteal support
4.There is a primary disturbance in mineralisation of the bone matrix leading to cartilage overgrowth.
5.There is excessive but largely un-mineralised bone matrix simulating increased cartilage

A

There is a insufficient, osteoid matrix formation with slowing of cartilage resorbtion (there is insufficient production of osteoid matrix by osteoblasts, Resorption of the cartilaginous matrix then fails or slows; as a consequence, there is cartilaginous overgrowth)

The primary disturbance is in the formation of osteoid matrix, rather than in mineralization or calcification, such as occurs in rickets.
In scurvy, the palisade of cartilage cells is formed as usual and is provisionally calcified.
However, there is insufficient production of osteoid matrix by osteoblasts.
Resorption of the cartilaginous matrix then fails or slows; as a consequence, there is cartilaginous overgrowth, with long spicules and plates projecting into the metaphyseal region of the marrow cavity, and sometimes widening of the epiphysis

281
Q

Concerning the early stages of osteoarthrosis which of the following statements is most
correct?
1.The concentration of proteoglycans increases
2.Matrix water content decreases
3.Chondrocytes proliferate forming clones
4.Cartilage and subchondral bone fragments are present in the joint in large quantities
5.Bony eburnation is observed

A

Chondrocytes proliferate forming clones

Early
Chondrocyte proliferation
increased matrix water
decreased proteoglycans in cartilage matrix
Fibrillation of cartilage + loss of superficial cartilage layers
Cartilage surface granular & soft

Intermediate
Nonspecific synovitis without pannus
Eventually sloughing of full-thickness portions of cartilage with exposure of subchondral bone
Friction results in bone eburnation
Subchondral bone plate thickens & underlying cancellous bone becomes sclerotic
Small fractures occur through subchondral bone plate
Fragments of bone & cartilage form loose bodies within joint (loose bodies,joint mice)

Later
Fractures through cartilage allow synovial fluid to be forced into subchondral regions forming fibrous walled subchondral cysts
Cysts may also reflect rarefaction due to osteoclastic activity &/or osteocyte death
Osteophytes develop at articular surface margins capped by fibrocartilage & hyaline cartilage that gradually ossifies
Synovium becomes congested + fibrotic

282
Q

Concerning rheumatoid nodules which of the following statements is most correct?
1.They typically occur in regions of skin subjected to pressure
2.They are usually manifest in patients with relatively mild disease
3.They only involve cutaneous structures
4.They occur in over 50% of patients
5.They are typically exquisitely tender

A

They typically occur in regions of skin subjected to pressure (rise in regions of the skin that are subjected to pressure, including the ulnar aspect of the forearm, elbows, occiput, and lumbosacral area)

Rheumatoid nodules
are the most common cutaneous lesion.
They occur in approximately 25% of patients, usually those with severe disease, and arise in
regions of the skin that are subjected to pressure, including the ulnar aspect of the forearm, elbows, occiput, and lumbosacral area.
Less commonly, they form in the lungs, spleen, pericardium, myocardium, heart valves, aorta, and other viscera.
Rheumatoid nodules are firm, nontender, and round to oval and in the skin arise in the
subcutaneous tissue.
Microscopically, they have a central zone of fibrinoid necrosis surrounded by a prominent rim of epithelioid histiocytes and numerous lymphocytes and plasma cells

283
Q

Which of the following statements concerning Paget’s disease of bone lS LEAST correct?
1.There is an initial osteolytic phase
2.Current evidence suggests it is due to a slow virus infection of bone
3.Involvement of the ribs, fibula or small bones or the hands or feet is unusual
4.Recognised complications include malignant fibrous histiocytoma and chondrosarcoma
5.Whist there is a proliferation of osteoclasts, osteoblasts and bone resulting in cortical
thickening, the overall lamellar pattern remains ordered in the absence of malignancy.

A

Whist there is a proliferation of osteoclasts, osteoblasts and bone resulting in cortical
thickening, the overall lamellar pattern remains ordered in the absence of malignancy. (The newly formed bone is disordered and architecturally unsound)

1.There is an initial osteolytic phase (initially active/osteolytic stage)
2.Current evidence suggests it is due to a slow virus infection of bone (Slow virus infection of osteoblasts / osteoclasts by paramyxovirus  promotes secretion of IL-6 from infected cells  potent stimulator of osteoclast activity & bone resorption)
3.Involvement of the ribs, fibula or small bones or the hands or feet is unusual (pelvis (75%) > femur > skull > tibia > vertebra > clavicle > humerus > ribs)
4.Recognised complications include malignant fibrous histiocytoma and chondrosarcoma
(malignant fibrous histiocytoma 25% and chondrosarcoma 5%)

Sarcomatous Change in ~ 1%  lytic lesion often with cortical breakthrough
50% osteosarcoma
25% malignant fibrous histiocytoma
15% remainder fibrosarcoma
5% chondrosarcoma
5% GCT
suggested clinically by change in type & severity of bone pain enlarging mass
pathological fracture

284
Q

Concerning ankylosing spondylitis which of the following statements is most correct?
1.It is only observed in individuals who are HLA-B27 positive
2.It is not associated with synovitis
3.It is the only condition in which syndesmophytes are observe
4.It is associated with rheumatoid factor positivity in over 50% of cases
5.It involves peripheral joints (e.g. knees, hips shoulders) in over 20% of patients.

A

It involves peripheral joints (e.g. knees, hips shoulders) in over 20% of patients.
(Peripheral large joints - hips, shoulders, knees are involved in up to 1/3)

hlab27 90%
chronic synovitis
reiters also has syndesmophytes

285
Q

Ewings sarcoma versus osteomyelitis, LEAST LIKELY if ? Rob p21
1.Fever
2.Afro-American

A

afro american - caucasians 90%

Ewings Sarcoma vs Osteomyelitis
Duration of symptoms shorter (<2 weeks)
May look indentical
M:F = 2:1
Caucasians in 96%, blacks are rarely afflicted
Of all bone sarcomas, Ewing sarcoma has the youngest average age at presentation as most patients are 10 to 15 years old
approximately 80% are younger than 20 years.
In approximately 85% of Ewing sarcomas and PNETs, there is a t(11;22)(q24;q12) translocation
Location: femur (25%), pelvis-ilium (14%), tibia (11%), humerus (10%), fibula (8%), ribs (6%)
long bones in 60%: flat bones in 40%
The treatment of Ewing sarcoma includes chemotherapy and surgery with or without radiation. The advent of effective chemotherapy has dramatically improved the prognosis from a dismal 5% to
15% to a 75% 5-year survival; at least 50% are long-term cures.

286
Q

Chondrosarcomas ? Rob p766
1.chondroblastoma may give rise to chondrosarcoma
2.grade I – III < 40% 5ys
3.chondroblastic osteosarcoma is part of chondrosarcoma classification
4.Occur 15-40 years

A

1.chondroblastoma may give rise to chondrosarcoma (Although a significant number of conventional chondrosarcomas arise in association with a preexisting enchondroma, few develop within an osteochondroma, chondroblastoma, or fibrous dysplasia or in the setting of Paget disease)

2.grade I – III < 40% 5ys (grade 1 – 90% , grade 2 80% , grade 3 40%)
3.chondroblastic osteosarcoma is part of chondrosarcoma classification (Osteosarcoma)
4.Occur 15-40 years (peak 40 -60 years)

287
Q

Synovial sarcoma
1.most common in tendon sheaths of fingers/toes
2.Metastasises to regional LN’s, lung and bones

A

Metastasises to regional LN’s, lung and bones

Synovial sarcoma
<10% intra articular
10% of soft tissue sarcomas (4th)
20’s-40’s
In the vicinity of the large joints of the extremities. 60-70% around lower extremities, especially knee
The histological hallmark of synovial sarcomas is the biphasic morphology of the tumour cells (i.e. Epithelial like and spindle cells)
5 year survival 25-60%
Mets to regional LN’s lung and skeleton

288
Q

Osteoblastoma vs Osteoma
1.no such thing as an osteoblastoma
2.spinal involvement
3. distinct histologically

A

spinal involvement (osteoblastoma involves the spine more frequently)

Identical histological features
differ in size, sites of origin, and symptoms

Osteoid Osteoma
<2cm
75% < 25 yo
50% femur and tibia.
Pediliction for appendicular skeleton
commonly in cortex, less frequently medulla
painful. Relieved by aspirin

Osteoblastoma
Involves spine more frequently
Dull ache
Not associated with marked bony reaction
involves the spine more frequently and is
usually larger than 2cm, and the
pain does not respond to salicyclates

289
Q

The feature most diagnostic of gout is:
1.Acute pain 1st MTP joint
2.-ve birefringent crystal on aspirate
3.para articular erosions
4.hyperuricemia

A

.-ve birefringent crystal on aspirate

290
Q

Rh nodules – least likely site
1.Pinna of ear
2.External surface forearm
3.Kidney
4.Spleen

A

kidney

Rheumatoid nodules are the most common cutaneous lesion. They occur in approximately 25% of patients, usually those with severe disease, and arise in regions of the skin that are subjected to pressure, including the ulnar aspect of the forearm, elbows, occiput, and lumbosacral area. Less
commonly, they form in the lungs, spleen, pericardium, myocardium, heart valves, aorta, and other viscera.
Microscopically, they have a central zone of fibrinoid necrosis surrounded by a prominent rim of epithelioid histiocytes and numerous lymphocytes and plasma cells

291
Q

CPPD, which is incorrect:
1.Hypo Mg
2.RA
3.OA
4.Hemochromatosis

A

RA

Three main associated conditions (i.e. often present when CPPD present – do not cause CPPD):
Primary hyperparathyroidism
Haemochromatosis
Gout (40% with gout have CPPD)
Other associations:
Prior joint damage
Hypomagnesaemia
Hypothyroidism
Ochronosis
Calcium pyrophosphate crystal deposition disease (CPPD), also known as pseudogout and chondrocalcinosis
It usually occurs in individuals over 50 years of age and becomes more common with increasing age, rising to a prevalence of 30% to 60% in those 85 years or older.
The sexes and races are equally affected.
CPPD is divided into sporadic (idiopathic), hereditary, and secondary types.
In the hereditary variant, the crystals develop relatively early in life and are associated with severe osteoarthritis. One family with this disorder showed linkage of the disease with chromosome 8q.

The secondary form is associated with various disorders, including
previous joint damage
hyperparathyroidism
hemochromatosis
hypomagnesemia
hypothyroidism
ochronosis
diabetes
gout. May mimic other disorders eg RA

whip a dof
wilsons
haemochromatosis, haemophilia, hypothyroid, hyperparathyroid, hypophos, hypomagnesaemia
idiopathic
pseudogout
arthritis, amyloid, acromegaly
dm
ochronosis
gout

292
Q

SLE, which is false: (GC)
1.Type III hypersensitivity reactions account for most of the systemic lesions
2.The presence of antibodies to dsDNA is virtually diagnostic
3.Discoid rash and oral ulcers are diagnostic criteria
4.Lupus anticoagulant is a misnomer
5.Inflammation of small arteries and veins is characteristic

A

Inflammation of small arteries and veins is characteristic F - acute necrotising vasculitis
affecting small arteries and arterioles. Necrosis and fibrinoid deposits within vessel wals containing antibody, DNA, complement fragments and fibrinogen; transmural and
perivascular infiltrate often present. Chronic stages show fibrous thickening with luminal
narrowing.

1.Type III hypersensitivity reactions account for most of the systemic lesions T - immune
complexes (DNA/anti-DNA), especially in glomeruli.
2.The presence of antibodies to dsDNA is virtually diagnostic T - as is anti-Smith Ag.
3.Discoid rash and oral ulcers are diagnostic criteria T - 11 criteria established by WHO are
used for diagnosis. Need 4 or more of these, serially or simultaneously. Others include:
malar rash, photosensitivity, nonerosive arthritis, serositis (pleuritis or pericarditis), renal involvement, seizures/psychosis, haematologic disorder, immunologic disorder (eg. antiDNA, anti-Sm, antiphospholipid Ab’s), positive ANA test.
4.Lupus anticoagulant is a misnomer T - Antiphospholipid syndrome is a hypercoagulable state in vivo: the antibodies induce direct platelet activation and/or interfere with endothelial cell production of prostaglandins. However, in vitro (in the absence of platelets and endothelium) the antibodies interfere with phospholipid complex assembly and thus
inhibit coagulation.

293
Q

Fragile X, which is incorrect: (GC)
1.Trinucleotide repeat mutation of a gene found on the X chromosome.
2.Long face with large mandible
3.Hyperextensible lax joints
4.Mental retardation
5.20% of males appear normal

A

Hyperextensible lax joints F

1.Trinucleotide repeat mutation of a gene found on the X chromosome. T - FMR1 gene.
2.Long face with large mandible T - also large everted ears and large testicles.
4.Mental retardation T - one of the most common causes of familial mental retardation. Also seen in 50% of carrier females.
5.20% of males appear normal T - clinically and cytogenetically normal. These “carrier
males” can transmit the disease to their grandsons through their phenotypically normal daughters.

Fragile X syndrome results from loss of FMR1 gene function and is characterised by mental
retardation, macro-orchidism, and abnormal facial features In the normal population there are about 29 CGG repeats in the FMR1 gene. Carrier males and females carry “premutations” with up to 200 repeats that can expand to 4000 repeats (full mutations) during oogenesis. Fragile X syndrome occurs when full mutations are transmitted to offspring.

294
Q

Suspected NAI, which is most concerning: (GC)
1.SIDS of a sibling 13 months old F
2.Healing fracture of the clavicle at 1 month.
3.Brachial plexus injury
4.Symmetrical periosteal reaction of distal femora
5.Bilateral irregularity of distal metaphyses

A

Healing fracture of the clavicle at 1 month T - especially if exuberant callus formation, or
other fractures in different stages of healing.

3.Brachial plexus injury F - may be related to birth trauma.
4.Symmetrical periosteal reaction of distal femora F - differential includes NAI (large
subperiosteal haematomas), but many other causes to consider, particularly if bilateral and
symmetrical. Scurvy, trauma, infection (TORCH, syphilis), neoplasm (metastatic neuroblastoma, lymphoma), Caffey’s, Kinky hair, PgE therapy, sickle cell disease,
hypervitaminosis A & D…
5.Bilateral irregularity of distal metaphyses F - NAI probably less likely if bilateral.
CHARMS: congenital infection (rubella, syphilis), copper deficiency, hypophosphatasia, achondroplasia, rickets, metaphyseal dysostosis, scurvy

295
Q

.Synovial sarcoma, which is most correct: (GC)
1.Calcification uncommon
2.Most common in tendon sheaths of fingers/toes
3.Originates from synoviocytes
4.Peak age 50-60 years old
5.Rarely involves the joint cartilage

A

Rarely involves the joint cartilage T - lesion is usually about 1cm removed from joint
cartilage, <10% involve joint.

1.Calcification uncommon F - amorphous calcification in 30%, often at periphery. More
likely to be calcified than other forms of mesenchymal sarcoma.
2.Most common in tendon sheaths of fingers/toes F - most common around knee. Other sites include hip, ankle (2/3 lower extremity), elbow, wrist, hands & feet, parapharyngeal region, abdominal wall.
3.Originates from synoviocytes F - malignant tumour of mesechymal cell origin, actual cell
of origin unclear. Resembles developing synovium.
4.Peak age 50-60 years old F - 20-40 yo.

296
Q

.Serum electrophoretic analysis and urinary Bence Jones protein assessment are normal.
Which of the following statements is most correct: (GC)
1.The findings exclude multiple myeloma
2.1 in 100 cases of multiple myeloma would fit this pattern
3.20% of case of multiple myeloma would fit this pattern
4.Bence Jones protein assessment should have done on serum
5.In approximately 70% of patients the electrophoretic pattern alone is normal

A

1 in 100 cases of multiple myeloma would fit this pattern T - 1% are non-secretory.

4.Bence Jones protein assessment should have done on serum F - excreted by the kidney.
5.In approximately 70% of patients the electrophoretic pattern alone is normal F - increased Ig levels in blood, urine, or both in 99%; blood + urine 60-70%; urine alone 20%.

297
Q

Myeloma diagnosed on x-ray, normal plasma electrophoresis with negative Bence Jones
proteins in urine. Which is false: [AB]
1.1% patients with multiple myeloma are negative secretors
2.Patient likely has Waldenstrom’s
3.Patient may be anaemic.
4.Patient may present with confusion.

A

Patient likely has Waldenstrom’s

99% of myeloma patients have abnormal urine (BJ proteins - light chains) or serum (M spike - single immunoglobulin) electrophoresis. Waldenstrom macroglobulinaemia is another monoclonal
gammopathy, and would have similar findings. It is associated with lymphoplasmacytic lymphoma. Myeloma patients may be anaemic (extensive marrow involvement) or have neurologic
effects from hypercalcaemia.

298
Q

Sclerotic lesion thought to be Paget’s. Least likely: [AB]
1.Japanese tourist
2.Female
3.Lytic lesion in same bone
4.Cranial nerve palsies

A

japanese

Paget disease is very rare in the Asian population. There is only a slight male predominance. Lytic lesions in the same bone may reflect the mixed phase of the disease or sarcomatous degeneration (about 1%, higher in polyostotic disease). Cranial nerve palsies are an uncommon but classic presentation.

299
Q

LCH. Which is false: [AB]
1.Letterer-Siwe can result in punched out skull lesions
2.Letterer-Siwe 2-7 yo
3.Can present in skull in teenager
4.Is associated with diabetes insipidus

A

Letterer-Siwe 2-7 yo false

EG, the most common form of LCH, can present up to age 30 with the skull most frequently affected. Letterer-Siwe, the least common form of LCH, presents before age 2 with prominent extraosseous involvement. The skull can have multiple lytic lesions causing “raindrop skull”.
Hand-Schuller-Christan, the third form, classically (50%) causes diabetes insipidus due to mass effect on or direct infiltration of the posterior pituitary.

300
Q

Regarding Amyloidosis, which is NOT correct: (JS)
1.2-5% cause renal disease
2.Is associated with TB
3.40% associated with haemodialysis
4.have types call lardaceous spleen and sago spleen
5.RA 5%

A

1.2-5% cause renal disease F – amyloidosis of the kidney is the most common form of
organ involvement, causing proteinuria and nephrotic syndrome with CRF and uraemia.
(Robbins)

2.Is associated with TB T – secondary amyloidosis is associated with TB, bronchiectasis, osteomyelitis, RA, AS, IBD, RA, haemodialysis (Robbins).
3.40% associated with haemodialysis ?T – not sure about the figures but 60-80% of
patients on long term dialysis have amyloid deposits in synovium, joints and tendon sheaths. (Robbins).
4.have types call lardaceous spleen and sago spleen T – Sago spleen: amyloid deposits
producing tapioca-like granules on gross inspection. Lardaceous spleen: fusion of these deposits gives rise to maplike areas. (Robbins)
5.RA 5% T – amyloid occurs in 3% of patients with RA (Robbins).

301
Q

Femur with OS and abnormal distal femur not associated with: (JS, with TW & GC)
1.Pagets
2.Fibrous dysplasia
3.Renal osteodystrophy
4.Radiation
5.Bone infarct

A

Renal osteodystrophy

302
Q

Chondromyxoid fibroma, which of these is most likely: (JS)
1.25yo female with metaphyseal lesion at site of a previous ABC
2.20 yo male involving the metaphysis
3.5 yo with Epiphyseal lesion
4.Another answer specifying an Epiphyseal location

A

20 yo male involving the metaphysis T - Typical location is metaphysis (50%) and in the 2nd to 3rd decade

303
Q

Which is most correct regarding the pathogenesis of Rheumatoid arthritis? (GC)
1.Interleukins are involved.
2.Infection is a trigger
3.Immune complexes play a pivotal role in the process.
4.Cross reaction occurs with hydroxyapatite crystals.

A

1.Interleukins are involved.T - CD4+ helper T cells produce cytokines (eg. TNF), that
activate macrophages and B cells. Activated macrophages produce IL-1, resulting in
proliferation of synovial cells and fibroblasts). IL-1 and TNF also account for the
constitutional symptoms (fever, malaise, weakness) that may be seen in RA.

2.Infection is a trigger. F - thought to be triggered by exposure of a genetically predisposed individual to an arthitogenic agent, possibly microbial. Adelaide notes state that EBV (and other microbial agents) have been postulated but firm evidence of a microbial trigger is not available.
3.Immune complexes play a pivotal role in the process. F - 80% of RA patients have
rheumatoid factor - a serum IgM autoantibody that binds to the Fc portion of the body’s own
IgG. RF immune complexes form in sera, synovial fluid and membranes. However, 20% do not have RF, suggesting that these autoantibodies are not essential for tissue injury in RA
(Robbins). Circulating IC’s contribute to extra-articular disease and may have a role in
joint destruction (Adelaide notes).
4.Cross reaction occurs with hydroxyapatite crystals. F - not considered a pathogenetic
variable.

304
Q

Which is most correct in regards to fibrous dysplasia (TW)
1.Benign bone lesion with all components of normal bone present
2.Benign bone lesion with metaplasia of connective tissue
3.Benign bone lesion with abnormal giant cells
4.Benign bone lesion with multiple thin blood-filled cystic cavities
5.Benign bone lesion with excessive fibrous proliferation

A

1.Benign bone lesion with all components of normal bone present - T - benign lesion due to a localised development arrest in which all the component sof normal bone are present but do not differentiate into theri mature structures (path notes). Defect in osteoblastic
differentiation and maturation (D 6th).

2.Benign bone lesion with metaplasia of connective tissue - F - Myositis ossificans - general term which refers to heterotopic soft tissue ossification with several aetiologies. It is due to metaplasia of intermuscular connective tissue and not to muscle inflammation (path notes)
3.Benign bone lesion with abnormal giant cells - F - GCT has osteoclast type multinucleated giant cells scattered evenly throughout tissue. Giant cells can also be seen in brown tumor,
giant cell reparative granuloma, chondroblastoma, PVNS.
4.Benign bone lesion with multiple thin blood-filled cystic cavities - F - ABC
5.Benign bone lesion with excessive fibrous proliferation - F - although FD may have
moderately cellular fibroblastic proliferation, see fibrous proliferation with the various
fibrous and fibro-osseous tumors (Robbins), eg. FCD, NOF, benign fibrous histiocytoma.

305
Q

Which demonstrates a hyperplastic synovium (TW)
1.Acute Gout
2.Rheumatoid arthritis
3.Osteoarthritis
4.Myositis ossificans
5.CPPD

A

.Rheumatoid arthritis - T - thick, oedematous hyperplastic synovium (path notes)

Acute Gout - F - supersaturation of synovial fluid with MSU. Microtophi in synovial cells.
Acute stage - get oedematous and congested synovium. In chronic tophaceous arthritis of
gout, get hyperplastic synovium (path notes).
3.Osteoarthritis - F - Articular cartilage is major cause of degenerative change in OA.
synovium shows minor alterations in comparsion to the destruction of the articular surface and is congested and fibrotic. Nonspecific synovitis without pannus. (Robbins / Path notes)
4.Myositis ossificans - F - metaplasia of intermuscular connective tissue and not to muscle inflammation (path notes)
5.CPPD - F

306
Q

Foreign body giant cells arise in (TW)
1.Rheumatoid arthritis
2.Gout
3.Psoriasis
4.Reiter’s
5.Ankylosing spondylitis

A

Gout - T - tophi are hallmark of gout - represent large aggregations of urate crystals - surrounded by intense inflammatory reaction of macrophages, lymphocytes, and foreign body giant cells (path notes)

307
Q

.Features of a giant cell tumour include (TW)
1.Peak incidence 10-20yo
2.Most common in flat bones of the chest and pelvis
3.10% sarcomatous transformation
4.lung metastases 0.5 – 5%
5.giant cells are monoclonal

A

10% sarcomatous transformation - T - sarcomatous transformation 5-10%

1.Peak incidence 10-20yo - F - epiphyses closed for GCT. 80% occur between 20-50yo
2.Most common in flat bones of the chest and pelvis - F - Knee 50-65% (distal femur 23-
30%, prox tibia 20-25%), distal radius 10-12%, flat bones 15%.
4.lung metastases 0.5 – 5% - ?F - although does metastasise to lungs, D6th and path notes say 5% (?F as not 0.5-5%).
5.giant cells are monoclonal - F - cannot find anything to say monoclonal. Histo:
multinucleated osteoclastic giant cells in a diffuse distribution in a background of
mononuclear cells intermixed throughout a spindle cell stroma.

308
Q

Morton’s neuroma, which of the following is most correct? (TW)
1.Most common between 4th and 5th digits
2.Not a true neuroma
3.Presents as a painful lump
4.Represents axonal degeneration
5.Ethanol ablation is not appropriate

A

Not a true neuroma - T - MN is perineural fibrosis entrapping a plantar digital nervs;
neuroma is a misnomer.

1.Most common between 4th and 5th digits - F - typically 2nd / 3rd intermetatarsal space
(rarely 1st / 4th).
3.Presents as a painful lump - F - numbness; burning/tingling electric forefoot pain
increasing with activity + wearing of narrow shoes.
4.Represents axonal degeneration - ?F - exact cause unknown, MN most likely represents an
entrapment neuropathy causing perineural fibrosis, nerve degeneration, leukocyte
infiltration, and epineural and endoneural vascular hyalinization that results in a
significantly thicker intermetatarsal nerve (AJR 2000).
5.Ethanol ablation is not appropriate - F - alcohol injection of MN has a high success rate and is well tolerated (AJR 2007)

309
Q

Chordoma is characterised by (TW)
1.Histologic absence of calcification
2.Arises from remnants of nucleus pulposus
3.Physaliferous cells
4.Low recurrence following radical surgery

A

Physaliferous cells - T - typical chordoma: cords + clusters of large bubblelike vacuolated (physaliferous) cells containing intracytoplasmic mucous droplets.

1.Histologic absence of calcification - F - amorphous calcification 50-75%
2.Arises from remnants of nucleus pulposus - F - originates from embryonic remnants of
nonochord / ectopic cordal foci (nonochord appears between 4th and 7th week of embryonic development, extends from Rathke pouch to coccyx and forms nucleus pulposus)
4.Low recurrence following radical surgery - F - almost 100% recurrence rate despite
radical surgery

310
Q

Which is true regarding rheumatoid arthritis? (TW)
1.RF is an antibody (IgG) vs the Fc portion of IgM
2.Commonest 5th-7th decades -
3.C spine involved in 50%
4.Female to male ratio of 10:1

A

Commonest 5th-7th decades - T - Peak age 45-65yo (D 6th ed). Peak incidence during 5th
to 8th decades

1.RF is an antibody (IgG) vs the Fc portion of IgM - F - RF is an autoantibody (IgM, but
also IgA, IgE) against Fc portion of IgG
3.C spine involved in 50% - ?F - Cervical spine involvement in RA is common, up to 90%
(Wheelesonline). Frequency of radiographic signs of involvement of cervical area is in the
range of 43-86% depending on the duration of the disease (eMedicine).
4.Female to male ratio of 10:1 - F - M:F 1:3 if <40yo. 1:1 if >40yo

311
Q

Which is true regarding enchondromas? (TW)
1.Increased prevalence of ovarian Ca and CNS glioma with Olliers disease
2.Generally always contains ring & arc type calcifications
3.More likely to be found in the distal phalanx than the proximal phalanx
4.Solitary more common than multiple

A

Solitary more common than multiple - T - usually solitary (D 6th ed).

1.Increased prevalence of ovarian Ca and CNS glioma with Olliers disease - F - Increased
prevalence of ovarian carcinoma, pancreatic carcinoma, CNS gioma, and GIT
adenocarcinoma with Maffucci’s syndrome (D 6th ed.)
2.Generally always contains ring & arc type calcifications - F - B&H - must have Ca++
unless in phalages.
3.More likely to be found in the distal phalanx than the proximal phalanx - F - 40% small
tubular bones of wrists and hand (most frequent tumor here), distal and mid aspects of metacarpals, proximal / middle phalanges (D 6th ed).

312
Q

Which is true regarding aneurysmal bone cysts? (TW)
1.60% found in the spine
2.Metaphyseal when found in a long bone
3.>50% associated with GCT
4.50% associated with other lesions

A

Metaphyseal when found in a long bone - T - eccentric in metaphysis in long bone (femur,
tibia, humerus, fibula)

1.60% found in the spine - F - 30% (path notes)
3.>50% associated with GCT - F - of the 30% of all ABC’s, 40% of these arise in GCT’s
(which = 12% total, path notes). 14% GCTs may have focal ABC components in tumor
centre (D 6th ed).
4.50% associated with other lesions - F - primary ABC 70%. 30% ABCs arise in preexisting bone tumors.

313
Q

7.Which is true regarding osteoporosis? (TW)
1.Postmenopausal osteoporosis is more common in women than men by a ratio of 20:1 -
2.Can occur secondary to malignancy
3.Postmenopausal osteoporosis is characterised by increased osteoblastic activity and decreased bone turnover -
4.Is characterised by reduced bone mass and abnormal composition

A

Can occur secondary to malignancy - T - Primary osteoporosis (postmenopausal, agerelated). Secondary (endocrine disorder, neoplasia, GI, rheumatoligc, drugs, miscellaneous)

1.Postmenopausal osteoporosis is more common in women than men by a ratio of 20:1 - F - postmenopausla 6:1 F:M. Age related osteoporosis 2:1 F:M.
3.Postmenopausal osteoporosis is characterised by increased osteoblastic activity and decreased bone turnover - F - increased osteoclast activity. Increase resoprtion. Essentially “high turnover” form of osteoporosis. Decreased oestrogen - increased IL-1 by blood moncytes - stimulator osteoclast recruitment and activity.
4.Is characterised by reduced bone mass and abnormal composition - F - reduced bone mass of normal composition.

314
Q

16-year-old boy with gynaecomastia has a +ve beta hCG on blood test. The most likely
cause for this is: (–)
3.Late onset of puberty
4.A testicular seminoma
5.A testicular non-seminomatous germ cell tumour
6.A mediastinal dermoid
7.A lamellar-variant hepatoma

A

A testicular non-seminomatous germ cell tumour

Germ cell tumors are the most common cancer in males between the ages of 15 and 35.
They are divided into seminomatous and nonseminomatous subtypes (embryonal carcinoma, yolk sac carcinoma, choriocarcinoma, teratomas).
Elevated AFP and βHCG function as reliable markers in some tumors. As a result of the increased βHCG, acting analogously to LH to stimulate the Leydig cell LH receptor, testicular estrogen production is also increased, which, in turn, can cause gynecomastia.

Gyanecomastia is one of the manifestations of Klinefelter syndrome and may occur in those with functioning testicular neoplasms, such as Leydig cell and, rarely, Sertoli cell tumors It may occur at any time during adult life when there is cause for hyperestrinism. The most important cause of hyperestrinism in the male is cirrhosis of the liver, since the liver is responsible for metabolizing estrogen. Drugs such as alcohol, marijuana, heroin, anabolic steroids used by some athletes and body builders, and some psychoactive agents have also been associated with gynecomastia

315
Q

Which of the following statements concerning the prostate gland is LEAST correct? (–)
1.Nodular hyperplasia arises from the peri-urethral glands.
2.Corpora amylacea are focal stromal calcifications
3.Carcinoma typically arises in the peripheral zone.
4.Urethral involvement is usually a late clinical feature in prostate carcinoma.
5.Histological grading of carcinoma provides important prognostic information

A

Corpora amylacea are focal stromal calcifications. - F - corpora amylacea are concretions that occur in benign prostate gland, seen in the lumen of the prostatic acini. [GC: inspissated proteinaceous secretory material within glandular lumina; glands are surrounded by proliferating stromal elements].

1.Nodular hyperplasia arises from the peri-urethral glands. - T - hyperplasia of prostatic
stromal and epithelial cells, resulting in the formation of large, fairly discrete nodules in the periurethral region of the prostate
3.Carcinoma typically arises in the peripheral zone. - T - in approximately 70% of cases,
carcinoma of the prostate arises in the peripheral zone of the gland, classically in a
posterior location, often rendering it palpable on rectal examination. Characteristically, on
cross-section of the prostate, the neoplastic tissue is gritty and firm, but when embedded
within the prostatic substance, it may be extremely difficult to visualize and be more readily apparent on palpation.
4.Urethral involvement is usually a late clinical feature in prostate carcinoma. - T - Urinary
symptoms occur late because posterior subcapsular location of tumour → but palpable on PR.
5.Histological grading of carcinoma provides important prognostic information. - T -
Grading is of particular importance in prostatic cancer because there is in general fairly good correlation between the prognosis and the degree of differentiation.

Nodular hyperplasia, still referred to by the redundant term BPH (all hyperplasias are benign), is an extremely common disorder in men over age 50.

316
Q

A 52-year-old man is diagnosed with prostatic carcinoma. He attends for his bone scan
with several Internet articles and asks you “what does a Gleason Score of 1 mean?” The most
correct reply is: (–)
1.A tumour entirely confined to the peripheral zone of the prostate
2.A tumour entirely confined by the prostatic capsule
3.There is no Gleason Score of one
4.A poorly differentiated / anaplastic tumour
5.An extremely well differentiated tumour

A

There is no Gleason Score of one

Grade 1 represents the most well differentiated tumors, in which the neoplastic glands are uniform and round in appearance and are packed into well-circumscribed nodules
By contrast, grade 5 tumors show no glandular differentiation, and the tumor cells infiltrate the stroma in the form of cords, sheets, and nests.
The other grades fall in between.
Because most tumors contain more than one pattern, it is usual to assign a primary grade to the dominant pattern and a secondary grade to the subdominant pattern.
The two numerical grades are then added to obtain a combined Gleason grade (grade is from 1–5) or score (score is never <2 – see below).
Thus, for example, a tumor with a dominant grade 3 and a secondary grade 4 would achieve a Gleason score of 7.
Tumors with only one pattern are treated as if their primary and secondary grades are the same, and, hence, the number is doubled.
Thus, under this schema the most well differentiated tumors have a Gleason score of 2 (1+1) and the least-differentiated tumors merit a score of 10 (5+5).
Grading is of particular importance in prostatic cancer because there is in general fairly good correlation between the prognosis and the degree of differentiation

317
Q

A 23-year-old man has scrotal pain and ultrasound shows bilateral testicular swelling
with a bilateral hyperaemia and a diffuse alteration in texture. Further questioning reveals a
week’s history of painful unilateral parotid swelling. The most likely explanation of these
findings is: (GC) (–)
1.Acute Myeloid Leukaemia
2.Ebstein Barr Virus Infection
3.Sarcoidosis
4.Mumps Virus Infection
5.Hairy cell Leukaemia

A

Mumps Virus Infection - T - orchitis complicates mumps infection in 20% of adult males, rarely in kids. Parotitis typically precedes the onset or orchitis by 1 week. 1/3 of postpubertal infected men dvp testicular atrophy, 2-10% become infertile.

Mumps virus is similar in structure to measles virus except that the large glycoprotein on its surface has both hemagglutinin and neuraminidase activities; spread by respiratory droplets and multiply
within upper respiratory epithelial cells, salivary glands, and T-cells in lymph nodes.
A transient viremia spreads the mumps virus to other glands and the CNS through the choroid plexus.

Mumps parotitis: bilateral in 70%; affected glands are enlarged, have a doughy consistency, moist,
glistening, reddish brown on cross-section. On microscopic examination, the gland interstitium is edematous and diffusely infiltrated by histiocytes, lymphocytes, and plasma cells that compress acini and ducts. Neutrophils and necrotic debris may fill the ductal lumen and cause focal damage
to the ductal epithelium.

Mumps orchitis: testicular swelling may be marked, caused by edema, mononuclear cell
infiltration, and focal hemorrhages; swelling may compromise the blood supply and cause areas of infarction. Sterility, when it occurs, is caused by scars and atrophy of the testis after resolution of viral infection.

Pancreas: lesions may be destructive, causing parenchymal and fat necrosis and
polymorphonuclear cell infiltration.
Mumps encephalitis: perivenous demyelinization and perivascular mononuclear cuffing.

318
Q

In epidymiditis which is MOST LIKELY to have epididymitis preceding orchitis ? (GC)
1.Syphilis
2.TB
3.Pseudomonas
4.E. Coli
5.Gonococcus

A

TB - T - usually begins in epididymis (haematogenous spread), with secondary
involvement of testis; prostate and seminval vesicles are usually also infected. Orchitis
rarely occurs without epididymitis.

1.Syphilis - F - testis usually involved first. Discrete gummas contribute to enlarged,
irregular testis.
3.Pseudomonas - F - ascending infection
4.E. Coli - F - ascending infection
5.Gonococcus - F - usually spreads from posterior urethra to prostate, SVs and epididymis. Testis involved only if untreated.

It is classically taught that, of the 3 major specific inflammatory states, gonorrhea and TB almost invariably arise in the epididymis, whereas syphilis affects first the testis.
In sexually active men <35yrs, the sexually transmitted pathogens C.trachomatis and
N.gonorrhoeae are the most frequent culprits.
In children and men >35yrs, the common urinary tract pathogens, such as E.coli and
Pseudomonas, are responsible for most infections.

Primary syphilis: chancre (hard, painless ulcer) usually on penis, resolves.
Secondary syphilis occurs 2mths later - generalised LN enlargement, condyloma lata on external genitalia (also oral cavity, pharynx).
Tertiary syphilis occurs in 1/3 of untreated pts, latency >5yrs, aortitis, chronic meningovascular disease, tabes dorsalis, general paresis, gummas (bone, skin, mm of upper airway and mouth; any organ can be affected).

319
Q

Choriocarcinoma of the testis t/f usually involves the whole testis with capsular invasion

A

F - typically, these tumors are
small, rarely larger than 5cm in diameter.

Choriocarcinoma is a highly malignant form of testicular tumor that is composed of both
cytotrophoblast and syncytiotrophoblast.
Identical tumors may arise in the placental tissue, ovary, or sequestered rests of totipotential cells (e.g., in the mediastinum or abdomen).
In its “pure” form, choriocarcinoma is rare, constituting less than 1% of all germ cell tumors.
foci of choriocarcinoma are much more common in mixed patterns
Despite their aggressive behavior, pure choriocarcinomas are usually small lesions. Often, they cause no testicular enlargement and are detected only as a small palpable nodule.
Because they are rapidly growing, they may outgrow their blood supply, and sometimes the primary testicular focus is replaced by a small fibrous scar, leaving only widespread metastases.
Hemorrhage and necrosis are extremely common.

Histologically the tumors contain two cell types:
syncytiotrophoblastic cell is large and has many irregular or lobular hyperchromatic nuclei and an abundant eosinophilic vacuolated cytoplasm. HCG can be readily demonstrated in their cytoplasm.
cytotrophoblastic cells are more regular and tend to be polygonal with distinct cell borders and clear cytoplasm; they grow in cords or masses and have a single, fairly uniform nucleus.

320
Q

3.Regarding testicular tumours ? Rob p660 (GC)
1.Choriocarcinoma are large haemorrhagic masses
2.Spermatocytic seminoma occurs in 15-40 year olds
3.Seminoma is most common homogenous testicular mass
4.Mixed tumours are rare in adults (<5%)
5.Teratoma is the second commonest adult tumour

A

Seminoma is most common homogenous testicular mass - T - account for 50% of GCTs;
large soft, well-demarcated, usually homogeneous grey-white tumours, typically confined to testis by an intact tunica albuginea. Micro: sheets of large uniform cells with distinct cell borders, glycogen-rich cytoplasm and round nuclei with conspicuous nucleoli; arranged in lobules with intervening fibrous septa; lymphocytic infiltrate; 25% stain positive for hCG.

1.Choriocarcinoma are large haemorrhagic masses - F - usually small lesions, <5cm
2.Spermatocytic seminoma occurs in 15-40 year olds - F - >65 years, ie. older than classic
seminoma (40-50yrs).
4.Mixed tumours are rare in adults (<5%) - F - 40% of all testicular GCTs. Most common
combination is teratoma/embryonal/yolk sac.
5.Teratoma is the second commonest adult tumour - F - Adults: pure forms rare (2–3%),
mixed more common (45%), regarded malignant. Children: pure forms common (2nd to yolk sac tumours), and usually benign.

321
Q

Nodular hyperplasia of prostate which is FALSE ? Rob p665 (GC) (–)
1.micro seen in 20% in men <40 years old -
2.occurs in periurethral and transitional zones
3.demonstate regions of infarction and metaplasia
4.less than 50% are symptomatic
5.has 2-3X time have increase risk of carcinoma

A

has 2-3X time have increase risk of carcinoma - F - not assocd with increased risk.

1.micro seen in 20% in men <40 years old - T - present in a significant no. of men by the
age of 40yo; frequency rises to 99% by the 8th decade.
2.occurs in periurethral and transitional zones - T - almost exclusively in periurethral and transitional zones of gland extending from bladder neck to verumontanum.
3.demonstate regions of infarction and metaplasia - T - areas of infarction are fairly
common in advanced cases of NH, and are frequently accompanied by foci of squamous
metaplasia.
4.less than 50% are symptomatic - T - clinical symptoms are seen in 10%, most commonly
lower urinary tract obstruction (inner portions of gland involved). Hesitancy, urgency, nocturia, poor stream, predisposed to recurrent UTIs, acute retention.

322
Q

Following NOT associated with increased risk of testicular malignancy: (GC)
1.Cryptorchidism
2.Testicular feminisation
3.Testicular microlithiasis
4.Klinefelters syndrome
5.TB of the testis

A

TB of the testis false

1.Cryptorchidism - T - 3 to 5-fold increase in risk, usually seminoma; due to foci of
intratubular germ cell neoplasia. Also infers an increased risk of Ca in the contralateral
descended testis; risk increases with the degree of ectopy; orchidopexy reduces (but does not elminate) the risk.
2.Testicular feminisation - T - increased risk with intersex syndromes, including androgen
insensitivity syndrome and gonadal dysgenesis. Gonadoblastoma is the most common malignancy in dysgenetic gonads.
3.Testicular microlithiasis - T - assocd with GCT in 40%. Defect in phagocytic activity of
Sertoli cells leaving degenerated intratubular debris behind.
4.Klinefelters syndrome - T - 47XXY; the major consequences of the extra sex chromosome,
usually acquired through an error of nondisjunction during parental gametogenesis, include hypogonadism, gynecomastia, and psychosocial problems. Klinefelter syndrome is a form of
primary testicular failure, with elevated gonadotropin levels due to lack of feedback
inhibition by the pituitary gland. Testicular atrophy leads to infertility in almost all pts.
Increased risk of breast carcinoma (20-fold cf. that in normal men), acute leukemia,
Hodgkin and NHL, CML, and other myeloproliferative diseases. Gonadal and extragonadal GCTs (mediastinal germ cell tumors, teratoma, teratocarcinoma, choriocarcinoma) may occur. Infertility alone can increase risk of intratubular germ cell neoplasia.

323
Q

Which is true regarding Bowen’s disease of penis? (GC & TW)
1.It is associated with HPV infection
2.Cowden’s disease
3.Turcot’s

A

It is associated with HPV infection – T – most commonly type 16.

In external male genitalia, 3 lesions that display histologic features of Ca in situ have been described: Bowen disease, erythroplasia of Queyrat, and bowenoid papulosis. All these have strong association with infection by HPV (type 16 in 80%).
Bowen disease: occurs in the genital region of men and women, usually in >35yo. In men, it is prone to involve the skin of the shaft of the penis and scrotum. Over the span of years, Bowen disease may become invasive and transform into typical SCC in approx 10% of patients.
Erythroplasia of Queyrat: on the glans and prepuce.
Bowenoid papulosis: younger age than Bowen’s and multiple (cf. solitary), pigmented reddishbrown papular lesions.

324
Q

Concerning Hodgkin’s disease, which of the following statements IS
INCORRECT? (–)
6.Nodular sclerosing sub-type has an excellent prognosis
7.Lymphocyte depletion sub-type has a poor prognosis
8.Long term survivors of chemotherapy and radiotherapy are of increased risk of developing lung cancer
9.Pulmonary involvement occurs by extension of disease from hilar nodes to
peribronchovascular interstitial tissue
10.Involvement of mesenteric nodes and Waldeyer’s ring is common

A

Involvement of mesenteric nodes and Waldeyer’s ring is common (uncommon)

6.Nodular sclerosing sub-type has an excellent prognosis (Most common form (65 – 75% cases) and has excellent prognosis)
7.Lymphocyte depletion sub-type has a poor prognosis (True but controversial entity)
8.Long term survivors of chemotherapy and radiotherapy are of increased risk of developing lung cancer (→ Myelodysplastic syndromes, AML, lung, stomach, melanoma, breast (esp young women treated with radiotherapy to chest), NHL)
9.Pulmonary involvement occurs by extension of disease from hilar nodes to
peribronchovascular interstitial tissue (pulmonary involvement usually extends from hilum in untreated Hodgkins)

325
Q

Which of the following statements concerning Langerhan’s Histiocytosis is
correct? (–)
1.The term encompasses the older designations Hand-Schuller-Christian disease and Eosinophilic Granuloma. Letterer-Siwe now considered a separate condition
2.In those under two years of age, bony lesions dominate the disease
3.The bones most commonly involved include the pelvis, scapula and clavicles
4.The diagnosis cannot be made without histology confirming Birbeck granules
5.It is associated with recurrent ear and respiratory infections

A

It is associated with recurrent ear and respiratory infections – T - Multifocal Langerhans cell histiocytosis usually affects children, who present with fever; diffuse eruptions, particularly on the scalp and in the ear canals; and frequent bouts of otitis media, mastoiditis, and upper respiratory tract infections.

Acute disseminated Langerhans cell histiocytosis ( Letterer-Siwe disease) occurs most frequently before 2 years of age but occasionally may affect adults.
Most of those affected have concurrent hepatosplenomegaly, lymphadenopathy, pulmonary lesions, and eventually destructive osteolytic bone lesions.

1.The term encompasses the older designations Hand-Schuller-Christian disease and Eosinophilic Granuloma. Letterer-Siwe now considered a separate condition – F - In the past, these disorders were referred to as histiocytosis X and subdivided into three
categories: Letterer-Siwe disease, Hand-Schuller-Christian disease, and eosinophilic
granuloma. These three conditions are now believed to represent different expressions of the same basic disorder
2.In those under two years of age, bony lesions dominate the disease – F - The dominant clinical feature is the development of cutaneous lesions that resemble a seborrheic eruption secondary to infiltrations of Langerhans histiocytes over the front and back of the trunk and on the scalp.
3.The bones most commonly involved include the pelvis, scapula and clavicles – F -
Virtually any bone in the skeletal system may be involved, most commonly the calvaria, ribs,and femur
4.The diagnosis cannot be made without histology confirming Birbeck granules – F -
Because Birbeck granules are not seen in all tumor cells by electron microscopy, the
detection of CD1a expression by immunohistochemical techniques aids in the diagnosis

326
Q

You are following up a patient whose skull x-ray you reported as suspicious
of myeloma. The hospital pathology computer shows that the serum electrophoretic analysis
and urinary Bence Jones protein assessment are normal. Which of the following statements is
most correct? (TW) (–)
1.These finding excludes multiple myeloma
2.Approximately 1 in 100 cases of multiple myeloma would fit this pattern
3.Approximately 20% of cases of multiple myeloma would fit this pattern
4.Bence Jones protein assessment should have been done on serum; in approximately 70%of patients the electrophoretic pattern alone is normal
5.These tests are not relevant to the diagnosis of multiple myeloma

A

Approximately 1 in 100 cases of multiple myeloma would fit this pattern – T - probably
most true. Nonsecretory MM in ~3%.

Multiple myeloma is characterized by the neoplastic proliferation of a single clone of plasma cells producing a monoclonal immunoglobulin.

Approximately 3% of patients with MM have no M-protein in the serum or urine on immunofixation at the time of Dx and are considered to have nonsecretory myeloma. The condition remains nonsecretory in most patients (76%).
The monoclonal Ig produces a high spike when serum or urine is subjected to electrophoresis.
Immunoelectrophoresis or immunofixation is used to identify the nature of monoclonal Ig
Quantitative analyses of monoclonal Ig usually reveal more than 3 gm of Ig per dl of serum and more than 6 mg of Bence Jones proteins per dl of urine.
The most common serum monoclonal Ig (M protein) is IgG, which is found in 55% of patients.
An additional 25% of cases are associated with an IgA M protein.
Myelomas expressing IgM, IgD, or IgE occur but are rare.
Excessive production and aggregation of M proteins leads to the hyperviscosity syndrome
(described under lymphoplasmacytic lymphoma) in approximately 7% of patients, most of whom have tumors that secrete IgA or IgG3 .
Bence Jones proteinuria and a serum M protein are both observed in 60 to 70% of all myeloma patients.
In approximately 20% of patients, however, Bence Jones proteinuria is present as a isolated finding.
It should also be noted that about 1% of myelomas are nonsecretory and thus occur in the absence of detectable serum or urine M proteins.
Hence, the absence of paraproteinemia and paraproteinuria does not exclude myeloma

327
Q

A 48 year old man is referred for a CT scan from his gastroenterologist for a
gastric marginal zone lymphoma / MALToma. Which of the following statements concerning
marginal zone lymphomas IS LEAST correct? (TW) (–)
1.They are a form of B cell lymphoma
2.They are most common in middle aged adults
3.Early dissemination is typical
4.They are associated with Helicobacter gastritis
5.They are associated with chronic inflammation secondary to autoimmune disorders

A

Early dissemination is typical (false tend to remain localized for long period)

1.They are a form of B cell lymphoma – T – NHL subtype of marginal zone lymphoma
includes 3 distinct diseases (appear to arise from post-germinal center marginal zone B
cells): Splenic MZ B-cell lymphoma; Extranodal MZ B-cell lymphoma of mucosa associated lymphoid tissue (MALT lymphoma); and Nodal MZ b-cell lymphoma.
2.They are most common in middle aged adults - T
4.They are associated with Helicobacter gastritis – T – related to chronic immune
stimulation. H. pylori infection / chronic gastritis association and the development of gastricMALT is the prototypical example.
5.They are associated with chronic inflammation secondary to autoimmune disorders – T – see ans d.

Arise within tissues affected by chronic inflammation
salivary gland in Sjögren disease
thyroid in Hashimoto thyroiditis
stomach in H pylori gastritis

328
Q

Which of the following statements concerning small lymphocytic lymphoma
IS LEAST correct? (TW) (–)
1.It differs from chronic lymphocytic leukaemia only in the extent of peripheral
lymphocytosis
2.The patients are frequently asymptomatic
3.Generalised lymphadenopathy and hepatosplenomegaly are seen in over half the patients
4.Approximately 10-20% of cases transform to a more aggressive lymphoid neoplasm
5.Prognosis is poor with a 10% 5 year survival

A

Prognosis is poor with a 10% 5 year survival – F - Median survival 4 – 6 yrs

1.It differs from chronic lymphocytic leukaemia only in the extent of peripheral
lymphocytosis – T – according to the WHO classification, B-cell small lymphocytic
lymphoma is considered to be identical to the mature B-cell neoplasm chronic lymphocytic
leukemia.
2.The patients are frequently asymptomatic – T – patients usually seek medical attention for
painless generalized lymphadenopathy, which has frequently been present for several years.
3.Generalised lymphadenopathy and hepatosplenomegaly are seen in over half the patients - T
4.Approximately 10-20% of cases transform to a more aggressive lymphoid neoplasm - t/f - Richter’s transformation is the development of an aggressive large-cell lymphoma in the
setting of underlying chronic lymphocytic leukemia / small lymphocytic lymphoma.
Incidence estimated at 2-9%.

Chronic lymphocytic leukemia ( CLL) and small lymphocytic lymphoma ( SLL) are
morphologically, phenotypically, and genotypically indistinguishable, differing only in the degree of peripheral blood lymphocytosis.
Generalized lymphadenopathy and hepatosplenomegaly are present in 50 to 60% of the cases
Overall the median survival is 4 to 6 years, but patients who present with minimal tumor burden may survive for 10 years or more
May transform to more aggressive forms
Prolymphocytic transformation (15 – 30%)
diffuse large B-cell lymphoma (Richter syndrome) (10%)

329
Q

A 24-year-old woman with Stage II lymphocyte predominate Hodgkin’s
lymphoma has been neutropaenic for 2 weeks in intensive care. Since her pre-treatment CT scan 3 months earlier her liver and spleen have increased in size and show new lesions up to 2 cm in size. Despite multiple antibiotics-she continues to decline and CT shows a new left MCA infarct. This is most compatible with: (–) (TW, GC, JS)
1.Systemic candidiasis
2.Bacterial endocarditis with systemic emboli /infarction
3.Drug resistant lymphocyte predominate Hodgkin’s lymphoma
4.Transformation to a more aggressive lymphoma
5.Non-bacteria thrombotic endocarditis with systemic emboli / infarction

A

1.Systemic candidiasis – T - arterial embolisation is more common in fungal IE than bacterial (larger vegetations). Most common sites are cerebral circulation, extremities and GIT.

2.Bacterial endocarditis with systemic emboli /infarction
3.Drug resistant lymphocyte predominate Hodgkin’s lymphoma
4.Transformation to a more aggressive lymphoma
5.Non-bacteria thrombotic endocarditis with systemic emboli / infarction

LP Hodgkins usually has a good prognosis
Stage II = Involvement of two or more lymph node regions on the same side of the diaphragm alone
(II) or with involvement of limited contiguous extralymphatic organ or tissue (IIE )
in contrast to other forms of HD, approximately 3 to 5% of tumors transform to diffuse large cell lymphomas, which are uniformly of B-cell type
Patients with diffuse large B-cell lymphoma typically present with a rapidly enlarging, often symptomatic mass at a single nodal or extranodal site.
Involvement of the gastrointestinal tract, skin, bone, or brain may be the presenting feature.
Waldeyer ring, the oropharyngeal lymphoid tissues that include the tonsils and adenoids, is also commonly involved. There may be large destructive masses in the liver and spleen
Bone marrow involvement occurs late in the disease, and rarely a leukemic picture may emerge

330
Q

A 32 year-old woman in the third trimester of her first pregnancy develops
HELP syndrome. This is: (–)
1.Hemolysis elevated liver enzymes and low platelets
2.Hepatic eosinophilia with low level of prothrombin
3.Haemolytic eosinophilic liver necrosis with low platelets
4.Hemorrhagic ncephalopathy/Ieukomaiacia with low platelets
5.Hemorrhagic encephalopathy and leukocyte lysis with platelet autolysis

A

Hemolysis elevated liver enzymes and low platelets

Hemolysis, Elevated Liver enzymes and low Platelet count - complicating Preeclampsia-eclampsia
Preeclampsia is a complication of pregnancy and is characterized by maternal hypertension, proteinuria, peripheral edema, coagulation abnormalities, and varying degrees of disseminated intravascular coagulation .
When hyperreflexia and convulsions occur, the condition is called eclampsia. Hepatic disease is distressingly common in preeclampsia, usually as part of a syndrome of hemolysis, elevated liver enzymes, and low platelets, dubbed the HELLP syndrome.

331
Q

Findings on ultrasound on 16year old female with spherocytosis, which is least
likely ? Rob p398 (–) (GC)
1.Gallstones
2.Renal stones
3.Splenomegaly
4.Enlarged lymph nodes

A

Renal stones - F

1.Gallstones - T - (bilirubin-rich stones may occur in extravascular haemolysis – ie. pigment stones , occurs 40-50%)
3.Splenomegaly - T - (occurs in most form of haemolytic anemias as there is active
hyperplasia of the mononuclear system. Splenomegaly occurs more often and is more
marked in HS than other haemolytic anaemias)
4.Enlarged lymph nodes - T - (extramedullary hematopoiesis), theoretically.

HS: autosomal dominant disorder caused by inherited mutations that affect RBC membrane skeleton, leading to loss of membrane and eventual conversion of RBCs to spherocytes, which are phagocytosed and removed in the spleen Manifested by anaemia, splenomegaly and jaundice.
Aplastic crises may be triggered by parvovirus B19 infection.

332
Q

In Thalassemia Major which is TRUE? Rob p403 (–) (GC)
1.Anaemia is secondary to abnormal β-haemoglobin
2.Present at age 6 to 9 months
3.Splenomegaly only occurs if cirrhosis

A

Present at age 6 to 9 months - T - (change over from HbF to HbA)

Thalassaemias are a group of autosoomal codominant disorders in which mutations in the alpha or β globin genes result in reduced Hb synthesis, causing a microcytic hypochromic anaemia. βthalassaemia major occurs when any two B0 and B+ alleles are inherited (homozygous or compound heterozygous), whilst β-thal minor occurs when only one abnormal allele is inherited.
Most common cause is mutations that lead to aberrant mRNA processing.
β+ results in reduced β globin synthesis
β0 results in total β globin absence
(ie. doesn’t form abnormal β globin)

333
Q

Elderly man with polycythemia. Non-associated finding on CT is: (–) (GC)
1.Emphysema
2.Renal mass
3.Chronic pancreatitis
4.Splenomegaly

A

Chronic pancreatitis - not assoc

Polycythemia - relative (haemoconcentration) or absolute (primary or secondary)

Primary: autonomous prolifn of myeloid stem cells, nearly all have mutation in JAK2 (tyrosine kinase), RBC is rendered hypersensitive to EPO (normal or low EPO levels), occurs in 40-60yo.

Secondary (increased erythropoietin levels):
Appropriate:
Lung disease, high altitude living, cyanotic heart disease
Inappropriate:
Erythropoitetin-secreting tumours
Renal cell carcinoma
Hepatoma
Cerebellar haemangioblastoma
Uterine Fibroids
Surreptitious erythropoieten use (athletes)

334
Q

Patient with suspected mononucleosis, which would be ATYPICAL ? Rob p417-
418 (–) (GC)
1.Abnormal LFT’s
2.Proteinuria with viral inclusion in epithelial cells in urine
3.Groin lymphadenopathy
4.Mononuclear meningitis
5.Atypical lymphocytes in FNA

A

Proteinuria with viral inclusion in epithelial cells in urine - F

1.Abnormal LFT’s - T - liver fxn is almost always transiently impaired to some degree.
Assocd with jaundice, elevated enzymes, disturbed appetite, and rarely even liver failure.
3.Groin lymphadenopathy - T - widespread lymphadenopathy with atypical lymphocytes
4.Mononuclear meningitis - T - occurs but is rare
5.Atypical lymphocytes in FNA - T - atypical lymphocytes occur in both peripheral blood
smears and lymph nodes FNA. Mainly cytotoxic T-cells; occasionally cells resembling Reed Sternberg cells are present.

Note : Infectious mononucleosis
herpes virus family associated with the development of Burkitt lymphoma and nasopharyngeal carcinoma
splenomegaly occurs

335
Q

Solitary plasmacytoma of nasal cavity. Which is TRUE ?
1.Must go onto MM
2.20% progress to multiple myeloma
3.mucosal covered
4.<2cm
5.bone expansion

A

20% progress to multiple myeloma - T - rate of progression to MM is lower than in
solitary bone plasmactyoma, ranging from 11-30% at 10yrs (eMed). Som says eventually
35-50% with primary extramedullary plasmacytoma develop MM.

  1. false
  2. submucosal growth
  3. can be several cm
  4. remodels bone

Solitary plasmacytoma:
may be skeletal (solitary bone P) or soft tissue (extramedullary) in origin
SBP occurs in a similar distribution to MM (predilection for red marrow of axial skeleton); classically causes bone destruction; rarely a cystic lesion, or slcerotic lesion (latter assocd with POEMS syndrome).
EMP occurs mainly in aerodigestive tract (80% in the head & neck) - 28% in nasal cavity, 22% in paranasal sinuses. Around 30-75% of pts with EMP are alive after 5yrs, however, the median length of survival after onset of MM is <2yrs. Treatment is with surgery and radiotherapy.

336
Q

Patient with Waldenstroms macroglobulinaemia, which is most correct ? (–)
(GC)
1.Bone lesions indistinguishable from multiple myeloma
2.Bone lesions in <3%
3.Bone lesions involving cortex
4.Solitary Bone lesion most likely similar sites as multiple myeloma but spares skull
5.Subperiosteal deposits – endosteal spread

A

Bone lesions in <3% - T - unlike myeloma, lytic bone lesions are “uncommon” (eMed).
Robbins says “does not produce lytic bone lesions”; Dahnert says “lytic lesions seen on
skeletal surveys in up to 20%”.

Lymphoplasmacytic lymphoma is a low grade lymophoid malignancy composed of mature
plasmacytoid lymphocytes.
Production of abnormal monocloncal IgM protein leads to Waldenstrom’s MG (hyperviscosity syndrome).
Sx: visual impairment (retinal vein distension, retinal h’ages), headaches/dizziness/deafness,
bleeding, cryoglobulinaemia (Raynaud, cold urticaria).
BM infiltration in >90%, may be diffuse or variegated replacement. Lymphadenopathy,
hepatosplenomegaly.
M-spike on serum/urine EP, no BJ proteinuria.

337
Q

What is the MOST CONTROVERSIAL (and rarest ?) subgroup in Hodgkins
lymphoma classification? Rob p431 (–)
1.Nodular sclerosing
2.Lyphocyte predominant
3.Lymphocytic proliferative
4.Lyphocyte deplete
5.Mixed cellularity

A

Lyphocyte deplete (very rare)
(lymph prolif not a subgroup)

There are three major subtypes of HD:
Nodular sclerosis
Mixed cellularity
Lymphocyte predominance
Additional variants, such as lymphocyte depletion HD, are rare and controversial entities

338
Q

Patient with polythaemia ruba vera which is NOT TRUE? Rob p439 (GC)
1.Transform to AML
2.Transform to CML
3.Develop/associated with myelofibrosis
4.Associated with splenomegaly and cirrhosis
5.Risk of stroke
6.Associated with Budd-Chiari

A

Transform to CML - F

1.Transform to AML - T - blast crisis occurs spontaneously in 2% of pts treated with
phlebotomy alone; in 15% who receive myelosuppresive treatment.
3.Develop/associated with myelofibrosis - T - “spent phase” - 20% progress
4.Associated with splenomegaly and cirrhosis - T - hepatosplenomegaly (foci of
extramedullary haematopoeisis), thrombosis (eg. Budd-Chiari syndrome, MI), infarcts (eg.
CVA, spleen, kidney), haemorrhages, pruritis (histamine from basophils), gout.
5.Risk of stroke - T - see above.
6.Associated with Budd-Chiari - T - see above.

339
Q

Patient with abnormal bleeding, normal platelets but decreased factor VIII +
cofactor ristocetin – DIAGNOSIS is? Rob p449 (–)
1.Haemophilia A
2.Haemophilia B
3.Von Willebrands disease
4.Lupus anticogulation
5.DIC

A

Von Willebrands

(Haemphilia A, factor VIII def, classic
haemophilia B, fractor IX, Christmas)

Patients with von Willebrand disease have prolonged bleeding time with normal platelet count. The plasma level of vWF, measured as the ristocetin cofactor activity, is reduced. Because vWF stabilizes factor VIII by binding to it, a deficiency of vWF gives rise to a secondary decrease in factor VIII levels

340
Q

DEFINITION of Thymoma: Rob p451
1.Any benign thymic tumour
2.Benign tumour of epithelial cells
3.Any tumour of thymic epithelial cells
4.Any tumour of thymus
5.Benign tumours of epithelial and mesenchymal origin

A

Any tumour of thymic epithelial cells

341
Q

AML: (–)
1.Philadelphia chromosome
2.Most common in children

A

philly philly

Most common in children (primarily affects adults 15-39 years, ALL is the one which
usually affects children)
primarily affects adults 15-39 years
accounts for only 20% of childhood leukaemias
Predisposing conditions
Alkylating agents
Myeloproliferative disorders
Aplastic anaemia
PNH
lymphadenopathy + hepatosplenomegaly due to infiltration by leukemic cells, less than ALL
meningeal infiltration - headaches, nausea, vomiting, papilloedema, CN palsies, seizures, coma intracerebral / subarachnoid haemorrhages from thrombocytopenia + leukostasis, less than ALL
60-80% show remission with chemotherapy but relapse within 12-18 months
15 – 30 % 5 yr survival
Prognosis influenced by chromosomal abnormalities:
t(8;21) or inversion of 16 = good
t(9;22) Ph or 11q23 = bad
Philidelphia chromosome usually in CML but does occur in AML and is associated with a bad prognosis

342
Q

Megaloblastic anaemia is not caused by: (–)
1.Pernicious anaemia
2.Crohn’s disease
3.Coeliac disease
4.Pancreatic calcification

A

pancreatic calcification

The two most common causes of megaloblastic anemia are vitamin B-12 deficiency (cobalamin)
and folinic acid deficiency.

In the cell:
cobalamin acts as a coenzyme in 2 reactions: the synthesis of methionine from homocysteine and the conversion of methylmalonyl CoA to succinyl CoA.
folate binds to and acts as a coenzyme with enzymes responsible for single carbon metabolism.

343
Q

Sickle cell anaemia. An unusual feature would be: (GC)
1.splenomegaly
2.splenic atrophy/ absent
3.fungal dactylitis
4.aplastic crises
5.bone infarction

A

fungal dactylitis - F

1.splenomegaly - T - in children, during the early phase of the disease, the spleen is
commonly enlarged up to 500g
2.splenic atrophy/ absent - T - marked congestion of the red pulp, due mainly to the trapping of sickled red cells in the splenic cords and sinuses. Erythrostasis in the spleen leads to thrombosis and infarction, or at least to marked tissue hypoxia. Continued scarring causes progressive shrinkage of the spleen so that by adolescence or early adulthood, only a small nubbin of fibrous tissue may be left; ie. autosplenectomy
4.aplastic crises -T - aplastic crisis represents a temporary cessation of bone marrow
activity, usually triggered by parvovirus infection of erythroid progenitor cells.
Reticulocytes disappear from the peripheral blood, and there is sudden and rapid worsening of anemia.
5.bone infarction -T - in children, painful bone crises are extremely common and often
difficult to distinguish from acute osteomyelitis. They frequently manifest as the hand-foot syndrome because of dactylitis of the bones of the hands or feet or both

344
Q

A 34-week obstetric ultrasound is requested to assess fetal growth. The
clinical information states says “Mother “O” /father “AB”, Haemolytic disease on the
newborn (HDNP)??” They have had no counselling and are worried about damage to this,
their second baby. What is the best summary of their situation? (–)
1.There is no risk of HDN from ABO incompatibility
2.There is no risk of HDN with their given blood groups
3.The risk depends how the last delivery was managed but may approach 100%.
4.Only 1 in 200 cases at risk of ABO related disease require treatment
5.Only 1 in 10 cases at risk of ABO related disease develop HDN, only 1 in 200 require
treatment

A

Only 1 in 10 cases at risk of ABO related disease develop HDN, only 1 in 200 require
treatment

Erythroblastosis fetalis is defined as a hemolytic disease in the newborn caused by blood-group incompatibility between mother and child When the fetus inherits red cell antigenic determinants from the father that are foreign to the mother, a maternal immune reaction may occur, leading to hemolytic disease in the infant. Basic to
such a phenomenon are leakage of fetal red cells into the maternal circulation and, in turn,
transplacental passage of maternal antibodies into the fetus. Any of the numerous red cell antigenic
systems may theoretically be involved, but the major antigens known to induce clinically significant immunologic disease are the ABO and certain of the Rh antigens.
Concurrent ABO incompatibility protects the mother against Rh immunization because the fetal red cells are promptly coated by isohemagglutinins and removed from the maternal circulation.
The antibody response depends on the dose of immunizing antigen; hence, hemolytic disease develops only when the mother has experienced a significant transplacental bleed (more than 1 ml of Rh-positive red cells).
The istotype of the antibody is important because immunoglobulin G (IgG) (but not immunoglobulin M [IgM]) antibodies can cross the placenta. The initial exposure to Rh antigen evokes the formation of IgM antibodies, so Rh disease is uncommon with the first pregnancy. Subsequent
exposure during a second or third pregnancy generally leads to a brisk IgG antibody response.
The pathogenesis of fetal hemolysis caused by maternal/fetal ABO incompatibility is slightly different than that caused by differences in the Rh antigens.
ABO incompatibility occurs in approximately 20 to 25% of pregnancies, but laboratory evidence of hemolytic disease occurs only in 1 in 10 of such infants, and the hemolytic disease is severe enough to require treatment in only 1 in 200 cases.
Several factors account for this.
First, most anti-A and anti-B antibodies are of the IgM type and hence do not cross placenta.
Second, neonatal red cells express blood group antigens A and B poorly.
Third, many cells other than red cells express A and B antigens and thus sop up some of the transferred antibody.
ABO hemolytic disease occurs almost exclusively in infants born to group O mothers because some have IgG anti-A and anti-B antibodies without obvious sensitization.
There is no effective protection against ABO reactions.

345
Q

A 24-year-old woman with lymphoma is seriously unwell after 2 weeks of ,
headache. Over the last 24 hours she has developed blackened areas of skin over her face, and a right proptosis. CT shows non-specific oedema of the face and orbit, diffuse sinus opacity, and absence of flow in the left cavernous sinus. Cultures of nasal swabs show non-septate irregular hyphae with right angle branches. The most likely organism is (–) (TW)
1.Apergillosus
2.Mucormycosis
3.Cryptococcus
4.Candida
5.Listeria

A

Mucormycosis – T – Phycomycetes (Mucor, Absidia, Rhizopus, Cunninghamella) form
nonseptate, irregularly wide (6-50um) fungal hyphae with frequent right-angle branching.

1.Apergillosus – F – aspergillus form fruiting bodies (esp in cavities) and septate filaments,
5-10um thick, branching at acute angles (40 degrees)
3.Cryptococcus – F – encapsulated yeast. Produced buds that characteristically are narrowbased and the organism is surrounded by a polysaccharide capsule.
4.Candida – F – polymorphic yeast (ie yeast cells, hyphae and pseudohyphae are produced). Pseudohyphae may be formed from budding yeast cells that remain attached to each other.
5.Listeria – F – gram positive, motile facultative intracellular bacterium.Gram positive rods.

Mucormycosis is an opportunistic infection of neutropenic persons and ketoacidotic diabetics caused by “bread mold fungi,” including Mucor, Absidia, Rhizopus, and Cunninghamella, which are collectively referred to as the Phycomycetes.
These fungi, which are widely distributed in nature and cause no harm to immunocompetent individuals, infect immunosuppressed patients somewhat less frequently than do Candida or
Aspergillus.
The three primary sites of Mucor invasion are the nasal sinuses, lungs, and gastrointestinal tract, depending on whether the spores (widespread in dust and air) are inhaled or ingested. In diabetics, the fungus may spread from nasal sinuses to the orbit, and brain, giving rise to rhinocerebral mucormycosis.
The Phycomycetes cause local tissue necrosis, invade arterial walls, and penetrate the periorbital tissues and cranial vault.
Meningoencephalitis follows, sometimes complicated by cerebral infarctions when fungi invade arteries and induce thrombosis. Phycomycetes form nonseptate, irregularly wide (6 to 50 mum) fungal hyphae with frequent right-angle branching, which are readily demonstrated in the necrotic
tissues by hematoxylin and eosin or special fungal stains
Lung involvement with Mucor may be secondary to rhinocerebral disease, or it may be primary in patients with hematologic neoplasms. The lung lesions combine areas of hemorrhagic pneumonia with vascular thrombi and distal infarctions.

346
Q

Procedure planned, what is the least likely risk factor for DIC: (–)
1.Prostatic cancer
2.Giant haemangioma
3.Sarcoidosis
4.Placental abruption

A

sarcoid

347
Q

Follicular lymphoma – T/F (TW)
1.Common 15-40 year olds
2.Nodular growth, follicular B cells
3.T(14;18) translocation
4.Transformation occurs to diffuse large B-cell lymphoma in 30 – 50%, rarely to aggressive form of lymphoblastic lymphoma or leukaemia
5.Is characterised by cells in stroma that can change to diffuse appearance
6.low grade on old working classification
7.Is indolent but incurable
8.Good survival
9.Poor chemo response
10.commonly arises in gut

A

1.Common 15-40 year olds – F - frequently presents in middle-aged individuals and the
elderly with a median age at Dx of 60.

2.Nodular growth, follicular B cells – T - lymphoma of follicle center B-cells (centrocytes and centroblasts), which has at least a partially follicular pattern.

3.T(14;18) translocation – T – Approx 85% of pts with FL have t(14;18) which results in the
overexpression of BCL-2, an oncogene that blocks programmed cell death.

4.Transformation occurs to diffuse large B-cell lymphoma in 30 – 50%, rarely to aggressive form of lymphoblastic lymphoma or leukaemia – T

5.Is characterised by cells in stroma that can change to diffuse appearance – T - 30-50%
transformation occurs to diffuse large B-cell lymphoma

6.low grade on old working classification – T – considered an indolent NHL, but incurable.

7.Is indolent but incurable - T

8.Good survival - ?T?F – depends on Early or Late stage presentation, plus treatment.
Proportion also have transformation to higher grade lymphoma per year.

9.Poor chemo response – T

10.commonly arises in gut – F – involvement of organs other than the lymphatic organs or
bone marrow is uncommon

Follicular lymphoma is the most common form of NHL in the United States, comprising about 45% of adult lymphomas. It usually presents in middle age and afflicts men and women equally. It is less common in Europe and rare in Asian populations. The neoplastic cells closely resemble normal germinal center B cells.
The overall median survival is 7 to 9 years and is not improved by aggressive therapy hence, the usual clinical approach is to palliate patients with low-dose chemotherapy or radiation when they become symptomatic
Although follicular lymphoma is incurable, it often follows an indolent waxing and waning course.

348
Q

Regarding lymphoma (TW & GC)
1.Lymphocyte deplete is the most common form
2.RS cell is derived from the B cell
3.HD and NHD often coexist
4.FNA cannot diagnose HD as it is the architecture of the lymph node that makes the diagnosis

A

RS cell is derived from the B cell - T – probably best answer seeing they emphasise this point at the path course. Neoplastic RS cells are now considered to be B-cells, typically CD30 / CD15 positive, and CD 20 / EMA / CD45 negative. They are not classified as B-cell lymphomas as they don’t act like them. Molecular studies have established a B-cell origin for RS cells in the vast majority of cases of HL.

1.Lymphocyte deplete is the most common form - F - rare entity. Nodular sclerosis type is
most common.
3.HD and NHD often coexist - F - can occur, but not often. “Composite lymphoma” - term
used to denote 2 distinct types of lymphoma occurring within a single organ or tissue.
Synchronous occurrence of 2 types of NHLs is more common than an NHL and HL (But
occurs).
4.FNA cannot diagnose HD as it is the architecture of the lymph node that makes the diagnosis – F(?T) - can Dx with FNA (RS cells). Lymph node excision Bx is used for
determining type which is based on morphologic and immunophenotypic features.

349
Q

60 yr old female with follicular lymphoma which is false? (TW)
1.Involves liver and spleen
2.Follicles and nodules on histology
3.Disease waxes and wanes with treatment
4.Atypical sex and age

A

Atypical sex and age – F – frequently presents in middle-aged individuals and the elderly
with a median age at Dx of 60. Slight female predominance (1.7:1 F:M). Rarely, follicular
lymphoma arises in children or adolescents.

1.Involves liver and spleen – T – involves spleen in 50% and liver in 50% of cases.
2.Follicles and nodules on histology – T – lymphoma of follicle center B-cells (centrocytes and centroblasts), which has at least a partially follicular pattern.
3.Disease waxes and wanes with treatment - T

350
Q

Which is not a type of Hodgkins lymphoma? (GC, TW)
1.Lymphocyte rich
2.Lymphocyte predominant
3.Lymphocyte depleted -
4.Mixed cellularity
5.Anaplastic

A

anaplastic

anaplastic large cell lymphoma (usually made up of T-lymphocytes)
REAL/WHO classification differs from older classifications in several ways. Most important is the recognition that there are two distinct group of diseases within the HLs:
“Classical” HL, which includes the nodular sclerosis HL, mixed cellularity HL, lymphocyte rich classical HL, and lymphocyte depleted HL subtypes. Nodular lymphocyte predominant HL.

351
Q

A 53-year-old female has increasing congestive cardiac failure and normal
vessels at coronary angiography. As a child she suffered rheumatic fever. The cardiac valves
most likely to be affected are: (JS)
1.Aortic alone
2.Mitral and aortic
3.Mitral and pulmonic
4.Aortic and pulmonic
5.Tricuspid and mitral

A

Mitral and aortic - T - Rheumatic heart disease is overwhelmingly the most frequent cause
of mitral stenosis (99% of cases). In patients with rheumatic heart disease, the mitral valve
alone is involved in 65 to 70% of the cases, mitral and aortic in about 25%. Similar but
generally less severe fibrous thickenings and stenoses can occur in the tricuspid valve and
rarely in the pulmonary.

352
Q

A 12-year-old male dies after a motor vehicle accident. At autopsy, mutiple yellow elongated lesions are seen on the intima of his otherwise normal abdominal aorta. Which of the following statements about this entity IS LEAST correct?
1.Coronary lesions are less common than aortic lesions
2.They are composed of lipid-filled foam cells and T lymphocytes
3.More common in Western children than in Africa
4.Can be seen in children less than 1 year of age
5.Not all progress to fibrous atheromatous plaques

A

More common in Western children than in Africa - F - Fatty streaks occur in all children
older than 10y regardless of geography, race, sex or environment.

1.Coronary lesions are less common than aortic lesions - T - Coronary fatty streaks are less common than aortic, beginning to form in adolescence, and they occur at the same anatomic sites that are later prone to develop plaques
2.They are composed of lipid-filled foam cells and T lymphocytes - T - composed of lipidfilled foam cells with T lymphocytes and extracellular lipid present in smaller amounts than in plaques
4.Can be seen in children less than 1 year of age - T
5.Not all progress to fibrous atheromatous plaques T - although fatty streaks may be
precursors of plaques, not all fatty streaks are destined to become fibrous plaques or more
advanced lesions)

353
Q

Which of the following IS NOT a correct regarding polyarteritis nodosa? (JS)
1.There is a transmural necrotising vasculitis of small to medium size arteries
2.There is associated glomerulonephritis
3.There is sparing of the pulmonary circulation
4.It is associated with Hepatitis B antigen positivity in over 20% of cases
5.Remission or cure is achieved in 90% with steroid and cyclophosphamide

A

There is associated glomerulonephritis - F - Renal involvement is one of the prominent
manifestations, BUT because there is no small vessel involvement, glomerulonephritis is not present.

1.There is a transmural necrotising vasculitis of small to medium size arteries - T - Systemic
vasculitis manifested by transmural necrotizing inflammation of small or medium-sized muscular arteries, typically renal and visceral, sparing pulmonary circulation. The
common manifestations are malaise, fever,weight loss, hypertension, abdominal pain, malaena, muscle aches, peripheral neuritis. Little association with ANCA. Angiography shows aneurysms or occlusions of the main visceral arteries in 50% of cases
3.There is sparing of the pulmonary circulation - T
4.It is associated with Hepatitis B antigen positivity in over 20% of cases - T - 30% patients have hepatitis B antigen in serum
5.Remission or cure is achieved in 90% with steroid and cyclophosphamide - T - Disease is
fatal if untreated but therapy is very effective.

354
Q

Which of the following would be an UNEXPECTED finding following coronary
angioplasty? (JS)
1.Luminal expansion
2.Plaque rupture
3.Thickened intact intima
4.Medial dissection
5.Proliferative restenosis in 30-50% of patients at 6 months

A

Thickened intact intima - F - The split encompasses the intima and media

1.Luminal expansion - T - The key elements of luminal expansion in angioplasty are plaque
rupture, medial dissection and stretching of the media of the dissected segment.
2.Plaque rupture - T
4.Medial dissection - T
5.Proliferative restenosis in 30-50% of patients at 6 months - T - The long-term success of angioplasty is limited by the development of proliferative restenosis that occurs in approximately 30 to 50% of patients within the first 4 to 6 months after angioplasty

355
Q

A 33-year-old woman has sudden onset of blindness in her left eye. The
multislice CT angiogram shows marked diffuse narrowing of her brachiocephalic, subclavian and common carotid arteries. There is also involvement both of her external and internal carotid arteries. This distribution is most suggestive of: (JS)
1.Takayasu arteritis
2.Atherosclerosis associated with familial hypercholesterolaemia
3.Giant cell arteritis
4.Polyarteritis nodosa -
5.Fibromuscular hyperplasia

A

Takayasu arteritis - T - clinical syndrome characterized principally by ocular disturbances and marked weakening of the pulses in the upper extremities (pulseless disease), related to fibrous thickening of the aortic arch with narrowing or virtual obliteration of the origins or more distal portions of the great vessels arising in the arch. It has been reported in most areas of the world, including the United States. The illness is seen predominantly in women
younger than 40 years old

2.Atherosclerosis associated with familial hypercholesterolaemia - F - causes premature
atherosclerosis in the same distribution as non-familial cases (ie abdominal aorta, ostia of major branches, coronary arteries, popliteal, descending thoracic, ICA and circle of Willis)
3.Giant cell arteritis - F - most common of the vasculitides, is an acute and chronic, often
granulomatous, inflammation of medium-sized and small arteries. It affects principally the arteries in the head– especially the temporal arteries– but also the vertebral and
ophthalmic arteries. The latter may lead to blindness. In other expressions of this disorder, lesions have been found in arteries throughout the body, and in some cases the aortic arch has been involved to produce so-called giant cell aortitis. The disease is most common in older individuals and rare before the age of 50
4.Polyarteritis nodosa - F - necrotising inflammation of medium sized or small arteries without GN or vasculitis within arterioles, capillaries or venules.
5.Fibromuscular hyperplasia - F - proliferation of the muscular and fibrous elements in
middle and large sized arteries of unknown aetiology. Affects renal arteries, ICA or
vertebral, iliac, visceral.

356
Q

Which of the following conditions IS NOT a cause (or association) of dissection ?
Rob 341 (JS)
1.Hypertension
2.Arteriosclerosis
3.Pregnancy
4.Marfan’s syndrome
5.Cystic medial degeneration

A

Arteriosclerosis - F - medial scarring obstructs advancement of a dissecting aneurysm

1.Hypertension - T - more than 90% of cases
3.Pregnancy - T - for unknown causes occurs shortly after pregnancy
4.Marfan’s syndrome - T - and other connective tissue disorders (Ehlers-Danlos syndrome, Turner’s) are associated with Cystsic Medial Degeneration
5.Cystic medial degeneration - T - CMD consists of elastic fragmentation and separation of the elastic and fibromuscular elements of the tunica media. This is the most frequently preexisting histologically detectable lesion in dissection but it can be found incidentally at autopsy in patients free from dissection.

357
Q

Kawasaki disease – which of the following is FALSE? Rob p350 (JS)
1.Mostly occur in children younger than 2 years of age
2.Is self-limited disorder with less than 1% death rate
3.>20% develop CVS sequale
4.characterised by recurrent attacks

A

characterised by recurrent attacks - F - Kawasaki syndrome is an arteritis involving large, medium and small arteries (often coronaries), associated with the mucocutaneous lymph node syndrome (fever, conjunctival and oral erythema and erosion, oedema of hands and feet, rash and cervical lymph nodes), usually in young children and infants. It is self limited.

KAWASAKI SYNDROME (MUCOCUTANEOUS LYMPH NODE SYNDROME)
arteritis involving large, medium-sized, and small arteries (often the coronary arteries)
associated with the mucocutaneous lymph node syndrome, usually in young children and infants (80% younger than 4 years old).
The acute illness is manifested by fever, conjunctival and oral erythema and erosion, edema of the hands and feet, erythema of the palms and soles, a skin rash often with desquamation, and enlargement of cervical lymph nodes
It is usually self-limited
Epidemic in Japan, the disease has also been reported in Hawaii and increasingly in the United States
Approximately 20% of patients develop cardiovascular sequelae, with a range of severity from asymptomatic vasculitis of the coronary arteries, coronary artery ectasia, or aneurysm formation to giant coronary artery aneurysms (7 to 8 mm) with rupture or thrombosis, myocardial infarction, or
sudden death.
Kawasaki syndrome is the leading cause of acquired heart disease in children in the United States
Acute fatalities occur in approximately 1% of patients owing to coronary arteritis with
superimposed thrombosis or ruptured coronary artery aneurysm.
Pathologic changes outside the cardiovascular system are rarely significant.
Although the vasculitis resembles that of polyarteritis nodosa, with necrosis and pronounced inflammation affecting the entire thickness of the vessel wall, fibrinoid necrosis is usually less prominent in Kawasaki syndrome

1.Mostly occur in children younger than 2 years of age - T - occurs in young children and
infants with 80% younger than 4 years of age
2.Is self-limited disorder with less than 1% death rate - T - acute fatalities occur in 1% of
patients owing to coronary arteritis with superimposed thrombosis or ruptured coronary artery aneurysm
3.>20% develop CVS sequale - T - approximately 20% develop CVS sequelae including vasculitis of coronary arteries, ectasia, aneurysm formation with rupture, thrombosis, MI or
sudden death

358
Q

Features of cystic lymphangioma do NOT include ? Rob p357
1.axillary location
2.child 18 months old
3.large cystic spaces
4.benign lesion
5.encapsulated

A

encapsulated - F - The margins of the tumor are not discrete, and these lesions are not
encapsulated. Their removal can therefore be difficult.

1.axillary location - T - Almost invariably occur in children in the neck or axilla (rarely
retroperitoneal)
2.child 18 months old - T - Lymphangiomas are congenital malformations with 50-65%
present at birth and 80-90% evident by 2 years
3.large cystic spaces - T - made up of massively dilated cystic spaces lined by endothelial cells and separated by scant connective tissue stroma
4.benign lesion - T

359
Q

Regarding polyarteritis nodosa. Which of the following is false: (GC)
1.The kidneys are most commonly involved in autopsy
2.Aneurysms are seen at angiography in 50%
3.70% are seropositive for Hepatitis B surface antigen
4.Churg Strauss syndrome frequently involves pulmonary and splenic vessels
5.Serum anti-neutrophil antibody titres correlate with disease activity

A

70% are seropositive for Hepatitis B surface antigen - F - 30% have hepatitis B
antigenaemia, and HBsAg-HBsAb immune complexes can be demonstrated in the lesions.

1.The kidneys are most commonly involved in autopsy - T - vessels of the kidneys, heart,
liver, and GIT are involved in descending order.
2.Aneurysms are seen at angiography in 50% - T - multiple small intrarenal microaneurysms typically at branchpoints in 50-60% - hallmark feature. Lesions usually
involve only part of the vessel circumference; the inflammatory process weakens the arterial wall and can lead to aneurysms or even rupture.
4.Churg Strauss syndrome frequently involves pulmonary and splenic vessels - T - aka
allergic granulomatosis and angiitis; vascular lesions can resemble PAN and microscopic
polyangiitis, but in the lung, heart, spleen, peripheral nerves and skin there are also intraand extravascular granulomas. PAN spares the pulmonary circulation.
5.Serum anti-neutrophil antibody titres correlate with disease activity - T - A positive ANCA argues against PAN and in favor of one of the ANCA-associated vasculitides (UpToDate), however Adelaide path notes state that p-ANCA is often present in serum and correlates with disease activity. eMed: ANCA are often present in the serum of patients with PAN; in such patients ANCA levels often correlate with disease activity. However ANCA can be present in patients with other vasculitides (Churg-Strausss, Wegener, microscopic polyangitis)

No correlation with ANCA, Robbins doesn’t mention ANA, and I am pretty sure this question is specific to wegners in which it is correlated with disease activity. So both c and e are incorrect. I think c is the least controversial false answer, but there are so many new classifications of PAN that
the waters get very muddy. Perhaps it is a bad recall.

360
Q

Concerning risk factors for atherosclerosis, which of the following is LEAST CORRECT:
(JS)
1.Most striking association is with elevated serum levels of LDL
2.After 45, hypertension is a greater risk factor than hypercholesterolaemia
3.Smoking increases incidence of sudden death among victims of heart attacks
4.There is a two fold increase in the incidence of MI in diabetics vs non-diabetics
5.Systolic hypertension is more important than diastolic hypertension

A

Systolic hypertension is more important than diastolic hypertension - F - Both systolic and
diastolic levels are important in increasing risk

361
Q

Regarding aortic coarctation, which of the following IS NOT a well
recognised association: (TW)
1.PDA –
2.Ventriculoseptal defect
3.Dural AVM
4.Berry aneurysms of the circle of Willis
5.Bicuspid aortic valve

A

dural avm

Coarctation – 50% associated with
bicuspid arotic valve (in 25-50%)
intracardiac malformations – PDA, VSD, AS, AR, ASD, TGV, ostium primum defect, truncus
arteriosus, double-outlet RV.
Non-cardiac malformations. Turner syndrome.
Cerebral berry aneurysms
Mycotic aneurysm distal to CoA.

362
Q

A couple attend for an obstetric scan at 37 weeks. The wife is concerned, as
she was exposed to rubella at 7 weeks. Blood samples confirmed seroconversion at that time but they declined any further counselling. Which of the following is the most correct statement? Rob p 251 (TW)
1.There is “no risk” (less than 0.01%) of tetatogenesis from infection at 7 weeks
2.There is minimal (< 2%) risk of congenital abnormality, as “successful” infection at seven weeks usually causes abortion
3.There is a 5% chance of cataract, deafness or cardiac defect
4.There is a 20% chance of cataract, deafness or cardiac defect
5.There is a 70% chance of cataract, deafness or cardiac defect

A

There is a 70% chance of cataract, deafness or cardiac defect

Maternal-fetal transmission occurs via hematogenous spread and varies with gestational age. Virus probably spreads through vascular system of developing fetus after infecting the placenta.
In the first TM, fetal infection rates up to 81%, dropping to 25% in 2nd TM, then increasing to 35%
at 27-30 weeks, to nearly 100% for fetuses exposed beyond 36wks.
However, risk of congenital defects after maternal infection is essentially limited to maternal infection in the first 16 weeks of pregnancy. Little, if any, risk of congenital rubella syndrome is associated with infection after 20wks gestation.
Congenital Rubella Syndrome is the presence of any of the follow defects or laboratory data, or a persistently elevated IgG titer that fails to drop 2-fold per month.
Audiologic defects, predominantly sensorineural deafness (60-75%)
Ophthalmologic abnormalities (10-25%) particularly cataracts, pigmentary and congenital glaucoma.
Cardiac malformations (10-20%) such as PDA or peripheral pulmonary artery stenosis
Neurologic sequelae (10-25%) ranging from meningioencephalitis to behaviour disorders and mental retardation.

363
Q

Short female patient with webbed neck and previous coarctation repair presents
for thyroid ultrasound, MOST LIKELY diagnosis ? Rob p233 (–)
1.Turners with thyroid adenoma
2.Turners with Hashimotos
3.Familial medullary carcinoma
4.Men II
5.Achondraplasia

A

hashys

Turners
Autoantibodies to thyroid in 50%  hypothyroidism in 50% of these
Approximately 1/3 of individuals with Turner syndrome have a thyroid disorder, usually
hypothyroidism.
Symptoms of this condition include decreased energy, dry skin, cold-intolerance and poor growth.
In most cases, it is caused by an immune system attack on the thyroid gland (also known as Hashimoto’s thyroiditis).
Although it is not known why thyroid disorders occur with a high frequency in Turner syndrome, the condition is easily treated with thyroid hormone supplements.

364
Q

439

A